Coordination Compound

Download as docx, pdf, or txt
Download as docx, pdf, or txt
You are on page 1of 76

Chapter – 9

COORDINATION COMPOUNDS

TRANSITION ELEMENTS
Three series of elements are formed by filling the 3d, 4d and 5dsubshells of electrons. Together
these comprise the dblock elements. They are often called ‘transition elements’ because their position in the periodic
table is between the sblock and pblock elements. Their properties are transitional between the highly reactive
metallic elements of the sblock, which typically forms ionic compounds and the elements of the pblock, which are
largely covalent. In the s and pblocks, electrons are added to the outer shell of the atom. In the dblock, electrons are
added to the penultimate shell, expanding it from 8 to 18 electrons. Typically the transition elements have an
incompletely filled dlevel. Group 12 (the zinc group) has a d 10 configuration and since the dsubshell is complete,
compounds of these elements are not typical and show some differences from the others. The transition elements make
up three complete rows of ten elements and an incomplete fourth row.
Thus, the transition elements are defined as those elements, which have partly filled
dorbitals, as elements and in any of their important compounds.
The general electronic configuration of the dblock elements can be represented as,
(n1)d19ns12
Depending on the subshell getting filled up the transition elements form three series.
The first transition series contain the elements from Sc (Z = 21) to Zn (Z = 30) and the 3dorbital gets filled up in this
series. In the second series, the 4dorbital gets filled up from Y (Z = 39) to Cd
(Z = 48). The 5dorbital from La (Z = 57) to Hg (Z = 80) gets filled up for the elements of the third series. The fourth
series starting with Ac is incomplete.

Group No. 3 4 5 6 7 8 9 10 11 12
At. No. (Z) 21 22 23 24 25 26 27 28 29 30
Symbol Sc Ti V Cr Mn Fe Co Ni Cu Zn
At. No. (Z) 39 40 41 42 43 44 45 46 47 48
Symbol Y Zr Nb Mo Tc Ru Rh Pd Ag Cd
At. No. (Z) 57 72 73 74 75 76 77 78 79 80
Symbol La * Hf Ta W Re Os Ir Pt Au Hg
At. No. (Z) 89
Symbol Ac *

14 Lanthanide elements
14 Actinide elements

Unlike the s and pblock elements of the same period, the dblock elements do not show much variation in
properties, both chemical and physical. This is because these elements differ only in the number of electrons in the
penultimate dshell. The number of electrons in the valence shell remains the same, ns 2, for most of the elements.

2 METALLIC CHARACTER
In the dblock elements, the penultimate shell of electrons is expanding. Thus, they have many physical and
chemical properties in common. Thus, all the transition elements are metals. They are good conductors of heat and

India’s first Colour STUDY MATERIALS for JEE, NEET ,OUAT & OTHERS ENTRANCE EXAMINATION Page 1
electricity, have a metallic luster and are hard, strong and ductile. They also form alloys with other metals. Copper
exceptionally is both soft and ductile and relatively noble.

3 VARIABLE OXIDATION STATE


One of the most striking features of the transition elements is that the elements usually exist in several
different oxidation states and the oxidation states change in units of one.
For example: Fe3+ and Fe2+, Cu2+ and Cu+ etc.
The oxidation states shown by the transition elements may be related to their electronic configurations.
Calcium, the sblock element preceding the first row of transition elements, has the electronic configuration:
Ca (Z = 20): 1s22s22p63s23p64s2 : [Ar]4s2
It might be expected that the next ten transition elements would have this electronic arrangement with from
one to ten delectrons added in a regular way: 3d1, 3d2, 3d3…3d10. This is true except in the cases of Cr and Cu. In
these two cases, one of the selectrons moves into the dsubshell, because of the additional stability of the exactly
halffilled or completely filled dorbital. Since the energies of (n1) d and nsorbitals are nearly equal, the transition
elements exhibit variable oxidation states. The oxidation states of the dblock elements are listed below.

Sc Ti V Cr Mn Fe Co Ni Cu Zn
Electronic
3d14s2 3d24s2 3d34s2 3d44s2 3d54s2 3d64s2 3d74s2 3d84s2 3d94s2 3d104s2
configuratio
n
3d54s1 3d104s1
Oxidation
I I
states
II II II II II II II II II II
III III III III III III III III III
IV IV IV IV IV IV IV
V V V V V
VI VI VI
VII

Thus, Sc could have an oxidation state of (II) if both selectrons are used for bonding and (III) when
two s and one delectrons are involved. Ti has an oxidation state (II) when both selectrons are used for bonding, (III)
when two s and one delectrons are used and (IV) when two s and two delectrons are used. Similarly, V shows
oxidation numbers (II), (III), (IV) and (V). In the case of Cr, by using the single s electron for bonding, we get an
oxidation number of (I); hence by using varying number of delectrons, oxidation states of (II), (III), (IV), (V) and
(VI) are possible. Mn has oxidation states (II), (III), (IV), (V), (VI) and (VII). Among these first five elements, the
correlation between electronic configuration and minimum and maximum oxidation states is simple and straight
forward. In the highest oxidation states of these first five elements, all of the s and delectrons are being used for
bonding. Thus, the properties depend only on the size and valency.
Once the d5 configuration is exceeded, i.e. in the last five elements, the tendency for all the
delectrons to participate in bonding decreases. Thus, Fe has a maximum oxidation state of (VI). However, the second
and third elements in this group attain a maximum oxidation state of (VIII), as in RuO 4 and OsO4. This difference
between Fe and the other two elements (Ru and Os) is attributed to the increased size and decreased attraction with the
nucleus.
The oxidation number of all elements in the elemental state is zero. In addition, several of the elements have
zerovalent and other lowvalent states in complexes. Low oxidation states occur particularly with bonding ligands
such as carbon monoxide and dipyridyl.
Some other important features about the oxidation states of transition elements
1. In group 8 (the iron group), the second and third row elements show a maximum oxidation state of (VIII)
compared with (VI) for Fe.
2. The electronic configurations of the atoms in the second and third rows do not always follow the pattern of the
first row. The configurations of group 10 elements (the nickel group) are:
Ni (Z = 28) : 3d84s2

India’s first Colour STUDY MATERIALS for JEE, NEET ,OUAT & OTHERS ENTRANCE EXAMINATION Page 2
Pd (Z = 46) : 4d105s0
Pt (Z = 78) : 5d96s1
3. Since a full shell of electrons is a stable arrangement, the place where this occurs is of importance in the
transition series. The dlevels are complete at copper, palladium and gold in their respective series.
Ni Cu : 3d104s1 Zn : 3d104s2
Pd : 4d105s0 Ag : 4d105s1 Cd : 4d105s2
Pt Au : 5d106s1 Hg : 5d106s2
4. Even though the ground state of the atom has a d 10 configuration, Pd and the coinage metals Cu, Ag and Au
behave as typical transition elements. This is because in their most common oxidation states, Cu(II) has a d 9
configuration and Pd(II) and Au(III) have d 8 configurations, that is they have an incompletely filled dlevel.
However, in zinc, cadmium and mercury, the ions Zn 2+, Cd2+ and Hg2+ have a d10 configuration. Because of
this, these elements do not show the properties characteristic of transition elements.
Compounds are regarded as stable if they exist at room temperature, are not oxidized by the air,
are not hydrolysed by water vapour and do not disproportionate or decompose at normal temperatures. Within each of
the transition metals of groups 312, there is a difference in stability of the various oxidation states that exist. In
general, the second and third row elements exhibit higher coordination numbers and their higher oxidation states are
more stable than the corresponding first row elements. Stable oxidation states form oxides, fluorides, chlorides,
bromides and iodides. Strongly reducing states probably do not form fluorides and/or oxides, but may well form the
heavier halides. Conversely, strongly oxidizing states form oxides and fluorides, but not iodides.
Oxides and halides of some elements of the first row:
Cr Mn Fe
IIO CrO MnO FeO
F CrF2 MnF2 FeF2
Cl CrCl2 MnCl2 FeCl2
Br CrBr2 MnBr2 FeBr2
I CrI2 MnI2 FeI2
III O Cr2O3 Mn2O3 Fe2O3
F CrF3 MnF3 FeF3
Cl CrCl3  FeCl3
Br CrBr3  FeBr3
I CrI3  
IV O CrO2 MnO2 
F CrF4 MnF4 
Cl CrCl4  
Br CrBr4  
I CrI4  
V F CrF5  
VI O CrO3  
F CrF6  
VII O  Mn2O7 

4 ATOMIC AND IONIC RADII


The covalent radii of the elements decrease from left to right across a row in the transition series,
until near the end when the size increases slightly. On passing from left to right, extra protons are placed in the nucleus
and extra orbital electrons are added. The orbital electrons shield the nuclear charge incompletely (d electrons shield
less efficiently than pelectrons, which in turn shield less effectively than selectrons). Because of this poor screening
by delectrons, the nuclear charge attracts all of the electrons more strongly, hence a contraction in size occurs.
The elements in the first group in the dblock (group 3) show the expected increase in size Sc  Y 
La. However, in the subsequent groups (412) there is an increase in radius of 0.1  0.2 Å between the first and
second member, but hardly any increase between the second and third elements. This trend is shown both in the
covalent radii and in the ionic radii. Interposed between lanthanum and hafnium are the 14 lanthanide elements, in
which the antipenultimate 4fsubshell of electrons is filled.

India’s first Colour STUDY MATERIALS for JEE, NEET ,OUAT & OTHERS ENTRANCE EXAMINATION Page 3
There is a gradual decrease in size of the 14 lanthanide elements from cerium to lutetium. This is called the
“lanthanide contraction”. The lanthanide contraction cancels almost exactly
the normal size increase on descending a group of transition elements. Therefore, the second and third row transition
elements have similar radii. As a result they also have similar lattice energies, solvation energies and ionization
energies. Thus, the differences in properties between the first row and second row elements are much greater than the
differences between the second and third row elements. The effects of the lanthanide contraction are less pronounced
towards the right of the dblock. However, the effect still shows to a lesser degree in the pblock elements that follow.
Covalent radii of the transition elements (in Å)
K Ca Sc Ti V Cr Mn Fe Co Ni Cu Zn
1.57 1.74 1.44 1.32 1.22 1.17 1.17 1.17 1.16 1.15 1.17 1.25
Rb Sr Y Zr Nb Mo Tc Ru Rh Pd Ag Cd
2.16 1.91 1.62 1.45 1.34 1.29  1.24 1.25 1.28 1.34 1.41
Cs Ba La Hf Ta W Re Os Ir Pt Au Hg
2.35 1.98 * 1.44 1.34 1.30 1.28 1.26 1.26 1.29 1.34 1.44
1.69

14 Lanthanide elements

5 DENSITY
The atomic volumes of the transition elements are low compared with elements in neighbouring
groups 1 and 2. This is because the increased nuclear charge is poorly screened and so attracts all the electrons more
strongly. In addition, the extra electrons added occupy inner orbitals. Consequently, the densities of the transition
metals are high. Practically, most of the elements have a density greater than 5 g cm 3. (The only exceptions are Sc:
3.0 g cm3 and Y
3
and Ti: 4.5 g cm ). The densities of the second row elements are high and third row values are even higher. The two
elements with the highest densities are osmium: 22.57 g cm 3 and iridium: 22.61 g cm3. Thus, iridium is the heaviest
element among all the elements of the periodic table.

6 MELTING AND BOILING POINT

The melting and boiling points of the transition elements are generally very high. Transition
elements typically melt above 1000°C. Ten elements melt above 2000°C and three melt above 3000°C (Ta: 3000°C,
W: 3410°C and Re: 3180°C). There are a few exceptions.
For example, La and Ag melts just under 1000°C (920°C and 960°C respectively). Other notable exceptions are
Zn(420°C), Cd(320°C) and Hg, which is liquid at room temperature and melts at 38°C. The last three behave
atypically because the dsubshell is complete and delectrons do not participate in metallic bonding.

7 IONISATION ENERGY
In a period, the first ionization energy gradually increases from left to right. This is mainly due to increase in
nuclear charge. Generally, the ionization energies of transition elements are intermediate between those of s and
pblock elements. The first ionization potential of the
5delements are higher than those of 3d and 4delements due to the poor shielding by 4felectrons.
From 3d  4d series, general trend is observed but not from 4d
 5d series because of incorporation of the 14 lanthanides elements between La and Hf. Third period of transition
elements have the highest ionisation energy. This reflects the fact that increase in radius due to addition of extra shell
is compensated by the decrease in radius due to lanthanide contraction.
As the radius of 4d and 5delements more or less remains the same,
due to which Zeff of elements of 5d series is higher, which results in high ionization energy of the 5delements of
transition series.

India’s first Colour STUDY MATERIALS for JEE, NEET ,OUAT & OTHERS ENTRANCE EXAMINATION Page 4
The ionisation energy values (in kJ/mole) of the transitions elements are given in the table
below:
3d series Sc Ti V Cr Mn Fe Co Ni Cu Zn
First I. E 631 656 650 652 717 762 758 736 745 906
4d series Y Zr Nb Mo Tc Ru Rh Pd Ag Cd
First I. E 616 674 664 685 703 711 720 804 731 876
5d series La Hf Ta W Re Os Ir Pt Au Hg
First I. E 541 760 760 770 759 840 900 870 889 1007

1000 5d elements
900
4d elements
First IE 800

700

600

500

Atomic no.

8 REACTIVITY OF METALS

Many of the metals are sufficiently electropositive to react with mineral


acids, liberating H2. A few have low standard electrode potentials and remain unreactive or noble. Noble character is
favoured by high enthalpies of sublimation, high ionization energies and low enthalpies of solvation. The high melting
points indicate high heats of sublimation. The smaller atoms have higher ionization energies, but this is offset by small
ions having high solvation energies. This tendency to noble character is most pronounced for the platinum metals (Ru,
Rh, Pd, Os, Ir, Pt) and gold.

9 FORMATION OF COMPLEX COMPOUNDS

The transition elements have characteristic tendency to form coordination compounds with Lewis bases, that
is with groups that are able to donate an electron pair. These groups are called ligands. A ligand may be a neutral
molecule such as NH3, or an ion such as Cl  or CN. Cobalt forms more complexes than any other element and forms
more compounds than any other element after carbon.
Co3+ + 6NH3 [Co(NH3)6]3+
Fe2+ + 6CN [Fe(CN)6]4
This ability to form complexes is in marked contrast to the s and pblock elements, which form
only a few complexes. The reason transition elements are so good at forming complexes is that they have small, highly
charged ions and have vacant low energy orbitals to accept lone pairs of electrons donated by other groups or ligands.
Complexes where the metal is in the (III) oxidation state are generally more stable than those where the metal is in the
(II) state.
Some metal ions form their most stable complexes with ligands in which the donor atoms are N, O
or F. Such metal ions include group 1 and 2 elements, the first half of the transition elements, the lanthanides and

India’s first Colour STUDY MATERIALS for JEE, NEET ,OUAT & OTHERS ENTRANCE EXAMINATION Page 5
actinides and the pblock elements except for their heavier members. These metals are called “classa acceptors” and
correspond to ‘hard’ acids.
In contrast the metals Rh, Ir, Pd, Pt, Ag, Au and Hg form their most stable complexes with the heavier elements of
groups 15, 16 and 17. These metals are called “classb acceptors” and correspond to ‘soft’ acids. The rest of the
transition metals and the heaviest elements in the pblock, form complexes with both types of donors and are thus
‘intermediate’ in nature.

Classification of elements on the basis of type of acceptors

Li Be B C N O
(a) (a) (a) (a) (a) 
Na Mg Al Si P S
(a) (a) (a) (a) (a) (a)
K Ca Sc Ti V Cr Mn Fe Co Ni Cu Zn Ga Ge As Se
(a) (a) (a) (a) (a) (a) (a) (a/b) (a/b) (a/b) (a/b) (a) (a) (a) (a) (a)
Rb Sr Y Zr Nb Mo Tc Ru Rh Pd Ag Cd In Sn Sb Te
(a) (a) (a) (a) (a) (a) (a/b (a/b) (b) (b) (b) (a/b (a) (a) (a) (a)
) )
Cs Ba La Hf Ta W Re Os Ir Pt Au Hg Tl Pb Bi Po
(a) (a) (a) (a) (a) (a) (a/b (a/b) (b) (b) (b) (b) (a/b) (a/b) (a/b) (a/b)
)
Fr Ra Ac
(a) (a) (a)

Ce Pr Nd Pm Sm Eu Gd Tb Dy Ho Er Tm Yb
(a) (a) (a) (a) (a) (a) (a) (a) (a) (a) (a) (a) (a)
Th Pa U Np Pu Am Cm Bk Cf Es Fm Md Mo
(a) (a) (a) (a) (a) (a) (a) (a) (a) (a) (a) (a) (a)
Common Co-Ordination Number Shown by Transition Elements of First Row
(i) Scandium
Sc3+ forms complexes with coordination number of 6. Examples of such complexes are [Sc(OH) 6]3, [ScF6]3
etc.
(ii) Titanium
Ti4+ forms complexes with a coordination number of 6. For example, [TiCl 6]2,
[Ti(SO4)3]2 etc.
(iii) Vanadium
V2+ forms mostly octahedral complexes (coordination number = 6), for example [V(H 2O)6]2+,
K4[V(CN)6].7H2O. But K4[V(CN)7].2H2O is also known with a pentagonal bipyramidal structure
(coordination number = 7).
V3+ forms octahedral complexes such as [V(H2O)6]3+.
V4+ is known to form square pyramidal complexes with a coordination number of 5. Example of such
complexes are [VOX4]2, [VO(OX)2]2, [VO(bipyridyl)2Cl]+ etc.
(iv) Chromium
Cr2+ forms octahedral complexes, such as [Cr(H2O)6]2+ and [Cr(NH3)6]2+ with coordination number 6.
Cr3+ forms octahedral complexes, such as [Cr(H2O)6]3+ and [Cr(H2O)5Cl]2+ with coordination number 6.
(v) Manganese
Mn2+ forms octahedral complexes such as [MnCl 6]4 and [Mn(en)3]2+ with coordination
number 6.
Mn3+ forms octahedral complexes such as K 3[Mn(CN)6] with coordination
number 6.
Mn4+ forms octahedral complexes such as K 2[MnF6] and K2[Mn(CN)6] with coordination
number 6.
(vi) Iron

India’s first Colour STUDY MATERIALS for JEE, NEET ,OUAT & OTHERS ENTRANCE EXAMINATION Page 6
Fe2+ forms mostly octahedral complexes like [Fe(H 2O)6]2+ but few tetrahedral halides with coordination
number 4 like [FeX4]2 are also known.
Fe3+ is known to form octahedral complexes such as [Fe(H2O)6]3+.
(vii) Cobalt
Co2+ is known to form both tetrahedral like [Co(H2O)4]2+] and octahedral such as [Co(H2O)6]2+ complexes.
Co3+ forms octahedral complexes. For example, [Co(NH 3)6]3+ and [Co(CN)6]3.

(viii) Nickel group


Ni2+ commonly forms octahedral and square planar complexes. Few tetrahedral, trigonal bipyramidal and
square based pyramidal structures are also formed.
Pd2+ and Pt2+ are all square planar.
Ni3+ forms octahedral compounds. For example, K 3[NiF6] and [Ni(en)2Cl2]Cl.
Pd4+ forms a few octahedral complexes like [PdX 6]2, where X = F, Cl or Br. These are generally reactive.
Halide complexes are decomposed by hot water, giving [PdX 4]2
and halogen. In contrast Pt 4+ forms large number of very stable octahedral complexes
like [PtCl6]2.
(ix) Copper, silver and gold
Cu+ forms tetrahedral complexes with Cl (for example, [Cu(Cl) 4]3) and linear complexes like [CuX2].
Cu2+ forms complexes both of coordination number 4 (like [CuX4]2) and of coordination number 6 {like
[Cu(en)3]2+, [Cu(H2O)3(NH3)3]2+}.
Ag+, Au+ forms complexes with coordination number 2 like ([M(CN)2].
(x) Zinc and cadmium
Zn2+ and Cd2+ forms both tetrahedral and octahedral complexes. For example, [MCl 4]2, [M(NH3)2Cl2],
[M(NH3)4]2+, [M(H2O)6]2+ etc.

10 COLOUR OF COMPLEX COMPOUNDS


Many ionic and covalent compounds of transition elements are coloured. In contrast compounds of the s and
pblock elements are almost always white. When light passes through a material, it is deprived of those wavelengths
that are absorbed. If wavelength of the absorption occurs in the visible region of the spectrum, the transmitted light is
coloured with the complementary colour to the colour of the light absorbed. Absorption in the visible and UV regions
of the spectrum is caused by changes in electronic energy. Thus, the spectra are sometimes called electronic spectra.
Colour may arise from an entirely different cause in ions with incomplete d or fsubshells. This source of
colour is very important in most of the transition metal ions.
In a free isolated gaseous ion, the five dorbitals are degenerate that is they are identical in energy. In actual
practice, the ion will be surrounded by solvent molecules if it is in solution, by other ligands if it is in a complex, or by
other ions if it is in a crystal lattice. The surrounding groups affect the energy of some dorbitals more than others.
Thus, the dorbitals are no longer degenerate and at their simplest they form two groups of orbitals of different
energy. Thus, in transition element ions with a partly filled dsubshell it is possible to promote electrons from one
dlevel to another dlevel of higher energy. This corresponds to a fairly small energy difference and so light it
absorbed in the visible region. The colour of a transition metal complex is dependent on how big the energy difference
is between the two dlevels. This in turn depends on the nature of the ligand and on the type of complex formed. Thus,
the octahedral complex [Ni(NH 3)6]2+ is blue, [Ni(H2O)6]2+ is green and [Ni(NO 2)6]4 is brownred. The colour changes
with the ligand used. The colour also depends on the number of ligands and the shape of the complex formed.
The source of colour in the lanthanides and the actinides is very similar, arising from
f  f transitions. With the lanthanides, the 4forbitals are deeply embedded inside the atom and are wellshielded
by the 5s and 5pelectrons. The felectrons are practically unaffected by complex formation. Hence, the colour
remains almost constant for the particular ion regardless of the ligand.
Some compounds of the transition metal are white, for example Cu 2Cl2, ZnSO4 and TiO2. In these compounds,
it is not possible to promote electrons within the dlevel. Cu+ and Zn2+ has a
d10 configuration and the dlevel is completely filled. Ti4+ has a d0 configuration and the dlevel is empty. In the series
Sc(III), Ti(IV), V(V), Cr(VI) and Mn(VII), these ions may all be considered to have an empty d subshell; hence dd
spectra are impossible and they should be colourless. However, as the oxidation state increases, these states become
India’s first Colour STUDY MATERIALS for JEE, NEET ,OUAT & OTHERS ENTRANCE EXAMINATION Page 7
increasingly covalent. Rather than forming highly charged simple ions, they form oxoions like TiO 2+, VO+2 ,
VO 3−
4
2−
, CrO 4
− +
and MnO4 . VO 2 is pale yellow, but CrO 4
2−
is strongly yellow coloured and MnO4

has an intense purple colour in solution, though the solid is almost black. The colour arises by charge transfer
mechanism.

In MnO4 , an electron is momentarily transferred from O to the metal, thus momentarily changing O 2 to O and
reducing the oxidation state of the metal from Mn(VII) to Mn(VI). Charge transfer requires the energy levels on the
two different atoms to be fairly close. Charge transfer always produces more intense colours than the colours
generated due to dd transitions. Charge transfer is also possible between metalion and metalion as seen in prussian
blue, Fe4[Fe(CN)6]3.
The s and pblock elements do not have a partially filled dsubshell, so there cannot be any dd transitions.
The energy required to promote an s or pelectron to a higher energy level is much greater and corresponds to
ultraviolet light being absorbed. Thus, compounds of s and pblock elements are typically not coloured.

11 MAGNETIC PROPERTIES
Compounds of the transition elements exhibit characteristic magnetic behaviour. Those, which
are attracted by a magnetic field, are termed as paramagnetic. Those, which are repelled by a
magnetic field, are called diamagnetic. Paramagnetic species have unpaired electrons in their
electronic configuration. Diamagnetic substances are those in which electrons are fully paired. In a
simple situation, where one may consider aquo complex ions, we have the following formulation.

Metal Electronic No. of unpaired e’s Metal Electronic No. of unpaired e’s
ion configuration ion configuration
Sc3+ 3d0 No unpaired electrons Ti3+ 3d1 1 unpaired electron
V3+ 3d2 2 unpaired electrons Cr3+ 3d3 3 unpaired electrons
Mn3+ 3d4 4 unpaired electrons Fe3+ 3d5 5 unpaired electrons
Mn2+ 3d5 5 unpaired electrons Fe2+ 3d6 4 unpaired electrons
Co2+ 3d7 3 unpaired electrons Ni2+ 3d8 2 unpaired electrons
Cu2+ 3d9 1 unpaired electron Cu+ 3d10 No unpaired electrons
Zn2+ 3d10 No unpaired electrons

Unpaired electrons in any species have, each, a spin angular momentum, which can be vectorially added to
yield a resultant spin angular momentum. This gives rise to a magnetic moment. Actually, there are two contributions
to the magnetic moment i.e., the magnetic moment due to orbital angular momentum and the spin magnetic moment.
In many situations, the environment in which a species is located has the effect of quenching out the orbital
contribution.
Thus, in such cases, only the spin magnetic moment is measured; in units of Bohr magneton. The spin

magnetic moment is given by


μs =√ n(n+2) in BM, where n is the number of unpaired electrons.
eh
Note : Bohr magneton has the value; BM =
4 π mo c where e = magnitude of electronic charge,

mo = rest mass of the electron and c = speed of light in vaccum. Typical values are Ti 3+, 3d1, 1×3 BM = 1.73 BM

and this agrees with the measured value. In many cases, the observed and calculated values in the spin magnetic
moment are in fair agreement. In fact, determination of spin magnetic moment helps us to know the number of
unpaired electrons in the complex/complex ion, which leads us to the bonding and structure elucidation of the
complex/complex ion.

12 CATALYTIC PROPERTIES
Transition metals and their compounds act as good catalysts for a variety of reactions.
The presence of empty dorbitals enables them to form various intermediates during a reaction, thus providing a
reaction path with lower activation energy for the reaction.
India’s first Colour STUDY MATERIALS for JEE, NEET ,OUAT & OTHERS ENTRANCE EXAMINATION Page 8
Many transition metals and their compounds have catalytic properties. Few of them are listed in the table
below:

TiCl3 Used as the ZieglerNatta catalyst in the production of polythene.


V2O5 Converts SO2 to SO3 in the contact process for making H2SO4.
MnO2 Used as a catalyst to decompose KClO3 to give O2.
Fe Promoted iron is used in the HaberBosch process for making NH3.
FeCl3 Used in the production of CCl4 from CS2 and Cl2.
FeSO4 and
Used as Fenton’s reagent for oxidizing alcohols to aldehydes.
H2O2
Pd Used for hydrogenation (e.g. phenol to cyclohexanone)
Pt/PtO Adams catalyst, used for reductions.
Pt Formerly used for SO2  SO3 in the contact process for making H2SO4.
Pt/Rh Formerly used in the Ostwald process for making HNO 3 to oxidize NH3 to NO.
Cu Is used in the manufacture of (CH3)2SiCl2 used to make silicones.
CuCl2 Deacon process of making Cl2 from HCl.
Ni Raney nickel is used in numerous reduction processes (e.g. manufacture of
hexamethylenediamine, production of H2 from NH3, reducing anthraquinone to
anthraquinol in the production of H2O2).

13 NON – STOICHIOMETRIC COMPOUNDS

A unique feature of the transition elements is that they sometimes form nonstoichiometric compounds. These
are compounds of indefinite structure and proportions. For example, iron(II) oxide (FeO) should be written with a bar
over the formula, FeO to indicate that the ratio of Fe and O atoms is not exactly 1 : 1. Analysis shows that the
formula varies between Fe0.94O and Fe0.84O. Nonstoichiometry of FeO is caused by defects in the solid structure.
Vanadium and selenium form a series of compounds ranging from VSe 0.98 to VSe2. These are given the
formulae:
VSe (VSe0.98  VSe1.2)
V 2 Se 3 (VSe1.2  VSe1.6)
V 2 Se 4 (VSe1.6  VSe2)
Nonstoichiometry is shown particularly among transition metal compounds of the group
16 elements (O, S, Se, Te). It is mostly due to the variable valency of transition elements. For example, copper is
precipitated from a solution containing Cu 2+ by passing in H2S.
The sulphide is completely insoluble, but this is not used as a gravimetric method for analyzing Cu because the
precipitate is a mixture of CuS and Cu2S.

14 INNER TRANSITION ELEMENTS


fblock elements are also referred as “inner transition elements”. These are two series of elements, formed by
the filling of 4f and 5fsubshells. The elements in which 4fsubshell is filled are called lanthanides and the elements
in which 5fsubshell is filled are called actinides.

15 ELECTRONIC CONFIGURATION
(i) Lanthanides :
Ce (Z=58) to Lu (Z=71) – (6th period)

Atomic
58 59 60 61 62 63 64 65 66 67 68 69 70 71
No. (Z)
Symbol Ce Pr Nd Pm Sm Eu Gd Tb Dy Ho Er Tm Yb Lu

India’s first Colour STUDY MATERIALS for JEE, NEET ,OUAT & OTHERS ENTRANCE EXAMINATION Page 9
Electronic Configuration: [Xe] 4f114 5d01 6s2
(ii) Actinides :
Th (Z = 90) to Lr (Z = 103) – (7th period)

Atomic
90 91 92 93 94 95 96 97 98 99 100 101 102 103
No. (Z)
Symbol Th Pa U Np Pu Am Cm Bk Cf Es Fm Md No Lr

Electronic Configuration: [Rn] 5f114 6d01 7s2

16 VARIABLE OXIDATION STATES OF LANTHANIDES AND ACTINIDES


Lanthanides exhibit (III) oxidation state (some elements show (II) and (IV) also). Many of the compounds are
coloured. In the lanthanide elements, there is regular decrease in the radius as the period is traversed. This is known as
“Lanthanide Contraction”. In case of actinides, it is called “Actinide Contraction”. In these elements, the electrons
are added to the antipenultimate shell. The addition of each electron to the 4forbitals results in a concomitant
increase in atomic number. Since, the addition of electrons is to the antipenultimate shell, there is no significant
change in the ultimate and penultimate shells. As a result of the increasing nuclear charge, there is a regular decrease
in the radius along the period.
Electronic structures and oxidation states for lanthanide series
Element Symbol Outer electronic Oxidation states
configuration
Lanthanum La [Xe]5d16s2 III
Cerium Ce [Xe]4f15d16s2 III IV
Praseodymium Pr [Xe]4f36s2 III (IV)
Neodymium Nd [Xe]4f46s2 (II) III
Promethium Pm [Xe]4f56s2 (II) III
Samarium Sm [Xe]4f66s2 (II) III
Europium Eu [Xe]4f76s2 II III
Gadolinium Gd [Xe]4f75d16s2 III
Terbium Tb [Xe]4f96s2 III (IV)
Dysprosium Dy [Xe]4f106s2 III (IV)
Holmium Ho [Xe]4f116s2 III
Erbium Er [Xe]4f126s2 III
Thulium Tm [Xe]4f136s2 (II) III
Ytterbium Yb [Xe]4f146s2 II III
Lutetium Lu [Xe]4f145d16s2 III

Electronic structures and oxidation states for actinide series


Outer electronic
Element Symbol Oxidation states
configuration
Actinium Ac 6d17s2 III
Thorium Th 6d27s2 III IV
Protactinium Pa 5f26d17s2 III IV V
Uranium U 5f36d17s2 III IV V VI
Neptunium Np 5f46d17s2 III IV V VI VII
Plutonium Pu 5f67s2 III IV V VI VII
Americium Am 5f77s2 II III IV V VI
Curium Cm 5f76d17s2 III IV
Berkelium Bk 5f97s2 III IV
Californium Cf 5f107s2 II III
Einsteinium Es 5f117s2 II III
Fermium Fm 5f127s2 II III
Mendeleviu Md 5f137s2 II III
m

India’s first Colour STUDY MATERIALS for JEE, NEET ,OUAT & OTHERS ENTRANCE EXAMINATION Page 10
Nobelium No 5f147s2 II III
Lawrencium Lr 5f146d17s2

The most important oxidation states (generally the most abundant and stable) are shown
in bold. Other wellcharacterized but less important states are shown in normal type. Oxidation states that are unstable
or in doubt are given in parentheses.

17 CO-ORDINATION COMPOUNDS SOME BASIC TERMS

SIMPLE SALTS
These are produced as a result of neutralisation of an acid by a base. For example,
NaOH + HCl  NaCl + H2O
When dissolved in water, they produce ions in the solution. Depending on the extent of neutralisation of the
acid or base, simple salts are further classified as normal, acid or basic salts.

MIXED SALTS
These salts contain more than one acidic or basic radicals. For example, NaKSO 4

MOLECULAR OR ADDITION COMPOUNDS


When solutions containing two or more simple stable salts in stoichiometric proportions are allowed to
evaporate, addition compounds are formed. For example,
KCl + MgCl2 + 6H2O  KCl.MgCl2.6H2O
(Carnalite)
K2SO4 + Al2(SO4)3 + 24H2O  K2SO4.Al2(SO4)3.24H2O
(Alum)
CuSO4 + 4NH3 + H2O  [Cu(NH3)4]SO4.H2O
(Complex)
4KCN + Fe(CN)2  K4[Fe(CN)6]
(Potassium ferrocyanide)
Addition compounds are of two types:
(a) Double salts (Lattice compounds):
Addition compounds, which exist as such in crystalline state only and lose their identity in solution are called
double salts. For example,
+ 2−
FeSO4.(NH4)2SO4.6H2O  Fe2+(aq) + 2 NH 4 (aq) + 2 SO4 (aq) + 6H2O
(b) Coordination compounds:
The addition compounds that results from the combination of two or more simple stable salts and retain their
identity in the solid as well as in dissolved state are called complex compounds. e.g.
K4[Fe(CN)6]  4K+ + [Fe(CN)6]4
A complex compound contains a simple cation and a complex anion or a complex cation and a simple anion or
a complex cation and a complex anion or a neutral molecule. Examples are K 4[Fe(CN)6], [Cu(NH3)4]SO4,
[Co(NH3)6] [Cr(CN)6] and Ni(CO)4 respectively. Thus, a complex ion is defined as “an electrically charged
radical, which consists of a central metal atom or ion surrounded by a group of ionic or neutral species.

LIGANDS
The neutral molecules or ions (usually anions) which are linked directly with the central metal atom/ion are
called ligands. In most of the complexes, ligands act as donor of one or more lone pairs to the central metal
atom/ion. It should be noted that in metallic carbonyls, the ligand, CO, acts as both donor and acceptor
(M CO).

COORDINATION NUMBER (OR LIGANCY)

India’s first Colour STUDY MATERIALS for JEE, NEET ,OUAT & OTHERS ENTRANCE EXAMINATION Page 11
The total number of atoms of ligands that can coordinate to the central metal atom/ion is called coordination
number. For example, in [Fe(CN)6]4, the coordination number of Fe2+ ion is 6.

COORDINATION SPHERE
The central metal ion and the ligands that are directly attached to it, are enclosed in a square bracket, called
coordination sphere or first sphere of attraction.

EFFECTIVE ATOMIC NUMBER (EAN)


Sidgwick extended the Lewis theory to account for the bonding in the coordination compounds. He
introduced the term coordinate bond for a shared electron pair if it initially belonged to one atom (donor
atom) only. In this case, the donor atom acts as a Lewis base and the metal ion acts as a Lewis acid. The metal
ion accepts the electron pairs till it achieves the next inert gas configuration. This is called the effective
number rule.
The total number of electrons, which the central metal atom appears to possess in the complex, including those
gained by it in bonding, is called effective atomic number of central metal ion. When the EAN was 36 (Kr), 54
(Xe) or 86 (Rn), the EAN rule was said to be followed.
For example, in [Co(NH3)6]3+ cobalt has an atomic number 27. In Co 3+ number of
electrons is 24. Each ammonia molecule donates a pair of electrons. So, EAN becomes
24 + (2  6) = 36.
In many cases it was found EAN in a complex should be equal to number of electrons present in next noble
gas.
There are exceptions as well. For example,
EAN of [Ni(NH3)6]+2 is 38 and [Cr(NH3)6]3+ is 33.

The EAN of metals in some metal complexes


Electrons
Atomic number Electrons on
Metal complex donated by the EAN
of metal metal ion
ligands
[Co(NO2)6]3 Co (27) 24 6  2 = 12 24 + 12 = 36
[Cd(NH3)4]2+ Cd (48) 46 42=8 46 + 8 = 54
[PtCl6] 2
Pt (78) 74 6  2 = 12 74 + 12 = 86
[Cr(CO)6] Cr (24) 24 6  2 = 12 24 + 12 = 36
[Ni(CO)4] Ni (28) 28 42=8 28 + 8 = 36
[Ag(NH3)2]Cl Ag (47) 46 22=4 46 + 4 = 50
K4[Fe(CN)6] Fe (26) 24 6  2 = 12 24 + 12 = 36
[Cu(NH3)4]SO4 Cu (29) 27 42=8 27 + 8 = 35
K2[Ni(CN)4] Ni (28) 26 42=8 26 + 8 = 34
K2[PtCl6] Pt (78) 74 6  2 = 12 74 + 12 = 86
K3[Cr(C2O4)3] Cr (24) 21 6  2 = 12 21 + 12 = 33
K2[HgI4] Hg (80) 78 42=8 78 + 8 = 86
As a theory, EAN rule is of no importance as it merely emphasizes the importance of the inert gas shell
stability in compounds. Even though metal carbonyls and related compounds seem to obey this rule, many
exceptions exist that invalidate the usefulness of the rule.

18 CLASSIFICATION OF LIGANDS
There are two ways ligands can be classified:
(I) Classification based on donor and acceptor properties of the ligands
(i) Ligands having one or more lone pair(s) of electrons are further classified as
(a) Ligands containing vacant type orbitals can receive back donated electrons from the metal ion in low
oxidation state. Examples of such ligands are CO, NO, CN  and unsaturated organic molecules. Such

India’s first Colour STUDY MATERIALS for JEE, NEET ,OUAT & OTHERS ENTRANCE EXAMINATION Page 12
ligands have filled donor orbitals in addition to vacant  acceptor orbitals. Thus, in the complexes

L). (M
formed by such ligands, both metal and the ligand act as donors and acceptors 

(b) Ligands, which have no vacant orbitals to get back donated electrons from the metal. e.g. H 2O, NH3, F
etc.
..
(ii) Ligands having no lone pair of electrons but bonding electrons. e.g. C2H4, C6H6 C5H9 etc.

(II) Classification based on the number of donor atoms present in the ligands:
Such ligands are of following types:
(i) Monodentate or unidentate ligands
The ligands that can coordinate to the central metal ion at one site only are called monodentate ligands. Such
ligands may be neutral molecules, negatively or positively charged ions. For example,

+
  
F , Cl , CN , OCN , H2O, NH3,  NH 2 NH 3 etc.
N O 2 , NO+,
A monodentate ligand having more than one lone pair of electrons may simultaneously coordinate with two
or more atoms and thus acts as a bridge between the metal ions.
In such a case, it is called a bridging ligand and the complex thus formed is known as bridged complex. For
example,
− 2−
OH, F, NH 2 , CO, O2, SO 4 etc.
(ii) Bidentate ligand
Ligands, which have two donor atoms and have the ability to coordinate with the central atom/ion at two
different sites are called bidentate ligands. For example,
.. ..
H2N(CH2)2NH2 (ethylenediamine)
(iii) Tridentate ligands
The ligands having three coordination sites are called tridentate ligands. For example,
.. .. ..
H2N – CH2 – CH2 – NH – CH2 – CH2 – NH2
e.g.,
Diethylenetriamine
(iv) Polydentate ligands
The ligands having four or more co-ordination sites are called polydentate ligands.
For example,
H H
N CH2 CH2 N
N (CH2)2 N
CH2 CH2
– –
(CH2)2 (CH2)2 H2C O=C–O OC=O CH2

N C O –O C
N
O O
H H H
H
Triethylenetetramine Ethylenediamine
(trine) tetraacetate ion (EDTA)
(v) Ambidentate ligands
They have two or more donor atoms but, while forming complexes only one donor atom is attached to the

metal ion. The examples of such ligands are CN –, NO 2 , NCS, NCO etc.
(vi) Chelating ligands
When a bidentate or a polydentate ligand is attached through two or more donor atoms to the same metal ion
forming a ring structure, the ligand is called chelating ligand.
The chelating ligands form more stable complexes than ordinary unidentate ligands.

19 IUPAC NOMENCLATURE OF COMPLEXES


The following rules are used for naming all types of complexes.
India’s first Colour STUDY MATERIALS for JEE, NEET ,OUAT & OTHERS ENTRANCE EXAMINATION Page 13
(1) In case of ionic complexes, cation is named first followed by the anion, irrespective of
the fact, whether cation or anion or both are complex. Simple cation and anion are named just like naming a
simple salt.
(2) Number of cations and anions are not mentioned while writing its name.
(3) There has to be a gap between the cation’s name and anion’s name. The gap should not exist anywhere else
and the name of cation and anion should be written in one continuous text.
(4) Within a complex ion, the ligands are named first in the alphabetical order followed by name of the metal ion,
which is followed by the oxidation state of metal ion in Roman numeral in parentheses except for zero.
(5) Name of all negative ligands ends with ‘o’ while the name of all positively charged ligands ends with ’ium’.
Neutral ligands have no special ending.

Name of Negative Ligands


Ligand Name Ligand Name
H hydrido HS mercapto
O2 oxo −
NH 2 amido
O2−
2
peroxo NH2 imido
O2H perhydroxo NO−3 nitrato
 
OH hydroxo ONO nitrito
F fluoro NO−2 nitro
Cl chloro N3 nitrido
Br bromo P3 phosphido
I iodo N−3 azido
2−
CO 3 carbonato CNO cyanato
C2 O2−
4
oxalato NCO isocyanato

CH 3 CO−2 acetato SCN thiocyanato or thiocyanatoS
 
CN cyano NCS isothiocyanato or thiocyanatoN
SO 2− sulphato hydrogencarbonato

4 HCO3
SO 2−
3
sulphito S 4 O2−
6
tetrathionato
S2 sulphido EDTA

( O2CCH2)2NCH2CH2N ethylenediaminetetraacetato
(CH2CO2)2
HSO−3 hydrogensulphito NH 2 CH 2 CO−2 glycinato
S 2 O2−
3
thiosulphato C5 H −5 cyclopentadienyl

Ligands whose names end in –“ite” or –“ate” become –“ito” or –“ato”, i.e., by replacing the ending –e with –
o.

Name of Neutral Ligands


Ligand Name Abbreviation Ligand Name Abbreviation
H2O aqua/aquo  NH2(CH2)2NH ethylenediamine (en)
2
NH3 ammine  CH3NH2 methylamine 
CO carbonyl  C6H6 benzene 
NO nitrosyl  N2 dinitrogen 
CS thiocarbonyl  O2 dioxygen 
NS thionitrosyl  Ph3P triphenylphosphine 
C5H5N pyridine (py) CH3COCH3 acetone 

Name of Positive Ligands


Ligand Name

India’s first Colour STUDY MATERIALS for JEE, NEET ,OUAT & OTHERS ENTRANCE EXAMINATION Page 14
NO+ nitrosonium
NO+2 nitronium

NH 2 NH +3 hydrazinium

(6) If the number of a particular ligand is more than one in the complex ion, the number is indicated by using
Greek numbers such as di, tri, tetra, penta, hexa, etc for number of ligands being 2, 3, 4, 5 and 6 respectively.
However, when the name of the ligand includes a number, for example, dipyridyl, ethylenediamine, then bis,
tris, tetrakis etc. are used in place of di, tri, tetra etc. The ligands for which such prefixes are used, their names
are placed in parenthesis.

(7) For deciding the alphabetical order of ligands, the first letter of the ligand’s name is to be considered and
prefixes di, tri, tetra, bis, tris, tetrakis etc. are not considered.

(8) Neutral and positive ion complexes have no special ending but complex negative ion ends with the suffix
‘ate’ attached to English names of the metal but in some cases ‘ate’ is attached to the Latin names of the metal.

Element Metal as named in anionic complex


Cobalt Cobaltate
Nickel Nickelate
Chromium Chromate
Iron Ferrate
Copper Cuprate
Silver Argentate
Lead Plumbate
Co–ordination sphere is named in continuum.

(9) For those complexes containing solvent of crystallization, it is indicated as: first write
the cation’s name, followed by anion’s name (obviously after a gap) followed by a gap
and then write the number of solvent molecules in Arabic numeral followed by a hyphen which is followed by
solvent’s name.
Coordination compounds containing complex cationic ion
[Pt(NH3)6]Cl4 Hexaammineplatinum(IV) chloride
[Co(NH3)4(H2O)Cl]Cl Tetraammineaquochlorocobalt(II) chloride
[Cu(en)2]SO4 Bis(ethylenediamine)copper(II) sulphate
+
[Cr(H2O)4Cl2] Tetraaquodichlorochromium(III) ion
[Fe(H2O)4(C2O4)]2SO4 Tetraaquooxalatoiron(III) sulphate
[Cr(NH3)4(ONO)Cl]NO3 Tetraamminechloronitritochromium(III) nitrate
[Ag(NH3)2]Cl Diamminesilver(I) chloride
[Co(NH3)5(NCS)]Cl2 Pentaammineisothiocyanatocobalt(III) chloride
[{(C6H5)3P}3Rh]Cl Tris(triphenylphosphine)rhodium(I) chloride

Coordination compounds containing complex anionic ion


K4[Fe(CN)6] Potassium hexacyanoferrate(II)
K3[Fe(CN)6] Potassium hexacyanoferrate(III)
K3[Cr(C2O4)3] Potassium trioxalatochromate(III)
K3[Co(C2O4)2Cl2] Potassium dichlorodioxalatocobaltate(III)
K2HgI4 Potassium tetraiodomercurate(II)
K2[PtCl6] Potassium hexachloroplatinate(IV)
Na[Ag(CN)2] Sodium dicyanoargentate(I)
[Ni(CN)4]2– Tetracyanonickelate(II) ion
Na3[Co(NO2)6] Sodium hexanitrocobaltate(III)
K3[Fe(CN)5NO] Potassium pentacyanonitrosylferrate(II)

India’s first Colour STUDY MATERIALS for JEE, NEET ,OUAT & OTHERS ENTRANCE EXAMINATION Page 15
Coordination compounds containing complex cationic and anionic ions:
[Cr(NH3)6] [Co(CN)6] Hexaamminechromium(III) hexacyanocobaltate(III)
[Pt(NH3)4] [CuCl4] Tetraammineplatinum(II) tetrachlorocuprate(II)
[Cr(NH3)6][Co(C2O4)3] Hexaamminechromium(III) trioxalatocobaltate(III)
[Pt(py)4] [PtCl4] Tetrapyridineplatinum(II) tetrachloroplatinate(II)

Nonionic coordination compounds


Fe(CO)5 Pentacarbonyliron(0)
[Co(NO2)3(NH3)3] Triamminetrinitrocobalt(III)
Cu(Gly)2 Diglycinatocopper(II)
Ni(DMG)2 Bis(dimethylglyoximato) nickel(II)

(10) Naming of the bridging ligands of the bridged polynuclear complexes:


Complexes having two or more metal atoms are called polynuclear complexes. In these complexes, the
bridging group is indicated by separating it from the rest of the complex by hyphen and adding the prefix
before the name of each different bridging group. Two or more bridging groups of the same type are
indicated by di, trietc. When a
bridging ligand is attached to more than two metal atoms or ions, this is indicated by a subscript to .

NH2
(en)2Co Co(en)2 (SO4)2 is named as
OH
Bis(ethylenediamine)cobalt(III)amidohydroxobis(ethylenediamine)cobalt(III)sulphate
or amidotetrakis (ethylenediamine)hydroxodicobalt (III) sulphate
OH

The complex (H2O)4Fe Fe(H2O)4 (SO4)2


OH
is named as: Tetraaquoiron(III)dihydroxotetraaquoiron(III) sulphate
The stable oxidation states of some of the transition metals of the three series are given below. These would be
helpful to find the oxidation states of the metal ions while naming complexes having cation and anion both as
complex species.

(i) First transition series

Sc Ti V Cr Mn Fe Co Ni Cu Zn
+3 +2, +3, +2, +3, +2, +3, +2, +3, +2, +3 +2, +3 +2, +3 +1, +2 +2
+4 +4, +5 +6 +4, +7

(ii) Second transition series

Y Zr Nb Mo Tc Ru Rh Pd Ag Cd
+3 +4 +3, +5 +6 +4, +6, +3 +3 +2, +4 +1 +2
+7

(iii) Third transition series

La Hf Ta W Re Os Ir Pt Au Hg

India’s first Colour STUDY MATERIALS for JEE, NEET ,OUAT & OTHERS ENTRANCE EXAMINATION Page 16
+3 +4 +5 +6 +4, +6, +3, +4, +1, +3, +2, +4 +1, +3 +1, +2
+7 +6 +4

20 WERNER’S CO-ORDINATION THEORY


Several theories were proposed to explain the observed properties of Co(III) ammines and of other similar
compounds like Pt(IV) ammines which had been prepared by then. It was only in 1893, that Werner presented a theory
known as Werner’s coordination theory which could explain all the observed properties of complex compounds.
Important postulates of this theory are
(i) Most elements exhibit two types of valencies: (a) primary valency and (b) secondary valency.
(a) Primary valency
This corresponds to the oxidation state of the metal ion. This is also called principal, ionisable or ionic
Valency. It is satisfied by negative ions and its attachment with the central metal ion is shown by dotted lines.

(b) Secondary or auxiliary valency


(i) It is also termed as coordination number (usually abbreviated as CN) of the central metal ion. It is non-ionic or
non-ionisable (i.e coordinate covalent bond type). This is satisfied by either negative ions or neutral
molecules. The ligands, which satisfy the coordination number are directly attached to the metal atom or ion
and are shown by thick lines. While writing down the formulae, these are placed in the coordination sphere
along with the metal ion. These are directed towards fixed positions in space about the central metal ion, e.g.
six ligands are arranged at the six corners of a regular octahedron with the metal ion at its centre. This
postulate predicted the existence of different types of isomerism in coordination complexes and after 19 years,
Werner actually succeeded in resolving various coordination examples into optically active isomers.
(ii) Every element tends to satisfy both its primary and secondary valencies. In order to meet this requirement a
negative ion may often show a dual behaviour, i.e., it may satisfy both primary and secondary valencies (since
in every case the fulfilment of coordination number of the central metal ion appears essential).
In all the ammine cobalt complexes, cobalt shows secondary valency (i.e., coordination number) of six and
primary valency (i.e., oxidation state) of three.
Designation of formation of Co(III) Ammines
On the basis of postulates of his theory, Werner designated the ammines as given in figure and formulated
them as described below
The molecule, CoCl3.6NH3 which is formulated as
[CoIII(NH3)6]3+(Cl–)3 has six NH3 molecules that
satisfy the secondary valency of the metal ion, viz.,
Co3+ ion and their attachment with the central metal NH3 Cl
NH3
ion is shown by thick lines. The primary valency (i.e., NH3
oxidation state of +3) is satisfied by three Cl – ions, Cl Co
which have been shown by dotted lines and are kept
outside the coordination sphere. As all the three Cl – NH3 NH3
ions are loosely bound, they are immediately NH3 Cl
precipitated as AgCl on the addition of AgNO 3 CoCl3.6NH3 or
solution. Thus the solution of this compound should [CoIII(NH3)6]3+(Cl–)3
conduct current to give four ions in all viz.
[Co(NH3)6]3+ and 3Cl–, which has been confirmed by
conductivity measurements.
In the molecule, CoCl3.5 NH3.H2O which is NH3 Cl
III NH3
formulated as [Co (NH3)5 (H2O)]Cl3, five NH3
NH3
molecules and one H2O molecule satisfy the secondary
Cl Co
valency (shown by thick lines in the designation).
Primary valency is satisfied by three Cl – ions. The H2O NH3
solution of this compound also conducts current and NH3 Cl
gives in all four ions: one complex ion, CoCl3.5NH3.H2O or
[CoIII(NH3)5(H2O)]3+ and three simple ions, 3Cl–. [CoIII(NH3)5 (H2O)]3+(Cl–)3

India’s first Colour STUDY MATERIALS for JEE, NEET ,OUAT & OTHERS ENTRANCE EXAMINATION Page 17
In the molecule CoCl35NH3 which is formulated as [CoIII(NH3)5Cl]2+(Cl–)2 on the basis of Werner’s theory
one Cl– ion does the dual function, since it satisfies both primary and secondary valency. Werner, therefore,
showed its attachment with the central metal ion by a combined dashed-solid line,––- . This Cl – ion, being
non-ionic, is not precipitated as AgCl by Ag + ions and hence it is different from the other two Cl – ions and has
been placed along with five NH3 molecules and central metal ion in the coordination sphere as shown in its
formulation. The other two Cl– ions, being ionic, are precipitated as AgCl by Ag + ions and the total number of
ions obtained is three: One complex ion, [Co III(NH3)5Cl]2+ and
 III 2+
two simple ions, 2Cl . Thus, [Co (NH3)5Cl] Cl2 satisfies both primary (+3) and secondary (Coordination
number = 6) of Co3+.
NH3
NH3 NH3

Cl Co Cl

H3N NH3
Cl
CoCl35NH3 or
[CoIII(NH3)5Cl]2+(Cl–)2

The formulation [CoIII(NH3)4Cl2]+Cl– of CoCl3. 4NH3 shows that it has only one ionic Cl– ion, which gets
precipitated as AgCl by AgNO3 solution. The conductivity measurements show that it has two ions in solution:
[CoIII(NH3)4Cl2]+ and Cl.
Cl
NH3 NH3

Cl Co

H3N Cl
NH3
CoCl34NH3 or
[CoIII(NH3)4Cl2]+ Cl–

The formulation [CoIII(NH3)3Cl3]0 of CoCl3.3NH3 has no ionic Cl ions and hence it behaves as a
nonelectrolyte.
Cl
NH3 NH3

Co

Cl Cl
NH3
CoCl3.3NH3 or [CoIII(NH3)3 Cl3]0

21 VALENCY BOND THEORY


The valence bond theory deals with the electronic structure of the central metal ion in its ground state, kind of
bonding, geometry and magnetic properties of the complexes. This theory takes into account the hybridisation
of vacant orbitals of central metal ion and was proposed by Linus Pauling, using hybridised orbitals. The main
points of the valence bond theory are as follows.
1. The central metal loses requisite number of electrons and forms the cation. The number of electrons lost
corresponds to the valency of the resulting cation.
2. The central metal ion makes available a number of empty s, p and d atomic orbitals equal to its coordination
number.
3. These vacant orbitals hybridise together to form hybrid orbitals which are the same in number as the atomic
orbitals hybridising together. These hybrid orbitals are vacant, equivalent in energy and have a definite
geometry.

India’s first Colour STUDY MATERIALS for JEE, NEET ,OUAT & OTHERS ENTRANCE EXAMINATION Page 18
4. The nonbonding metal electrons occupies the inner orbitals and they do not take part in the hybridisation.
The electrons are grouped in accordance with the Hund’s rule of maximum multiplicity. However, under the
influence of a strong ligand, they may be forced to pair up against the Hund’s rule.
5. The dorbitals involved in the hybridisation may be either inner (n1) dorbitals or outer
ndorbitals.
6. Each ligand (donor group) must contain a lone pair of electrons.
7. Vacant hybrid orbitals of the metal atom or ion overlap with the filled (for example, containing lonepair of
electrons) orbitals of the ligands to form a covalent bond represented as M σ
⃗ L. The bond is also
referred to as coordinate bond.
8. In addition to the bond, a bond may be formed by overlap of a filled metal dorbital with a vacant ligand
orbital (M L). This usually happens in complexes of metal ions of low oxidation states.
9. If the complex contains unpaired electrons, the complex is paramagnetic in nature, whereas, if it does not
contain any unpaired electron, the complex is diamagnetic in nature.

Difference between Inner and Outer Orbital Octahedral Complexes


Inner Orbital octahedral complex Outer orbital octahedral complex
2 3 3 2
1. d sp hybridisation 1. sp d hybridisation
2. The inner orbital complexes are formed with 2. The outer orbital complexes are formed
covalent metal ligand bonds. with ionic bond.
3. Low spin complexes 3. High spin complexes

VBTOCTAHEDRAL COMPLEXES
On the basis of VBT, octahedral complexes are of two types:
1. Innerorbital octahedral complexes, which result from d 2sp3 hybridisation of the central metal atom/ion.
2. Outer orbital octahedral complexes, which result from sp 3d2 hybridisation.

C.N Examples
Hybridisation Geometry
.
sp [Ag(NH3)2]+, [Ag(CN)2], [Cu(NH3)2]+
2 Linear
(4s, 4px)
sp2 [HgI3]
3 Trigonal planar
(6s, 6px, 6py)
sp3 [NiCl4]2, [Cu(CN)4]3, [Ni(CO)4], [Zn(NH3)4]2+
4 Tetrahedral
(4s, 4px, 4py, 4pz)
dsp2 [Ni(CN)4]2
4 3d Square planar
( x 2− y 2 , 4s, 4px, 4py,
4pz)
dsp3 Trigonal [Fe(CO)5]3, [CuCl5]3
5 3 dz2
( , 4s, 4px, 4py, 4pz) bipyramidal
dsp3 [Ni(CN)5]3
3d Square
5 ( x 2− y 2 , 4s, 4px, 4py, pyramidal
4pz)
3d 3d Inner orbital [Ti(H2O)6]2+, [CrF6]3 uses (n1) dorbitals.
d2sp3( x 2− y 2 , z2 ,
6
4s, 4px, 4py, 4pz) octahedral
sp3d2(4s, 4px, 4py, 4pz, Outerorbital [Co(H2O)6]3+, [Zn(NH3)6]2+, [CoF6]3 uses
6 4d 4d ndorbitals.
x 2− y2
, z2 ) octahedral
(a) Inner orbital octahedral complexes

India’s first Colour STUDY MATERIALS for JEE, NEET ,OUAT & OTHERS ENTRANCE EXAMINATION Page 19
The formation of these complexes can be explained on the basis of VBT by considering the complex ion, viz
[Co(NH3)6]3+.
[Co(NH3)6]+3
3d 4s 4p
Co atom
(3d74s24p°)
(a)
Free Co3+ ion
(3d64s°4p°)
(b) in ground state
Co3+ ion in [Co(NH3)6]3+
(c)
[Co(NH3)6]3+ XX XX XX XX XX XX

n=0 2 3
(d) d sp

Here n represents the number of unpaired electrons and ‘XX’ represents an electron pair donated by each of
free six NH3 ligands. The two electrons of the electron pair have opposite spin. The above complex ion is
diamagnetic as all the electrons are paired.
3d 3d 2 2
In order to make 3d electrons paired, the two unpaired electrons residing in z2 and x − y orbitals
are forced by the six NH 3 ligands to occupy 3dyz and 3dzx orbitals. By doing so, all the 3d electrons become
3d 3d 2 2
paired and also at the same time, two 3d orbitals namely z2 , x − y , hybridise together with. 4s, 4px,
2 3
4py and 4pz orbitals to give six d sp hybrid orbitals which, being empty accepts the six electron pairs donated
by six NH3 ligand molecules.

[Cr(NH3)6]3+
3d 4s 4p
Cr atom
(3d54s1)
(a)
Free Cr3+ ion
(3d34s°4p°)
(b) in ground state
Cr3+ ion in [Cr(NH3)6]3+
(c)
[Cr(NH3)6]3+ XX XX XX XX XX XX
n=3
2 3
(d) d sp

The above complex ion is paramagnetic as there are three unpaired electrons.

= √ n(n+2)=√ 3(3+2)=√ 15 B.M.


[Cr(CN)6]4
3d 4s 4p
Cr atom
(3d54s1)
(a)
Cr2+ ion
(3d4)
(b) in ground state
Cr2+ ion in
[Cr(CN)6]4
(c)
[Cr(CN)6]4 XX XX XX XX XX XX
n=2
2 3
(d) d sp
The above complex ion is paramagnetic since two unpaired electrons are present.

India’s first Colour STUDY MATERIALS for JEE, NEET ,OUAT & OTHERS ENTRANCE EXAMINATION Page 20
= √2(2+2)= 8 B.M.

Other examples of inner orbital octahedral paramagnetic complexes are [Ti(H 2O)6]3+ (n=1), [Mn(CN) 6]4 (n =
1), [Mn(CN)6]3 (n = 2), [Fe(CN) 6]3 (n =1), [Co(CN)6]4 (n = 1 in 5s orbital) while the examples of inner
orbital octahedral diamagnetic complexes are: [Fe(CN) 6]4, [Co(CN)6]3, [Co(H2O)6]3+, [Co(NH3)6]3+,
[Co(NO2)6]3, [Pt(NH3)6]4+ etc.
All these complexes result from d2sp3 hybridisation of the central metal ion.

(b) Outer orbital octahedral complexes


Octahedral complexes resulted from sp 3d2 hybridisation, using outer d and outer s and p orbitals are called
outerorbital octahedral complexes.
[CoF6]3
3d 4s 4p 4d
Co atom
(3d74s2)
(a)

Co3+ ion
(b) in ground state
Co3+ ion in
[CoF6]3
ion in excited
(c) state
X X X X X X
[CoF6]3 ion X X X X X X
n=4
3 2
(d) sp d

4d 4d
In this complex ion, it is x 2− y2 and z2 orbitals that mix with one 4s and three 4p orbitals to give six
sp d hybrid orbitals, which being empty, accept the six electron pairs denoted by each of the six F  ligands. It
3 2

is paramagnetic as there are four unpaired electrons.

= n(n+2)= 4 (4 +2)= 24 B.M.


√ √ √
Some other examples of outer orbital octahedral paramagnetic complex are:
[Cr(H2O)6]2+ (n = 4), [Mn(H2O)6]2+ (n = 5), [Fe(H2O)6]2+ (n = 4), [Fe(NH3)6]2+ (n = 4)
[Fe(H2O)6]3+ (n = 5), [Fe(F)6]3 (n = 5), [CoF6]3 (n = 4),
[Co(NH3)6]2+ (n = 3)
[Co(H2O)6]2+ (n = 3), [Ni(NH3)6]2+ (n = 2),
[Ni(H2O)6]2+ (n = 2), [Ni(NCS)6]4 (n = 2),
[Ni(NO2)6]4 (n = 2), [CuF6]3 (n = 2).
[Zn(NH3)6]2+ (n = 0) is an example of outer orbital octahedral diamagnetic complex.

VBTTETRAHEDRALCOMPLEXES(Coordination no. = 4, sp3 hybridisation)


Tetrahedral complexes result from sp 3 hybridisation. In sp3 hybridisation the s and three porbitals should
belong to the same shell. The formation of tetrahedral complexes by VBT can be explained by considering the
complex ion like [MnCl4]2. This complex ion is paramagnetic corresponding to the presence of five unpaired
electrons and hence the configuration of Mn2+ ion in the free state and in the complex ion remains the same.
3d 4s 4p
Mn
(3d54s24p°)
(a)
Free Mn2+ ion
5
(b) (3d 4s°4p°)
Mn2+ ion in
2
(c) [MnCl4] ion

India’s first Colour STUDY MATERIALS for JEE, NEET ,OUAT & OTHERS ENTRANCE EXAMINATION Page 21
3d 4s 4p
2
[MnCl4] ion XX XX XX XX
n=5 3
(d) sp

 = √ n(n+2)=√ 5(5+2)=√ 35 B.M.


Examples of some paramagnetic tetrahedral complexes are
[NiCl4]2 (n = 2), [Ni(NH3)4]2+ (n = 2)
[MnBr4]2 (n = 5), [FeCl4]2 (n = 4),
[CoCl4]2 (n =3), [CuCl4]2 (n = 1) etc.
While the examples of some diamagnetic tetrahedral complexes are:
[Ni(CO)4] (n = 0), [Cu(CN)4]3 (n = 0), [Zn(NH3)4]2+ (n = 0), [ZnCl4]2 (n = 0), [Cd(CN)4]2 (n = 0), etc. All
these complexes result from sp3 hybridisation of the central metal atom/ion.

VBTSQUARE PLANAR COMPLEXES (Coordination no. = 4, dsp2 hybridisation)


d 2 2
Square planar complexes result from dsp 2 hybridisation. In dsp2 hybridisation, dorbital should be x −y
orbital (belonging to the lower shell) while s and p orbitals should be from the higher shell. The two porbitals
d 2 2
should be px and py orbitals. The selection of x −y ,
px and py orbitals is based on the fact that all these orbitals lie in the same plane. The formation of square
planar complexes by VBT can be explained by considering the complex ion like [Ni(CN) 4]2. The
measurement of magnetic moment value for [Ni(CN) 4]2 ion has shown that  = 0 i.e. the complex ion has no
unpaired electron and hence
1. Although it is diamagnetic.
valence bond theory provides a satisfactory representation of the
3d 4s 4p
complex compound 2
based 2 2
upon the concept of orbital hybridisation, it cannot
z y y z
account for the relativex stabilities for xdifferent shapes and coordination numbers in
Ni atom
(3d84s24p°)metal complexes.
(a) 2.VBT cannot explain as to why Cu(+2) forms only distorted octahedral complexes
even when all the six ligands are identical.
Free Ni2+ ion 3.The valence bond theory does not provide any satisfactory explanation for the
in ground state
existence of inner orbital and outer orbital complexes.
8
(3d , 4s°, 4p°)
4.Sometimes the theory requires the transfer of electron from lower energy to the
(b)
+2
Ni ion in higher energy level, which is very much unrealistic in absence of any energy supplier.
[Ni(CN)4]2(For example, this happens in the case of [CuX 4]2).
ion in excited5.The changes in the properties of the metal ion along with the ligands and the simple
(c) state metal ions can not be explained. For example, the colour changes associated with
electronic transition within d3dorbitals x2 y2 4sare affected
4px 4py on4pz formation of complex, but the
2valence bond theory does not offer any explanation.
[Ni(CN)4] ion XX XX XX XX
6.Sometimes the samen =metal 0 acquires different geometry when formation of complex
2
(d) takes place with different ligands. The theory dspdoes not explain as to why at one time
the electrons must be rearranged against the Hund’s rule while, at other d 2 2times the
In order to electronic
make all the 3delectrons
configuration is paired, one unpaired electron residing in 3 x − y orbital is forced by
not disturbed.
7.The

energy change3of d z 2the metal orbitals on d 2formation
2
of complex is difficult to be
the four CNcalculated
ligand tomathematically.
occupy orbital. Now 3 x − y , 4s, 4px, 4py orbitals mix together to form four
dsp2 hybrid 8.VBT
orbitalsfails
which, being empty,
to explain the finer accept
detailstheoffour electron
magnetic pairs donated
properties by the
including thefour CN  ligand ions.
magnitude
Examples of ofparamagnetic square planar
the orbital contribution complexes
to the magneticare: moments.
2 2+ 2
[Cu(CN)4] 9.The (n =VBT1), [Cu(NH ) ] (n = 1), [CuCl
does not explain why certain complexes
3 4 4 ] derivedarefrommore(NH 4)2[CuCl
labile 4] (n
than the = 1) etc. While the
others.
2 2
examples of10. diamagnetic
It does notsquare planar complexes
give quantitative are: [Ni(CN)
interpretation 4] (n = 0), [PtCl
of thermodynamic = 0), [Pt(NH3)4]2+ (n =
4] (n stabilities
or kinetic
0) etc. of coordination compounds.
11. It does not make exact predictions regarding the tetrahedral and square planar
structure of 4-coordinate complexes.
12. It does not tell about the spectral properties of coordination compounds.

India’s first Colour STUDY MATERIALS for JEE, NEET ,OUAT & OTHERS ENTRANCE EXAMINATION Page 22
Limitations of VBT

The above points may be made clear with the help of the following examples:
(i) d1, d2, d3 systems
Ti(22)  [Ar]3d24s2 Ti3+  [Ar]3d1
V(23)  [Ar]3d34s2 V3+  [Ar]3d2
Cr (24)  [Ar]3d54s1 Cr3+  [Ar]3d3
3d 4s 4p

d2sp3 hybridization
In all the three systems two vacant 3d orbitals (n  1) d orbitals are available for d2sp3 hybridisation. Hence,
these systems may accept six lone pairs from six ligands and thus they form octahedral complexes:
[Ti(H2O)6]3+, [V(H2O)6]3+, [Cr(H2O)6]3+
Since due to complexation, the unpaired electrons in (n  1) d orbitals are not disturbed, the magnetic
moment of free metal ions remains intact in octahedral complexes.
In addition to d2sp3 hybridisation, d1, d2, d3 systems may undergo sp3 or dsp2 hybridization forming tetrahedral
or square planar complexes respectively.
Since in sp3 or dsp2, the delectrons are not disturbed, the magnetic moment of free metal ion remains intact
in tetrahedral or square planar complexes.
(ii) d4, d5 and d6 systems

India’s first Colour STUDY MATERIALS for JEE, NEET ,OUAT & OTHERS ENTRANCE EXAMINATION Page 23
Mn(III), Fe(III), Co(III), Fe(II)
3d 4s 4p 4d

sp3d2
In d4, d5 and d6 systems, in ground state two 3d orbitals are not vacant to participate in d 2sp3 hybridization
forming octahedral complexes. Hence, two dorbitals of outer shell are involved in hybridisation and the
complexes are formed as outer orbital octahedral complexes. The energies of the various orbitals are in the
order: 4s < 3d < 4p < 5s < 4d. Since, the energy gap between 4s and 4d is large, the sp 3d2 hybridisation is not
perfect hybridization and hence outer orbital octahedral complexes are comparatively less stable.
Moreover, 4d orbitals are more extended in space than 3d orbitals and hence sp 3d2 hybrid orbitals are also
more extended in space than d 2sp3 hybrids. So, bond length in outer orbital octahedral complexes is
comparatively longer and so they are less stable.
In sp3d2 hybridisation, 3d electrons are not disturbed and hence magnetic moment of free metal ions remains
intact in outer orbital octahedral complexes.
Other possibilities:
3d 4s 4p

d2sp3
After maximum pairing of 3d electrons, two 3d orbitals may be made vacant for d 2sp3 hybridisation forming
octahedral complexes.
As two dorbitals of inner shell are involved in hybridisation, complexes are said to be inner orbital octahedral
complexes.
4s < 3d < 4p ; the energy of the orbitals involved in hybridisation is in continuation.
The d2sp3 hybridisation is perfect and at the same time due to less extension of 3d orbitals in space, bond
length is also short. So, inner orbital octahedral complexes are more stable than outer orbital octahedral
complexes.
As the pairing of 3d electrons is forced in d 2sp3 hybridisation in these systems
(d4, d5 and d6), hence the magnetic moment of the free metal ion undergoes change on complexation.
In addition to inner orbital octahedral and outer orbital octahedral complexes, d 4, d5 and d6 systems may also
form tetrahedral and square planar complexes by sp 3 and dsp2 hybridisation.
(iii) d7, d8 and d9 systems:
Co(II), Ni(II), Cu(II)
3d 4s 4p 4d

sp3d2
In d7, d8 and d9 systems, two vacant 3d orbitals cannot be made available for d 2sp3 hybridization even after
maximum pairing. So, there is no chance of the formation of inner orbital octahedral complexes by d 2sp3
hybridisation. However, these systems may undergo sp3d2 hybridization forming outer orbital octahedral
complexes with same magnetic properties as in free metal. In d 7, d8 and d9 systems, sp3 hybridizations can
easily occur favouring the formation of tetrahedral complexes with unchanged magnetic character.
3d 4s 4p 5s 4d
Co2+
under the influence
of NH3 as ligand
d2sp3
In d7 system, after maximum pairing of the electrons in three of the dorbitals and promoting one electron to
5s or 4d, two 3d orbitals may be made vacant for d 2sp3 hybridisation and formation of inner orbital octahedral
complex may take place with one unpaired electron.
However, with the promotion of one 3d electron to 5s or 4d, it becomes loosely bonded to the nucleus and
hence, it may easily be removed and so, Co(II) will easily be oxidised into Co(III). Virtually the oxidation of
Co(II) has been found to be easy in the formation of inner orbital octahedral complexes by a d 7 system.

India’s first Colour STUDY MATERIALS for JEE, NEET ,OUAT & OTHERS ENTRANCE EXAMINATION Page 24
In d7 and d8 systems, after maximum pairing of 3d electrons, one 3d orbital may be vacated for dsp 2
hybridization and hence d7 and d8 systems favour the formation of square planar complexes with changed
magnetic nature.
However, in the case of d9, even after maximum pairing of electrons in 3d, one d orbital is not made available
for dsp2 hybridization. So, there is no question of the formation of square planar complexes by d 9 systems.
(iv) d10 system:
Zn(II), Cu(I)
3d 4s 4p 4d

sp3
sp3d2
In d10 system, 3d orbitals are completely filled up. So, it may form tetrahedral complexes by sp 3 hybridization
or outer orbital octahedral complexes by sp3d2 hybridisation.
Magnetic properties of the free metal ion remains unchanged in tetrahedral or outer orbital octahedral
complexes.

If the ligand is very weak like F, H2O, Cl etc. it does not force the pairing of 3d electrons and
Note

hence outer orbital octahedral complexes are formed by sp 3d2 hybridisation. But if the ligand is
2−
strong like CN, (COO)2 , ethylenediamine (en) etc., it forces the paring of
3d electrons and hence inner orbital octahedral complexes are formed by d 2sp3 hybridization.

22 FACTORS AFFECTING THE STABILITY OF COMPLEXES


1. A coordination compound is formed in solution by the stepwise addition of ligands to a metal ion. The overall
stability constant is given by
[ MLn]
n
M + nL MLn; Kf = [ M ] [ L]
1
Kf is called instability constant. Higher the value of Kf, more stable is the complex.
2. Higher is the charge density on the central metal ion, greater is the stability of the complexes. For example,
[Fe(CN)6]3– is more stable than [Fe(CN)6]4–

3. More is the basic character of ligand, more stable is the complex.


For example, the cyano and amino complexes are far more stable than the halo complexes.
4. Chelating ligands form more stable complexes than the monodentate ligands.

23 CRYSTAL FIELD THEORY


In crystal field theory, we assume the ligands to be the point charges and there is interaction between the
electrons of the ligands and the electrons of the central metal atom or ion. The five d orbitals in an isolated gaseous
metal atom or ion are degenerate.
This degeneracy is maintained if an spherically symmetrical negative field surrounds the metal atom/ion. However,
when ligands approach the central metal atom/ion, the field created is not exactly spherically symmetrical and the
degeneracy of the dorbitals is lifted. It results in the splitting of dorbitals and the pattern of splitting depends upon
the nature of the crystal field. This splitting of dorbitals energies and its effects, form the basis of the crystal field
treatment of the coordination compounds.
Ligands that cause large degree of crystal filed splitting are termed as strong field ligands. Ligands that cause
only a small degree of crystal filed splitting are termed as
weak field ligands. The common ligands can be arranged in ascending order of crystal field splitting energy. The order
remains practically constant for different metals and this series is called the spectrochemical series.

India’s first Colour STUDY MATERIALS for JEE, NEET ,OUAT & OTHERS ENTRANCE EXAMINATION Page 25
I < Br < S2 < Cl ~ SCN ~ N3 < NO3 < F < OH < CH3CO2 < ox2 < H2O ||
< NCS < EDTA4 < NH3 ~ Py < en < NO2 < H ~ CH3 < CO ~ CN

The spectrochemical series is an experimentally determined series. It is difficult to explain the order as it incorporates
both the effect of  and bonding. The halides are in the order expected from electrostatic effects. In other cases, we
must consider covalent bonding to explain the order. A pattern of increasing donation is as follows:

Halides donors < O donors < N donors < C donors

The crystal field stabilization produced by the strong CN  is almost double that of halide ions. This is
attributing bonding in which the metal donates electrons from a filled
t2g orbital into a vacant orbital on the ligand. In a similar way, many unsaturated N donors and C donors may also act
as acceptors.

Crystal field effects in octahedral coordination entities

Let us assume that the six ligands are positioned symmetrically along the Cartesian axis with the metal atom
or ion at the origin. As the ligands approach the central metal atom or ion, the energy of the d orbitals of the
central metal atom or ion increases. If the field created by the ligands is spherical, then the increase in the
energies of all the d-orbitals is the same. However, under the influence of octahedral field, the energies of the
d d
dorbitals lying along the axis (i.e. z2 and x2− y 2 ) increases more than the dorbitals

lying between the axis (i.e. dxy, dyz and dxz). Thus, the degenerate dorbitals (with no field effect or spherical
field effect) splits up into two sets of orbitals (i) the lower energy set,
d d z2
t2g (dxy, dyz and dxz) and (ii) the higher energy set, e g ( x2− y 2 and ). The energy separation is denoted by
o or 10 Dq. (where o stands for octahedral field), as shown below:

eg
d x2  y 2 , dz2

6 Dq

Energy 10 Dq

4 Dq

t 2g
d xy , d yz , d xz
Average energy of the
dorbitals in dorbitals in spherical Splitting of dorbitals in
free ion crystal field an octahedral crystal field

Significance of o

A strong field ligand approaches the central metal atom/ion strongly and thus
the magnitude of o is high. Hence, in the case of strong field ligand, the magnitude of o is greater than the
pairing energy (the energy required to pair up two negatively charged electrons having opposite spin in an
orbital). However, under the influence of weak field ligand, o < P (where P represents the pairing energy).

India’s first Colour STUDY MATERIALS for JEE, NEET ,OUAT & OTHERS ENTRANCE EXAMINATION Page 26
Now, let us consider the d4 configuration of the central metal atom/ion. The first three electrons will go into t 2g
orbitals using Hund’s rule of maximum multiplicity. The fourth electron will go in the e g orbital when the

ligands are weak as, o< P giving the configuration t32g e1g . But if the ligands are strong then the fourth

electron will pair up with any of the singly occupied t 2g orbitals (as o > P) to give the configuration t 42g e 0g .

Crystal field effects in tetrahedral coordination entities

Under the influence of tetrahedral field, the d-orbital splitting is smaller as compared to the octahedral field
4
splitting. For the same metal, the same ligands and metal-ligand distances, it can be shown that t = 9 o.
Consequently the orbital splitting energies are not sufficiently large for forcing pairing and therefore low spin
or spin paired configurations are rarely observed.

t 2g
d xy , d yz , d xz

4 Dq

10 Dq
Energy
6 Dq

eg
d x2 y 2 , dz 2
Average energy of
dorbitals in the d orbitals in Splitting of dorbitals in
free ion spherical crystal field tetrahedral crystal field

LIMITATIOIN OF CRYSTAL FIELD THEORY

1. The assumption that the interaction between metal- ligand is purely electrostatic
cannot be said to be very realistic.
Limitation

2. This theory takes only d-orbitals of a central atom into account. The s and p orbits are
not considered for the study.
3. The theory fails to explain the behavior of certain metals which cause large splitting
while others show small splitting. For example, the theory has no explanation as to
why H2O is a stronger ligand as compared to OH–.
4. The theory rules out the possibility of having p bonding. This is a serious drawback
because p bonding is found in many complexes.
5. The theory gives no significance to the orbits of the ligands. Therefore it cannot
explain any properties related to ligand orbitals and their interaction with metal
orbitals

24 ISOMERISM
The compounds having same chemical formula but different structural arrangement of their atoms and hence
different physical and chemical properties are called isomers and the phenomenon is called isomerism.
Isomerism in complexes are of two types:
(i) Structural Isomerism
(ii) Stereoisomerism

India’s first Colour STUDY MATERIALS for JEE, NEET ,OUAT & OTHERS ENTRANCE EXAMINATION Page 27
Isomerism

Structural Stereoisomerism
isomerism

(i) Ionisation Isomerism (i) Geometrical or cis-trans isomerism


(ii) Hydrate isomerism (ii) Optical isomerism
(iii) Ligand isomerism
(iv) Coordination isomerism
(v) Linkage isomerism
(vi) Coordination position isomerism
(vii) Polymerisation isomerism

STRUCTURAL ISOMERISM
This isomerism arises due to the difference in structures of coordination compounds and are of the following
types.
(a) Ionisation Isomerism
Complexes that have the same empirical formula and are produced by the interchange of the position of the
ligands inside the complex zone and outside the complex zone are called ionisation isomers. They give
different ions e.g.

(i) [Co(NH3)4Cl2]NO2 [Co(NH3)4Cl2]+ + NO 2
[Co(NH3)4Cl(NO2)]Cl [Co(NH3)4Cl(NO2)]+ + Cl–

(ii) [Co(NH3)5SO4]Br [Co(NH3)5SO4]+ + Br
2−
[Co(NH3)5 Br]SO4 [Co(NH3)5 Br]++ + SO 4
The number of ions in a solution can be determined by conductivity measurement.
More the number of ions in a solution more is the conductivity. Greater the charge on ions, greater is the
conductivity of solution.

(b) Hydrate isomerism


This type of isomerism arises due to the different position of water molecules inside and outside the coordination
sphere. For example,
(i) [Cr(H2O)6]Cl3 (violet), does not lose water over H2SO4 and all Cl ions are immediately precipitated by
(Ag+) ions.
(ii) [Cr(H2O)5Cl]Cl2H2O (green), loses H2O over H2SO4 and two Cl ions are precipitated by (Ag+) ions.

(iii) [Cr(H2O)4 Cl2] Cl2H2O (green), loses two water molecules over H2SO4 and only one Cl ion is
precipitated by Ag+ ions.

(c) Ligand Isomerism


Some ligands themselves are of capable of existing as isomers, for example diamino propane can exist both as
1, 2–diaminopropane (pn) and 1,3–diaminopropane, also called trimethylenediamine (tn)
1 2 3 1 2 3 1 2 3
CH3 – CH2 – CH3 CH2 – CH – CH3 CH2 – CH2 – CH2
Propane NH2 NH2 NH2 NH2
1,2–diamino propane (pn) 1,3–diamino propane (tn)
When these ligands (for example, pn and tn) are associated to form complexes,
the complexes are isomers of each other.
e.g. [Co(pn)2Cl2]+ and [Co(tn)2Cl2]+ ions.

India’s first Colour STUDY MATERIALS for JEE, NEET ,OUAT & OTHERS ENTRANCE EXAMINATION Page 28
(d) Coordination Isomerism
If both cation and anion of a complex compound are complex, there may be an exchange of ligands between
the two coordination spheres, giving rise to isomers known as coordination isomers. e.g.
(i) [Co(NH3)6] [Cr(CN)6] and [Cr(NH3)6] [Co(CN)6]
(ii) [Cu(NH3)4] [PtCl4] and [Pt(NH3)4] [CuCl4]

(e) Linkage Isomerism


Those complexes in which the ligands can coordinate with the central metal ion through either of the two
atoms, give rise to the linkage isomerism.

The best known ligands of this type are NO –2 , SCN



and S 2 O 2−
3 ions. In complexes containing
NO –2 ion as ligand, NO –2
ion may attach with the central ion either through
O-atom or through N-atom.
(i) [Co(NH3)5 (NO2)] Cl2  Pentaamminenitrocobalt(III) chloride.
[Co(NH3)5 (ONO)]Cl2  Pentaamminenitritocobalt(III) chloride.

(f) Coordination Position Isomerism


In some poly-nuclear complexes, interchange of the ligands between the metal atoms which are present as a
part of the complex is possible. This type of interchange of ligands between the metal atoms gives rise to
coordination position isomerism for example,
NH2
[(NH3)4 Co Co (NH3)2 Cl2]+2 (unsymmetrical form)
O2
and
NH2
[Cl (NH3)3 Co Co (NH3)3 Cl]+2 (symmetrical form)
O2
are coordination position isomers.

(g) Polymerisation Isomerism


This is not the true isomerism because it occurs between compound having the same empirical formula, but
different molecular weights. For example, [Pt(NH 3)2Cl2], [Pt(NH3)4] [Pt(Cl4), [Pt(NH3)4] [Pt(NH3)Cl3]2 and
[Pt(NH3)3Cl]2 [PtCl4] all have the same empirical formula.

STEREOISOMERISM OR SPACE ISOMERISM


When two compounds contain the same ligands coordinated to the same central ion, but the arrangement of
ligands in the space is different, the two compounds are said to be stereoisomers and the type of isomerism is
called stereoisomerism.
Stereoisomerism is of two types:-
(i) Geometrical or cis-trans isomerism
(ii) Optical or mirror image isomerism.
Geometrical isomerism
Geometrical isomerism is due to ligands occupying different positions around the central metal ion. The
ligands occupy positions either adjacent to one another or opposite to one another. These are referred to as cis-
form and trans-form respectively. This type of isomerism is, therefore, also referred to as cis-trans isomerism.

(a) Geometrical isomerism in 4–coordinate complex


Complexes having central metal atom with coordination number = 4 may be either tetrahedral or square
planar. Geometrical isomerism cannot arise in tetrahedral complexes because this geometry contains all the
ligands in the cis (i.e., adjacent) position with respect to each other, i.e., each ligand is equidistant from the
other three ligands and all the bond angles are the same (109.5°). Hence geometrical isomerism can not be
expected in tetrahedral complexes.

India’s first Colour STUDY MATERIALS for JEE, NEET ,OUAT & OTHERS ENTRANCE EXAMINATION Page 29
Square planar complexes of [Ma 4], [Ma3b] and [Mab3] type (a and b are monodentate ligands) do not show
geometrical isomerism, since every conceivable spatial arrangement of the ligands a round the metal ion is
exactly equivalent.
(1) [Ma2b2] type complexes
a b b a

Mn+ Mn+

a cis-form b
a trans-form b
e.g. [Pt+2 (NH3)2 Cl2], [Pt2+(NH3)2 Br2] and
[Pd2+ (NH3)2(NO2)2] are square planar complexes which exhibit cis-trans isomerism.
(2) [Mabcd] type complexes
Square planar complexes of this type exist in three isomeric forms for example, [Pt 2+ (NH3) (Py) (Cl) (Br)]
exist in the following structures.
NH3 Py NH3 Py NH3 Br

Pt2+ Pt2+ Pt2+

Cl Br Br Cl Cl Py
2+ + 2+
[Pt (NO2)(Py) (NH3) (NH2OH)] and [Pt (C2H4) (NH3) (Cl) (Br)] are other examples of square planar
complexes which exist in three isomeric forms.
(3) [Ma2bc] type complexes
Square planar complexes of this type also show cis-trans isomerism. For example,
[Pd2+Cl2BrI]2– exists in the following cistrans
Br Cl 2 Br Cl 2–

Pd2+ Pd2+

I cis-isomer Cl Cl I
trans-isomer
(4) [M(AB)2]n type complexes
Here M is the central metal ion and (AB) represents an unsymmetrical bidentate ligand. (A) and (B) are the two
ends (i.e., coordinating atoms) of the bidentate ligand. Such type of complexes also show cis and trans
isomerism.
A A n A B n

M M

B cis-form B B A
trans-form

For example, [Pt2+(gly)2] ; Here gly represents the glycinato ligand, NH 2CH2COO– which has N and O atoms as its
donor atoms.
H2 N N
H2C–H2N N–CH2

Pt2+ gly Pt2+ gly


or

O=C–O O–C=O O
cis-isomer trans-form O

India’s first Colour STUDY MATERIALS for JEE, NEET ,OUAT & OTHERS ENTRANCE EXAMINATION Page 30
H2 N
H2C–N O
O–C=O

gly Pt2+
Pt2+ or gly

O=C–O N–CH2 O
trans-isomer H2 trans-form N

(b) Geometrical isomerism in 6–coordinate complexes


A complex compound having the central metal ion with coordination number equal to 6 has octahedral shape.
The system used for numbering different positions of the ligands in an octahedral geometry has been shown
below.
1
5
2
Positions 1 and 2: cis
M Positions 1 and 6: trans

4 3
6
The octahedral complexes of the types [Ma6], [M(AA)3] and [Ma5b] do not show geometrical isomerism.
The following octahedral complexes give two or more geometrical isomers
(1) [Ma4b2]n type complexes:
b b
a b n a a n

M M

a a a a
a b
cis–form trans–form

Examples of such complexes are [Co3+(NH3)4Cl2]+ , [Co3+(NH3)4(NO2)2]+ etc.

(2) [Ma3b3]n type complexes:


a a
b a n b a n

M M

b a b b
b a
Facial (fac) Meridional (mer)

(When each trio of donor atoms (viz the ligands a, a and a) occupy adjacent positions at the corners of an octahedral
face, we have facial (fac) isomer. When the positions are around the meridian of the octahedron, we get Meridional
(mer) isomer.
Complexes like [Co3+(NH3)3Cl3], [Cr3+(H2O)3F3], [Pt3+(Py)3Cl3], [Cr3+(NH3)3Cl3], [Ru3+(H2O)3Cl3], [Pt4+(NH3)3Br3]+ etc
exhibit such type of isomerism.

(3) [Ma2b2c2] type complexes


Octahedral complexes of this type can exist theoretically in five geometrical isomers. Out of these only three
have been prepared.
For example, consider [Pt4+(NH3)2(Py)2Cl2]2+ ion. It can theoretically exist in the following structures.

India’s first Colour STUDY MATERIALS for JEE, NEET ,OUAT & OTHERS ENTRANCE EXAMINATION Page 31
Cl
Py NH3 Cl NH3
2+ Py NH3 Py
2+ Cl 2+

Pt4+
Pt4+ Pt4+

NH3 Py
Cl Py NH3 Py Cl
Cl NH3
(i)
(ii) (iii)
Py Py
Cl NH3 2+ Cl Py 2+

Pt4+ Pt4+

Cl NH3 Cl NH3
Py NH3
(iv) (v)

(4) [M(AA)2a2] type complexes: Here (AA) represents a symmetrical bidentate ligand in which A and A are two
identical co-ordinating atoms.
Examples of such complexes are [Co3+(en)2Cl2]+, [Co3+(en)2(NH3)2]3+, [Co3+(en)2(NO3)2]+, [Cr3+(en)2Cl2]+, [Cr3+
(C2O4)2 (H2O)2] (where en = ethylenediamine).
en N + Cl
N Cl N +
N

Co3+ en Co3+ en

N
Cl N N
en N Cl
cisisomer transisomer

25 ZINC COMPOUNDS
Zinc Oxide (Zincwhite or Chinese white or Philosopher’s wool) ; ZnO
Preparation:
2Zn + O2 ⃗
Δ 2ZnO
ZnCO3 ⃗ Δ ZnO + CO2
2Zn(NO3)2 ⃗Δ 2ZnO + 4NO2 + O2
Zn(OH)2 ⃗ Δ ZnO + H2O
Very pure zinc oxide is prepared by mixing a solution of zinc sulphate with sodium carbonate. The
precipitated basic zinc carbonate on heating gives pure zinc oxide.
4ZnSO4 + 4Na2CO3 + 3H2O  ZnCO3·3Zn(OH)2 + 4Na2SO4 + 3CO2
White ppt

ZnCO3·3Zn(OH)2 ⃗
Heat 4ZnO + 3H2O +CO2
Properties
(i) It is a white powder. It becomes yellow on heating and again turns white on cooling.
(ii) It is very light. It is insoluble in water. It sublimes at 400ºC.
(iii) It is an amphoteric oxide and dissolves readily in acids forming corresponding zinc salts and alkalies
forming zincates.
ZnO + H2SO4  ZnSO4 + H2O
ZnO + 2HCl  ZnCl2 + H2O
ZnO + 2NaOH  Na2ZnO2 + H2O
Sodium zincate
(iv) When heated in hydrogen above 400ºC, it is reduced to metal.
India’s first Colour STUDY MATERIALS for JEE, NEET ,OUAT & OTHERS ENTRANCE EXAMINATION Page 32
ZnO + H2  Zn + H2O
It is also reduced by carbon into zinc.
ZnO + C  Zn + CO
(v) When zinc oxide is heated with cobalt nitrate, a green mass is formed due to formation of cobalt zincate,
which is known as Rinmann’s green.
2Co(NO3)2  2CoO + 2NO2 + O2
ZnO + CoO  CoZnO2 or CoO·ZnO

Zinc Chloride, ZnCl2 · 2H2O


Preparation :
ZnO + 2HCl ⃗
Δ ZnCl2 + H2O
ZnCO3 + 2HCl ⃗ Δ ZnCl2 + CO2 + H2O
Zn(OH)2 + 2HCl ⃗
Δ ZnCl2 + 2H2O
Anhydrous zinc chloride cannot be obtained by heating crystals of hydrated zinc chloride as hydrolysis occurs
and basic chloride (zinc hydroxy chloride) is formed which on further heating gives zinc oxide.
ZnCl2·2H2O ⃗
Δ Zn(OH)Cl + HCl + H2O
Zn(OH)Cl ⃗ Δ ZnO + HCl
The anhydrous zinc chloride is obtained by heating zinc in the atmosphere of dry chlorine or dry HCl gas.
Zn + Cl2  ZnCl2
Zn + 2HCl  ZnCl2 + H2
This can also be formed by distilling zinc powder with mercuric chloride.
Zn + HgCl2  ZnCl2 + Hg
Properties:
(i) Anhydrous zinc chloride is a white solid, deliquescent and soluble in water. It melts
at 660ºC and boils at 730ºC.
(ii) Hydrated zinc chloride on heating forms zinc hydroxy chloride or zinc oxychloride.

ZnCl2·2H2O ⃗
Δ Zn(OH)Cl + HCl + H2O
2ZnCl2·2H2O ⃗
Δ Zn2OCl2 + 2HCl + 3H2O
Zinc oxychloride
(iii) When H2S is passed through the solution, a white precipitate of zinc sulphide is formed.
ZnCl2 + H2S  ZnS + 2HCl
White ppt
(iv) When NaOH is added, a white precipitate of zinc hydroxide appears which dissolves in excess of sodium
hydroxide forming sodium zincate.
ZnCl2 + 2NaOH  Zn(OH)2 + 2NaCl
White ppt
Zn(OH)2 + 2NaOH  Na2ZnO2 + 2H2O
(v) On adding NH4OH solution, a white precipitate of zinc hydroxide appears which dissolves in excess of
ammonia forming a complex salt.
ZnCl2 + 2NH4OH  Zn(OH)2 + 2NH4Cl
White ppt
Zn(OH)2 + 2NH4OH + 2NH4Cl  [Zn(NH3)4]Cl2 + 4H2O
Tetramminezinc(II) chloride
(vi) When the solution of zinc chloride is treated with a solution of sodium carbonate, a white precipitate of
basic zinc carbonate is formed.
4ZnCl2 + 4Na2CO3+3H2O  ZnCO3·3Zn(OH)2 + 8NaCl+3CO2
Basic zinc carbonate
White ppt
But when a solution of sodium bicarbonate is used, a white precipitate of normal zinc carbonate is formed.

India’s first Colour STUDY MATERIALS for JEE, NEET ,OUAT & OTHERS ENTRANCE EXAMINATION Page 33
ZnCl2 + 2NaHCO3  ZnCO3 + 2NaCl + H2O + CO2
White ppt
(vii) Anhydrous zinc chloride absorbs ammonia gas and forms an addition compound.
ZnCl2 + 4NH3  ZnCl2·4NH3
(viii) Its syrupy solution dissolves cellulose. Its syrupy solution when mixed with zinc oxide (ZnO)
sets to a hard mass forming an oxychloride, ZnCl2·3ZnO.

Zinc Sulphate (White Vitriol) ZnSO4·7H2O


Preparation :
Zn + H2SO4 (dil.)  ZnSO4 + H2
ZnO + H2SO4 (dil.)  ZnSO4 + H2O
ZnCO3 + H2SO4 (dil.)  ZnSO4 + H2O + CO2
Properties:
(i) It is a colourless, crystalline solid. It is an efflorescent substance. It is freely soluble in water.
(ii) On heating, the following changes occur.
Above 39ºC Above 70ºC
ZnSO4·7H2O ZnSO4·6H2O ZnSO4·H2O
Below 70ºC 5H2O
H2O 800ºC
H2O
Above
800ºC
O2 + SO2 +ZnO ZnSO4
(anhydrous)

ZnSO4 ⃗
800 ° C ZnO + SO3
SO3 ⃗Δ SO2 + ½O2
(iii) When sodium hydroxide is added to the solution of zinc sulphate, a white precipitate of zinc hydroxide
appears which dissolves in excess of NaOH forming sodium zincate.
ZnSO4 + 2NaOH  Zn(OH)2 + Na2SO4
White ppt
Zn(OH)2 + 2NaOH  Na2ZnO2 + 2H2O
(iv) When sodium carbonate solution is added to the solution of zinc sulphate, a white precipitate of basic zinc
carbonate is formed.
4ZnSO4 + 4Na2CO3 + 3H2O  ZnCO3·3Zn(OH)2 + 4Na2SO4 + 3CO2
White ppt
However, when the solution of sodium bicarbonate is added, normal zinc carbonate is formed.
ZnSO4 + 2NaHCO3  ZnCO3 + Na2SO4 + H2O + CO2
White ppt
(v) With alkali metal sulphates and (NH4)2SO4, it forms double sulphates such as K2SO4·ZnSO4·6H2O.

26 SILVER COMPOUNDS

Silver Nitrate (Lunar caustic), AgNO3


Preparation:
3Ag + 4HNO3 ⃗
Heat 3AgNO3 + NO + 2H2O
(Dilute)
Properties:
(i) It is a colourless crystalline compound, soluble in water and alcohol. It melts at 212ºC.
(ii) In contact with organic substances, it blackens due to decomposition into metallic silver. Thus, it leaves
black stain when comes in contact with skin and clothes.
It produces burning sensation like caustic and leaves a blackishwhite stain (moon like colour) on skin
and thus called as Lunar caustic. It is decomposed by light also and therefore stored in coloured bottles.
(iii) On heating above its melting point, it decomposes to silver nitrite and oxygen.

India’s first Colour STUDY MATERIALS for JEE, NEET ,OUAT & OTHERS ENTRANCE EXAMINATION Page 34
2AgNO3 ⃗ Δ 2AgNO2 + O2
When heated in a red hot tube, it decomposes to metallic silver.
2AgNO3 ⃗ Δ 2 Ag + 2NO2 + O2
(iv) Solutions of halides, phosphates, sulphides, chromates, thiocyanates, sulphates and thiosulphates, all give
a precipitate of the corresponding silver salt with silver nitrate solution.
AgNO3 + NaCl  AgCl + NaNO3
White ppt
AgNO3 + NaBr  AgBr + NaNO3
Pale yellow ppt
AgNO3 + NaI  AgI + NaNO3
Yellow ppt
3AgNO3 + Na3PO4  Ag3PO4 + 3NaNO3
Yellow ppt
2AgNO3 + K2CrO4  Ag2CrO4 + 2KNO3
Brick red ppt
AgNO3 + NaCNS  AgCNS + NaNO3
White ppt
2AgNO3 + Na2SO4  Ag2SO4 + 2NaNO3
White ppt
2AgNO3 + Na2S2O3  Ag2S2O3 + 2NaNO3
White ppt
Ag2S2O3 + H2O  Ag2S + H2SO4
Black ppt
(v) Solid AgNO3 absorbs ammonia gas with the formation of an addition compound, AgNO 3·3NH3.
(vi) When treated with a solution of NaOH, it forms precipitate of silver oxide. Originally, it has brown
colour but turns black when dried.
2AgNO3 + 2NaOH  Ag2O + 2NaNO3 + H2O
Brown ppt
(vii) When KCN is added to silver nitrate, a white precipitate of silver cyanide appears which dissolves in
excess of KCN forming a complex salt, potassium argentocyanide.
AgNO3 + KCN  AgCN + KNO3
White ppt
AgCN + KCN  K[Ag(CN)2]
Potassium argentocyanide
(viii)When sodium thiosulphate is added to silver nitrate, a white precipitate of silver thiosulphate appears.
This precipitate, however, dissolves in excess of sodium thiosulphate forming a complex salt.
2AgNO3 + Na2S2O3  Ag2S2O3 + 2NaNO3
White ppt
Ag2S2O3 + 3Na2S2O3  2Na3[Ag(S2O3)2]
Sodium argentothiosulphate
(ix) AgNO3 reacts with iodine in two ways:
(a) 6AgNO3 (excess) + 3I2 + 3H2O  AgIO3 + 5AgI + 6HNO3
Yellow ppt
(b) 5AgNO3 + 3I2 (excess) + 3H2O  HIO3 + 5AgI + 5HNO3
Yellow ppt
(x) Silver is readily displaced from an aqueous silver nitrate solution by the base metals, particularly, if the
solution is somewhat acidic.
2AgNO3 + Cu  2Ag + Cu(NO3)2
2AgNO3 + Zn  2Ag + Zn(NO3)2
(xi) Phosphine, arsine and stibine all precipitate silver from silver nitrate solution.
PH3 + 6AgNO3 + 3H2O  6Ag + 6HNO3 + H3PO3
AsH3 + 6AgNO3 + 3H2O  6Ag + 6HNO3 + H3AsO3
India’s first Colour STUDY MATERIALS for JEE, NEET ,OUAT & OTHERS ENTRANCE EXAMINATION Page 35
(xii) All halogen acids, except HF, precipitate silver halides from aqueous solution of AgNO 3.
AgNO3 + HX  AgX + HNO3
Silver fluoride (AgF) is soluble in water.
(xiii)When NH4OH is added to silver nitrate solution, a brown precipitate of silver oxide appears which
dissolves in excess of ammonia forming a complex salt.
2AgNO3 + 2NH4OH  Ag2O + 2NH4NO3 + H2O
Brown ppt
2Ag2O + 2NH4NO3 + 2NH4OH  2[Ag(NH3)2]NO3 + 3H2O
The ammonical solution of AgNO3 gives the following reaction:
(a) It reacts with acetylene to form white precipitate of silver acetylide.
2AgNO3 + 2NH4OH + C2H2  Ag2C2 + 2NH4NO3 + 2H2O
Silver acetylide
White ppt
(b) It converts glucose to gluconic acid.
Ag2O + C6H12O6  2Ag + C6H12O7
Silver mirror
(c) It oxidises formaldehyde to formic acid.
Ag2O + HCHO  2Ag + HCOOH
Silver mirror
Silver Oxide, (Ag2O)
Preparation:
2AgNO3 + 2NaOH  Ag2O + 2NaNO3 + H2O
Brown ppt
Properties:
It is brownish powder insoluble in water and thermally unstable. It decomposes to silver and oxygen.
2Ag2O  4Ag + O2
Ag2O is soluble in aqueous ammonia.
Silver thiosulphate, Ag2S2O3
Preparation:
Addition of more than the equivalent amount of sodium thiosulphate to a solution of silver acetate or fluoride,
when a white precipitate of silver thiosulphate is formed.
2AgF + Na2S2O3  2NaF + Ag2S2O3
Properties:
It forms needlelike crystals. It dissolves in excess of sodium thiosulphate solution producing a complex.
Ag2S2O3 + 3Na2S2O3  2Na3[Ag(S2O3)2]
Silver thiosulphate is decomposed by water giving play of colour test, changing from white to black through
yellow and brown, when silver nitrate solution is mixed with dilute sodium thiosulphate solution.
Na2S2O3 + 2AgNO3  2NaNO3 + Ag2S2O3
Ag2S2O3 + H2O  H2SO4 + Ag2S
Cupric Oxide, CuO (Black oxide of Copper)
Preparation
It is prepared by the following methods.
(i) By heating Cu2O in air or by heating copper for a long time in air. The temperature should not exceed
above 1100ºC.
⃗Δ
Cu2O + ½ O2 ¿ 1100o C 2CuO

Δ
2Cu + O2 ¿ 1100o C 2CuO
(ii) By heating cupric hydroxide also, cupric oxide can be obtained.
Cu(OH)2 ⃗ Δ CuO + H2O
(iii) By heating copper nitrate also, cupric oxide can be obtained.

India’s first Colour STUDY MATERIALS for JEE, NEET ,OUAT & OTHERS ENTRANCE EXAMINATION Page 36
2Cu(NO3)2 ⃗ Δ 2CuO + 4NO2 + O2
(iv) On commercial scale, it is obtained by heating malachite, which is found in nature.
CuCO3·Cu(OH2)2 ⃗
Δ 2CuO + CO2 + H2O
Properties:
(i) It is a black powdery substance and is stable towards moderate heating.
(ii) The oxide is insoluble in water but dissolves in acids forming corresponding salts.
CuO + 2HCl  CuCl2 + H2O
CuO + H2SO4  CuSO4 + H2O
CuO + 2HNO3  Cu(NO3)2 + H2O
(iii) When heated to 11001200ºC, it is converted into cuprous oxide with evolution of oxygen.
4CuO  2Cu2O + O2
(iv) It is reduced to metallic copper by reducing agents such as hydrogen, carbon and carbon monoxide.
CuO + H2  Cu + H2O
CuO + C  Cu + CO
CuO + CO  Cu + CO2

Cupric Chloride (CuCl2·2H2O)


Preparation:(i) The metal or cupric oxide or cupric hydroxide or copper carbonate is dissolved in
concentrated HCl. The resulting solution on crystallization gives green crystals of hydrated cupric chloride.
2Cu + 4HCl + O2  2CuCl2 + 2H2O
CuO + 2HCl  CuCl2 + H2O
CuCO3.Cu(OH)2 + 4HCl  2CuCl2 + 3H2O + CO2
(ii) Anhydrous cupric chloride is obtained as a dark brown mass when copper metal
is heated in excess of chlorine gas or by heating hydrated cupric chloride in HCl gas
at 150ºC.
Cu + Cl2  CuCl2

150O c
CuCl2·2H2O HCl gas CuCl2 + 2H2O

Properties:
(i) It is a deliquescent compound and is readily soluble in water. The dilute solution is blue but the
concentrated solution is green. It changes to yellow when concentrated HCl is added. The blue colour is
due to complex cation [Cu(H2O)4]2+ and yellow colour is due to the complex anion [CuCl 4]2 and green
when both are present.
(ii) The aqueous solution is acidic due to hydrolysis of Cu2+.
CuCl2 + 2H2O Cu(OH)2 + 2HCl
(iii) The anhydrous salt on heating forms Cu2Cl2 and Cl2.
2CuCl2  Cu2Cl2 + Cl2
While the hydrated salt on strong heating gives CuO, Cu2Cl2, HCl and Cl2.
3CuCl2·2H2O  CuO + Cu2Cl2 + 2HCl + Cl2 + 5H2O
(iv) It is readily reduced to Cu2Cl2 by copper turnings or SO2 gas or hydrogen
(nascent form obtained by the action of HCl on Zn) or SnCl 2.
CuCl2 + Cu  Cu2Cl2
2CuCl2 + SO2 + 2H2O  Cu2Cl2 + 2HCl + H2SO4
2CuCl2 + 2[H]  Cu2Cl2 + 2HCl
2CuCl2 + SnCl2  Cu2Cl2 + SnCl4
(v) A pale blue precipitate of basic cupric chloride, CuCl 2·3Cu(OH)2 is obtained when NaOH is added.
CuCl2 + 2NaOH  Cu(OH)2 + 2NaCl
CuCl2 + 3Cu(OH)2  CuCl2·3Cu(OH)2

India’s first Colour STUDY MATERIALS for JEE, NEET ,OUAT & OTHERS ENTRANCE EXAMINATION Page 37
Blue ppt
It dissolves in ammonium hydroxide forming a deep blue solution. On evaporating this solution,
deepblue crystals of tetramminecupric chloride are obtained.
CuCl2 + 4NH4OH  Cu(NH3)4Cl2·H2O + 3H2O
Copper Sulphate (Blue Vitriol) CuSO4·5H2O
Preparation:
(i) Copper sulphate is prepared in the laboratory by dissolving cupric oxide or hydroxide or carbonate in
dilute sulphuric acid. The solution is evaporated and crystallized.
CuO + H2SO4  CuSO4 + H2O
Cu(OH)2 + H2SO4  CuSO4 + 2H2O
CuCO3.Cu(OH)2 + 2H2SO4  2CuSO4 + 3H2O + CO2
(ii) On a commercial scale, it is prepared from scrap copper. The scrap copper is placed in a perforated
lead bucket, which is dipped into hot dilute sulphuric acid. Air is blown through the acid. Copper sulphate
is crystallized from the solution.
Cu + H2SO4 + ½ O2 (air)  CuSO4 + H2O
Properties:
(i) It is a blue crystalline compound and is fairly soluble in water.
(ii) Heating effect
CuSO4·5H2O crystals effloresce on exposure to air and are converted into a pale blue powder,
CuSO4·3H2O. When heated to 100ºC, bluish white monohydrate CuSO4·H2O is formed. The monohydrate
loses the last molecule of water at 230ºC giving the anhydrous salt of CuSO 4, which is white.
Exposure 100ºC 230ºC

CuSO4·5H2O CuSO4·3H2O CuSO4·H2O CuSO4


2H2O Pale Blue 2H2O Bluish white H2O White
Blue

Anhydrous copper sulphate (white) regains its blue colour when moistened with a drop of water (test
of water).
If the anhydrous salt is heated at 720ºC, it decomposes into cupric oxide and sulphur
trioxide. CuSO4 ⃗
720 ° C CuO + SO3
SO3  SO2 + ½O2

(iii) Action of NH4OH


With ammonia solution, it forms the soluble blue complex. First it forms a precipitate of Cu(OH) 2 which
dissolves in excess of ammonia solution.
CuSO4 + 2NH4OH  Cu(OH)2 + (NH4)2SO4
Blue ppt
Cu(OH)2 + 2NH4OH + (NH4)2SO4  Cu(NH3)4SO4 + 4H2O
Tetramminecupric sulphate
The complex is known as Schweitzer’s reagent, which is used for dissolving cellulose in the manufacture
of artificial silk.
(iv) Action of alkalies
With alkalies, CuSO4 forms a pale blue precipitate of copper hydroxide.
CuSO4 + 2NaOH  Cu(OH)2 + Na2SO4
Blue ppt
(v) Action of potassium iodide
First cupric iodide is formed, which decomposes to give white cuprous iodide
and iodine.
[CuSO4 + 2KI  CuI2 + K2SO4] x 2
2CuI2  Cu2I2 + I2
2CuSO4 + 4KI  Cu2I2 + 2K2SO4 + I2
(vi) Action of potassium cyanide
First cupric cyanide is formed which decomposes to give cuprous cyanide and cyanogen gas. Cuprous
cyanide dissolves in excess of potassium cyanide to form a complex, potassium cuprocyanide
[K3Cu(CN)4].
India’s first Colour STUDY MATERIALS for JEE, NEET ,OUAT & OTHERS ENTRANCE EXAMINATION Page 38
[CuSO4 + 2KCN  Cu(CN)2 + K2SO4] x 2
2Cu(CN)2  Cu2(CN)2 + (CN)2
Cu2(CN)2 + 6KCN  2K3Cu(CN)4
2CuSO4 + 10KCN  2K3Cu(CN)4 + 2K2SO4 + (CN)2

27 IRON COMPOUNDS
Ferrous Sulphate (Green Vitriol), FeSO47H2O
Preparation:
(i) By the oxidation of pyrites under the action of water and atmospheric air.
2FeS2 + 7O2 + 2H2O  2FeSO4 + 2H2SO4
(ii) It is obtained by dissolving scrap iron in dilute sulphuric acid.
Fe + H2SO4  FeSO4 + H2
The solution is crystallized by the addition of alcohol as ferrous sulphate is sparingly soluble in it.
(iii) It can also be prepared in the laboratory from the Kipp’s waste. Heating with a small quantity of iron fillings
neutralizes the excess of sulphuric acid. The solution is then crystallised.
(iv) Commercially, ferrous sulphate is obtained by the slow oxidation of iron pyrites in the presence of air and
moisture. The pyrites are exposed to air in big heaps.
2FeS2 + 2H2O + 7O2  2FeSO4 + 2H2SO4
The free sulphuric acid is removed by the addition of scrap iron. On crystallization, green crystals are obtained.
Properties:
(i) Hydrated ferrous sulphate (FeSO47H2O) is a green crystalline compound. Due to atmospheric
oxidation, the crystals acquire brownishyellow colour due to formation of basic ferric sulphate.
4FeSO4 + 2H2O + O2  4Fe(OH)SO4
Basic ferric sulphate
(ii) Action of heat
300 ° C ¿
⃗ ⃗
High
FeSO47H2O −¿ 7H2 O 2FeSO4 temp . Fe2O3 + SO2 + SO3
Ferrous ammonium sulphate (Mohr’s Salt)
(NH4)2SO4.FeSO4.6H2O

Preparation:
The double salt is best prepared by making saturated solutions of pure ferrous sulphate and pure ammonium sulphate
in air free distilled water at 40°C. Both the solutions are mixed and allowed to cool. Generally, few drops of sulphuric
acid and a little iron wire are added before crystallisation so as to prevent oxidation of ferrous sulphate into ferric
sulphate. The salt is obtained as pale green crystals.

Properties:
It is pale green crystalline compound, which does not effloresce like ferrous sulphate. It is less readily
oxidised in the solid state. It is, therefore, a better volumetric reagent in preference of ferrous sulphate.
Chemically, it is similar to ferrous sulphate. All the chemical reactions observed in the case of ferrous sulphate
are given by ferrous ammonium sulphate.
Ferric chloride (FeCl3)
This is the most important ferric salt. It is known in anhydrous and hydrated forms.
The hydrated form consists of six water molecules, FeCl 3.6H2O.
Preparation:
(i) Anhydrous ferric chloride is obtained by passing dry chlorine gas over heated iron fillings. The vapours are
condensed in a bottle attached to the outlet of the tube.
2Fe + 3Cl2  2FeCl3
(ii) Hydrated ferric chloride is obtained by the action of hydrochloric acid on ferric carbonate, ferric hydroxide or
ferric oxide.
Fe2(CO3)3 + 6HCl  2FeCl3 + 3H2O + 3CO2
Fe(OH)3 + 3HCl  FeCl3 + 3H2O

India’s first Colour STUDY MATERIALS for JEE, NEET ,OUAT & OTHERS ENTRANCE EXAMINATION Page 39
Fe2O3 + 6HCl  2FeCl3 + 3H2O
The solution on evaporation and cooling deposits yellow crystals of hydrated ferric chloride,
FeCl3.6H2O.
Properties:
(i) Anhydrous ferric chloride is a dark red deliquescent solid. It is sublimed at about 300°C and its vapour
density corresponds to dimeric formula, Fe 2Cl6. The dimer dissociates at high temperature to FeCl 3. The
dissociation into FeCl3 is complete at 750°C. Above this temperature, it breaks into ferrous chloride and
chlorine.
750°C Above 750°C
Fe2Cl6 2FeCl3 2FeCl2 + Cl2
(ii) Anhydrous ferric chloride behaves as a covalent compound as it is soluble in nonpolar solvents like
ether, alcohol, etc. It is represented by chlorine bridge structure.
Cl Cl Cl
Fe Fe
Cl Cl Cl
(iii) It dissolves in water. The solution is acidic in nature due to its hydrolysis as shown below.
FeCl3 + 3H2O Fe(OH)3 + 3HCl
The solution is stabilised by the addition of hydrochloric acid to prevent hydrolysis.
(iv) Anhydrous ferric chloride absorbs ammonia.
FeCl3 + 6NH3  FeCl3.6NH3
(v) Ferric chloride acts as an oxidising agent.
(a) It oxidises stannous chloride to stannic chloride.
2FeCl3 + SnCl2  2FeCl2 + SnCl4
(b) It oxidises SO2 to H2SO4.
2FeCl3 + SO2 + 2H2O  2FeCl2 + H2SO4 + 2HCl
(c) It oxidises H2S to S
2FeCl3 + H2S  2FeCl2 + 2HCl + S
(d) It liberates iodine from KI.
2FeCl3 + 2KI  2FeCl2 + 2KCl + I2
(e) Nascent hydrogen reduces FeCl3 into FeCl2.
FeCl3 + H  FeCl2 + HCl
(vi) When ammonium hydroxide is added to the solution of ferric chloride, a reddishbrown precipitate of
ferric hydroxide is formed.
FeCl3 + 3NH4OH  Fe(OH)3 + 3NH4Cl
(vii) When a solution of thiocyanate ions is added to ferric chloride solution, a deep red colouration is
produced due to formation of a complex salt.
FeCl3 + NH4CNS  Fe(SCN)Cl2 + NH4Cl
or FeCl3 + 3NH4CNS  Fe(SCN)3 + 3NH4Cl
(viii) Ferric chloride forms a complex, prussian blue with potassium ferrocyanide.
4FeCl3 + 3K4Fe(CN)6  Fe4[Fe(CN)6]3 + 12KCl
Prussian blue
(Ferri ferrocyanide)
(ix) On heating hydrated ferric chloride FeCl 3.6H2O, anhydrous ferric chloride is not obtained. It is changed to
Fe2O3 with evolution of H2O and HCl.
2[FeCl3.6H2O] ⃗ Heat Fe2O3 + 6HCl + 9H2O
Hydrated ferric chloride may be dehydrated by heating with thionyl chloride.
FeCl3.6H2O + 6SOCl2  FeCl3 + 12HCl + 6SO2
Ferrous oxide
Preparation:
Iron can burn in oxygen when heated, producing magnetic oxide of iron, Fe 3O4
(an equimolar mixture of FeO and Fe2O3).
3Fe + 2O2  Fe3O4

India’s first Colour STUDY MATERIALS for JEE, NEET ,OUAT & OTHERS ENTRANCE EXAMINATION Page 40
Pure iron has no action with pure water but steam, reacts with redhot iron liberating hydrogen and forming
Fe3O4.
3Fe + 4H2O  Fe3O4 + 4H2
Ferrous oxide (FeO) can be found wherever there is iron exposed to the oxygen in the atmosphere.
The oxide is a black powder, formed by heating ferric oxide with hydrogen at 300°C or by heating ferrous
oxalate in absence of air at 160°170°C.

Fe2O3 + H2 ⃗
300o C 2FeO + H2O

FeC2O4 160−170o C
⃗ FeO + CO + CO2

Properties:
(i) Iron oxide is naturally black in colour and it appears as a solid crystalline substance
at room temperature.
(ii) Melting point of FeO is 1370°C and its density is 5.7 g/cm3.
(iii) Iron oxide will decompose into its elements before boiling.
(iv) It is oxidized in air with incandescence (pyrophoric).
(v) It is sparingly soluble in water and as a basic oxide dissolves in dilute acids to give ferrous salts.
(vi) Iron oxide is commonly used as a pigment for colouring all sorts of materials like paints, plastics and
rubber. It is also used for the dye in tattoos.
(vii) The iron and oxide ions in iron oxide are bonded with ionic bonds, making iron oxide a salt. There is a 1 :
1 ratio of iron ions to oxide ions and being a salt, iron oxide does not have individual molecules, but forms
geometrical structure with all of the ions bonded by electrostatic forces.
Passivity of Iron
The inertness exhibited by metals under conditions when chemical activity is to be expected is called chemical
passivity. The following are the common properties of iron.
(a) It evolves hydrogen gas, when made to react with dilute HCl or dilute H 2SO4.
(b) It precipitates silver from silver nitrate solution and copper from copper sulphate solution.
But if a piece of iron is first dipped in concentrated nitric acid for sometime and then made to react with the
above regents, neither hydrogen is evolved nor silver or copper are precipitated. Thus, iron by treatment with
concentrated nitric acid has lost its usual properties or it has been rendered inert or passive. Such behaviour is
not only shown by iron but also by many other metals like Cr, Co, Ni, Al etc. This phenomenon is known as
passivity and the chemical substances, which bring passivity, are called passivators.
Other oxidising agents can render iron passive like chromic acid, KMnO 4, concentrated H2SO4 etc. The
passivity of the iron is believed to be due to formation of an extremely thin film (invisible) of oxide on the
surface of iron. Passive iron can be made active by scratching or heating in a reducing atmosphere of H 2 or
CO, or heating in HNO3 upto 75°C.

Corrosion of Iron
Corrosion is defined as the gradual transformation of a metal into its combined state because of the reaction
with the environment. Metals are usually extracted from their ores. Nature tries to convert them again into the
ore form. The process, by which the metals have the tendency to go back to their combined state, is termed
corrosion.
Rust (Fe2O3  H2O)
Water
Fe2+ Fe2+

Cathode
Iron
Anode
e

When iron is exposed to moist air, it is found to be covered with a reddishbrown coating, which can easily be
detached. The reddish brown coating is called ‘rust’. Thus, the corrosion of iron or formation of the rust is
called rusting. The composition of the rust is not certain but it mainly contains hydrated ferric oxide,
2Fe2O3.3H2O, together with a small quantity of ferrous carbonate. The rust is formed by the action of water on

India’s first Colour STUDY MATERIALS for JEE, NEET ,OUAT & OTHERS ENTRANCE EXAMINATION Page 41
iron in the presence of dissolved oxygen and carbon dioxide. It has been observed that impure iron is more
prone to rusting.
The following are the favourable conditions for the rusting or iron
(i) Presence of moisture
(ii) Presence of a weakly acidic atmosphere
(iii) Presence of impurity in the iron.
Various theories have been proposed to explain the phenomenon of rusting of iron but the most accepted
theory is the modern electrochemical theory. When impure iron comes in contact with water containing
dissolved carbon dioxide, a voltaic cell is set up. The iron and other impurities act as electrodes while water
having dissolved oxygen and carbon dioxide acts as an electrolyte. Iron atoms pass into the solution as ferrous
ions.
Fe  Fe2+ + 2e
Iron, thus, acts as anode.
The impurities act as cathode. At the cathode, the electrons are used in forming hydroxyl ions.
H2O + O + 2e  2OH
In presence of dissolved oxygen, ferrous ions are oxidised to ferric ions, which combine with hydroxyl ions to
form ferric hydroxide.
Fe3+ + 3OH  Fe(OH)3
Rust
[2Fe2+ + H2O + O  2Fe3+ + 2OH]
Corrosion or rusting is a surface phenomenon and thus, the protection of the surface prevents the corrosion.
Iron can be protected from the rusting by use of following methods:
(i) Applying paints, lacquers and enamels on the surface of iron.
(ii) By forming a firm and coherent protective coating of ferrosoferric oxide. This is done by passing steam
over hot iron.
(iii) By coating a thin film of zinc, tin, nickel, chromium, aluminium, etc.

28 POTASSIUM PERMANGANET
Preparation
1. 2MnO2 + 4KOH + O2 ⃗
Fuse 2K2MnO4 + 2H2O
Pyrolusite Potassium manganates
(Green)
2MnO2 + 2K2CO3 + O2 ⃗ Fuse 2K2MnO4 + 2CO2
Instead of oxygen any other oxidising agent such as KNO 3 may also be used.
2. The fused mass is extracted with water and current of Cl 2 or O3 or CO2 is passed so as to convert
manganates into permanganate.
2K2MnO4 + Cl2  2KMnO4 + 2KCl
2K2MnO4 + H2O + O3  2KMnO4 + 2KOH + O2
3K2MnO4 + 2CO2  2KMnO4 + MnO2 + 2K2CO3
Manufacture
⃗ Fuse with KOH
MnO42−

electrolytic oxidation
MnO−4
oxidise with air or KNO 3 in alkaline solution
MnO2
Permanganate
Properties
(i) KMnO4 is a purple coloured crystalline compound. It is fairly soluble in water.
(ii) When heated alone or with an alkali, it decomposes evolving oxygen.
2KMnO4  K2MnO4 + MnO2 + O2
4KMnO4 + 4KOH  4K2MnO4 + 2H2O + O2
(iii) On treatment with concentrated H2SO4, it forms manganese heptaoxide which decomposes explosively on
heating.

India’s first Colour STUDY MATERIALS for JEE, NEET ,OUAT & OTHERS ENTRANCE EXAMINATION Page 42
2KMnO4 + 3H2SO4  2KHSO4 + (MnO3)2SO4 + 2H2O
(MnO3)2SO4 + H2O  Mn2O7 + H2SO4
3
O
Mn2O7  2MnO2 + 2 2
(iv) KMnO4 acts as an oxidising agent in alkaline, neutral or acidic solutions.
(a) In alkaline solution
KMnO4 is first reduced to manganate and then to insoluble manganese dioxide. Colour changes first from
purple to green and finally becomes colourless. However, brownish precipitate is formed.
2KMnO4 + 2KOH  2K2MnO4 + H2O + O
2K2MnO4 + 2H2O  2MnO2 + 4KOH + 2O
2KMnO4 + H2O ⃗
Alkaline 2MnO2 + 2KOH + 3[O]

or 2 MnO−4 + H2O  2MnO2 + 2OH + 3[O]


(b) In neutral solution
Brownish precipitate of MnO2 is formed.
2KMnO4 + H2O  2MnO2 + 2KOH + 3[O]

or 2 MnO−4 + H2O  2MnO2 + 2OH + 3[O]



or MnO4 + 2H2O + 3e  MnO2 + 4OH
(c) In acidic solution (in presence of dilute H2SO4)
Manganous sulphate is formed. The solution becomes colourless.
2KMnO4 + 3H2SO4  K2SO4 + 2MnSO4 + 3H2O + 5[O]

or 2 MnO−4 + 6H+  2Mn2+ + 3H2O + 5[O]



or MnO4 + 8H+ + 5e  Mn2+ + 4H2O
The important oxidation reactions are:
(i) Ferrous salts are oxidised to ferric salts.
2KMnO4 + 3H2SO4  K2SO4 + 2MnSO4 + 3H2O + 5[O]
[2FeSO4 + H2SO4 + [O]  Fe2(SO4)3 + H2O]  5
2KMnO4 + 10FeSO4 + 8H2SO4  5Fe2(SO4)3 + K2SO4 + 2MnSO4 + 8H2O

or 2 MnO 4 + 10Fe2+ + 16H+  10Fe3+ + 2Mn2+ + 8H2O
(ii) Iodide is evolved from potassium iodide.
2KMnO4 + 3H2SO4  K2SO4 + 2MnSO4 + 3H2O + 5[O]
[2KI + H2SO4 + [O]  K2SO4 + I2 + H2O]  5
2KMnO4 + 10 KI + 8H2SO4  6K2SO4 + 2MnSO4 + 5I2 + 8H2O

or 2 MnO 4 + 10I + 16H+  5I2 + 2Mn2+ + 8H2O
(iii) 2KMnO4 + 3H2SO4 + 5H2S  K2SO4 + 2MnSO4 + 5S + 8H2O
(iv) 2KMnO4 + 5SO2 + 2H2O  K2SO4 + 2MnSO4 + 2H2SO4
(v) 2KMnO4 + 5KNO2 + 3H2SO4  K2SO4 + 2MnSO4 + 5KNO3 + 3H2O
COOH
5
(vi) COOH + 2KMnO4 + 3H2SO4  K2SO4 + 2MnSO4 + 10CO2 + 8H2O
In neutral medium
(i) H2S is oxidised to sulphur
2KMnO4 + H2O  2MnO2 + 2KOH + 3[O]
[H2S + [O]  H2O + S]  3
2KMnO4 + 3H2S  2KOH + MnO2 + 2H2O + 3S
(ii) Manganese sulphate is oxidised to MnO2
2KMnO4 + H2O  2MnO2 + 2KOH + 3[O]
[MnSO4 + H2O + [O]  MnO2 + H2SO4]  3
2KOH + H2SO4  K2SO4 + 2H2O

India’s first Colour STUDY MATERIALS for JEE, NEET ,OUAT & OTHERS ENTRANCE EXAMINATION Page 43
2KMnO4 + 3MnSO4 + 2H2O  K2SO4 + 5MnO2 + 2H2SO4
(iii) Sodium thiosulphate is oxidised to sulphate and sulphur
2KMnO4 + 3Na2S2O3 + H2O  2KOH + 2MnO2 + 3Na2SO4 + 3S
In alkaline medium
(i) It oxidises ethylene to ethylene glycol.
CH2 CH2OH
+ H2O + [O]
CH2 CH2OH
In alkaline medium it is called Bayer’s reagent.

29 POTASSIUM DICHROMATE
Manufacture
1. 4FeO.Cr2O3 + 8Na2CO3 + 7O2  8Na2CrO4 + 2Fe2O3 + 8CO2
Chromite ore (from air)
2. Na2CrO4 is extracted with water, thereby leaving Fe2O3 (insoluble) behind and unconverted ore.
3. 2Na2CrO4 + H2SO4  Na2Cr2O7 + Na2SO4 + H2O
4. This solution is concentrated, when Na 2SO4 crystallizes out. On further concentration, Na 2Cr2O7 crystals
are obtained.
5. The hot saturated solution of Na 2Cr2O7 is mixed with KCl. NaCl precipitates out from the hot solution,
which is filtered off. On cooling the mother liquor, crystals of K 2Cr2O7 separates out.
Properties:
It is orangered coloured crystalline compound. It is moderately soluble in cold water but freely soluble in hot
water. It melts at 398°C. On heating strongly, it decomposes liberating oxygen.
2K2Cr2O7  2K2CrO4 + Cr 2 O 3 + 3/2O2
On heating with alkalies, it is converted to chromate, i.e. the colour changes from orange to yellow. On
acidifying, yellow colour again changes to orange.
K2Cr2O7 + 2KOH  2K2CrO4 + H2O
Cr 2 O2−
7 + 2OH  2CrO 2−
4 + H2O
Orange Yellow
2CrO 2−
4 + 2H+  Cr 2 O2−
7 + H2O
Yellow Orange
In alkaline solution, chromate ions are present while in acidic solution, dichromate ions are present. Potassium
dichromate reacts with hydrochloric acid and evolves chlorine.
K2Cr2O7 + 14HCl  2KCl + 2CrCl3 + 7H2O + 3Cl2
It acts as a powerful oxidising agent in acidic medium (dilute H 2SO4).
Cr 2 O2−
7 + 14H+ + 6e  2Cr3+ + 7H2O
The oxidation state of Cr changes from +6 to +3. Some typical oxidation reactions
are given below:
(i) Iodine is liberated from potassium iodide.
K2Cr2O7 + 4H2SO4  K2SO4 + Cr2(SO4)3 + 4H2O + 3[O]
[2KI + H2SO4 + [O]  K2SO4 + I2 + H2O]  3
K2Cr2O7 + 6KI + 7H2SO4  4K2SO4 + Cr2(SO4)3 + 7H2O + 3I2
The equation in terms of electron method may also be written as
Cr 2 O2−
7 + 14H+ + 6e  2Cr3+ + 7H2O
6I  3I2 + 6e
Cr 2 O2−
7 + 14H+ + 6I  2Cr3+ + 3I2 + 7H2O
(ii) Ferrous salts are oxidised to ferric salts.
K2Cr2O7 + 4H2SO4  K2SO4 + Cr2(SO4)3 + 4H2O + 3[O]
[2FeSO4 + H2SO4 + [O]  Fe2(SO4)3 + H2O]  3
K2Cr2O7 + 6FeSO4 + 7H2SO4  3Fe2(SO4)3 + Cr2(SO4)3 + 7H2O + K2SO4
India’s first Colour STUDY MATERIALS for JEE, NEET ,OUAT & OTHERS ENTRANCE EXAMINATION Page 44
2−
or 6Fe2+ + Cr 2 O 7 + 14H+  6Fe3+ + 2Cr3+ + 7H2O
(iii) Sulphites are oxidised to sulphates
K2Cr2O7 + 4H2SO4  K2SO4 + Cr2(SO4)3 + 4H2O + 3[O]
[Na2SO3 + [O]  Na2SO4]  3
K2Cr2O7 + 3Na2SO3 + 4H2SO4 + 3Na2SO4  K2SO4 + Cr2(SO4)3 + 4H2O
2− 2− 2−
or Cr 2 O 7 + 3 SO 3 + 8H+  3 SO 4 + 2Cr3+ + 4H2O
(iv) H2S is oxidised to sulphur
K2Cr2O7 + 4H2SO4 + 3H2S  K2SO4 + Cr2(SO4)3 + 7H2O + 3S
2−
or Cr 2 O 7 + 3H2S + 8H+  2Cr3+ + 7H2O + 3S
(v) SO2 is oxidised to H2SO4
K2Cr2O7 + 4H2SO4  K2SO4 + Cr2(SO4)3 + 4H2O + 3[O]
[SO2 + [O] + H2O  H2SO4]  3
K2Cr2O7 + H2SO4 + 3SO2  K2SO4 + Cr2(SO4)3 + H2O
2− 2−
or Cr 2 O 7 + 3SO2 + 2H+  2Cr3+ + 3 SO 4 + H2O
When the solution is evaporated, chromealum is obtained.
(vi) It oxidises ethyl alcohol to acetaldehyde and acetaldehyde to acetic acid.
C2H5OH ⃗
[O] CH3CHO [⃗ O] CH3COOH
Ethyl alcohol Acetaldehyde Acetic acid

It also oxidises nitrites to nitrates, arsenites to arsenates, thiosulphate to sulphate and sulphur (S 2 O2−
3 +

O  SO 2−
4 + S), HBr to Br2, HI to I2, etc.

Chromyl chloride test


This is a test of chloride. When a mixture of a metal chloride and potassium dichromate is heated with
concentration H2SO4, orange red vapours of chromyl chloride are evolved.
K2Cr2O7 + 2H2SO4  2KHSO4 + 2CrO3 + H2O
[NaCl + H2SO4  NaHSO4 + HCl]  4
[CrO3 + 2HCl  CrO2Cl2 + H2O]  2
K2Cr2O7 + 6H2SO4 + 4NaCl  2KHSO4 + 4NaHSO4 + 2CrO2Cl2 + 3H2O
When chromyl chloride vapours are passed through NaOH solution, yellow coloured solution is obtained.
4NaOH + CrO2Cl2  Na2CrO4 + 2NaCl + 2H2O
Yellow solution

29 OXIDES AND HALIDES OF TIN AND LEAD


CHLORIDES OF TIN
Stannous chloride, SnCl2
Preparation:
Tin dissolved in hot concentrated hydrochloric acid yields SnCl 22H2O on concentrating and crystallization.
Sn + 2HCl + 2H2O  SnCl2.2H2O + H2
Properties:
The hydrated salt on heating forms the oxychloride.
SnCl22H2O  Sn(OH)Cl + HCl + H2O
This hydrolysis can be prevented by the presence of excess HCl, with some pieces of tin added.
Aqueous and nonaqueous solutions of tin(II) salts are capable of acting as reducing agents, but they must be
stored under an inert atmosphere because air oxidation is spontaneous and rapid.
Sn2+(aq) + ½ O2(g) + 2H+(aq)  Sn4+ (aq) + H2O(l) ; (E° = 1.08 V)
Stannous chloride reacts with NaOH forming a white precipitate of tin(II) hydroxide which dissolves in excess
of NaOH forming sodium stannite.

India’s first Colour STUDY MATERIALS for JEE, NEET ,OUAT & OTHERS ENTRANCE EXAMINATION Page 45
SnCl2 + 2NaOH  Sn(OH)2 + 2NaCl
Sn(OH)2 + 2NaOH  Na2SnO2 + 2H2O
Sodium stannite
Sodium stannite is oxidised by atmospheric oxygen to form sodium stannate, Na 2SnO3. From a solution of
stannous chloride, H2S precipitates brown SnS, soluble in ammonium polysulphides.
SnCl2 + H2S  SnS + 2HCl
SnS + (NH4)2S2  (NH4)2SnS3 (ammonium thiostannate)
SnCl2 is a powerful reducing agent, as the following reactions illustrate.
2FeCl3 + SnCl2  2FeCl2 + SnCl4
2HgCl2 + SnCl2  SnCl4 + Hg2Cl2 (white precipitate of mercurous chloride)
Hg2Cl2 + SnCl2  2Hg + SnCl4 (hot condition)
Grey
2 KMnO4 + 16HCl + 5SnCl2  2KCl + 2MnCl2 + 8H2O + 5SnCl4
K2Cr2O7 + 14HCl + 3SnCl2  2KCl + 2CrCl3 + 7H2O + 3SnCl4
2CuCl2 + SnCl2  2CuCl + SnCl4
White ppt
HNO3 + 6HCl + 3SnCl2  NH2–OH + 2H2O + 3SnCl4
Hydroxylamine
Stannous chloride reduces nitro compounds to amino compounds and iodine to iodides.
C6H5–NO2 + 6HCl + 3SnCl  C6H5–NH2 + 3SnCl4 + 2H2O
2
I2 + 2HCl + SnCl2  SnCl4 + 2HI
Anhydrous stannous chloride, a glassy substance is prepared by heating tin in a stream of HCl or with
mercuric chloride.
Sn + 2HCl  SnCl2 + H2
Sn + HgCl2  SnCl2 + Hg
Excess
Anhydrous SnCl2 forms a dimer (Sn2Cl4) in the vapour, dissolves in organic solvents and forms many addition
compounds with NH3. e.g. SnCl22NH3.
In aqueous and nonaqueous solutions Sn(II) forms trihalo complexes, such as [SnCl 3], where the pyramidal
structure indicates the presence of a stereochemically active lone pair. The [SnCl 3] ion can serve as a soft
donor to dmetal ions. One unusual example of this ability is the red cluster compound Pt 3Sn8Cl20, which is
trigonal bipyramidal.
Uses:
As a reducing agent in the laboratory, as a mordant in dyeing and in the preparation of purple of Cassius.
Stannic chloride, SnCl4 (Butter of Tin)
Preparation:
Dry Cl2 gas when passed over heated tin in a retort forms SnCl 4.
Sn + 2Cl2  SnCl4
Liquid SnCl4 is thus collected in a cooled receiver protected from moisture.
It is also made by heating Sn with excess of HgCl 2.
Sn + 2HgCl2  SnCl4 + 2Hg
Another method of preparation is from heated SnO 2 by passing Cl2 + S2Cl2 (sulphur monochloride) vapour over
it.
2SnO2 + 3Cl2 + S2Cl2  2SnCl4 + 2SO2
It is also obtained by the removal of tin from (i.e., detinning of) scrap tin plates by chlorine.

Properties:
It is a colourless fuming liquid, soluble in organic solvents and volatile in nature. These properties indicate its
covalent nature. It forms hydrates with a limited quantity of water but undergoes hydrolysis with excess of
water.

India’s first Colour STUDY MATERIALS for JEE, NEET ,OUAT & OTHERS ENTRANCE EXAMINATION Page 46

H 2O
SnCl4 (lim ited quantity ) SnCl43H2O, SnCl45H2O, SnCl46H2O
SnCl45H2O is known as ‘butter of tin’ or ‘oxymuriate of tin’. It is used as a mordant and also for weighing
silk.
SnCl4 + H2O Sn(OH)Cl3 + HCl
Sn(OH)Cl3 + 3H2O Sn(OH)4 + 3HCl
This hydrolytic reaction is slow, reversible and can be suppressed by HCl, with which the following reaction

occurs: SnCl4 + 2Cl  [SnCl6]2. Salts with this ion e.g. (NH4)2SnCl6 are known as chlorostannates. Other
addition compounds are obtained with NH3, PCl5 etc., e.g. SnCl4 4NH3. The structure of SnCl4 is
Cl
Sn
Cl Cl
Cl
The tetrachloride, bromide and iodide of tin are molecular compounds, but the tetrafluoride has a structure
consistent with it being an ionic solid because the small F  ion permits a six coordinate structure.
Uses:
Butter of tin is used as a mordant and for weighing silk.

OXIDES OF TIN
Tin(II) oxide, SnO
Preparation:
SnO is precipitated by boiling stannous chloride solution with sodium carbonate or by heating the hydroxide
or oxalate in absence of air.
SnCl2 + Na2CO3  2NaCl + SnO + CO2
SnC2O4  SnO + CO + CO2
When freshly precipitated, the oxide has the composition 2SnO.2H2O.

Properties:
It is usually an olive green powder, which gives greyish crystals in contact with water. When heated in air, it
forms the dioxide. Both the oxide and hydrated oxide dissolve in acids forming stannous salts and in alkalies,
forming stannites.
Uses:
SnO acts as strong reducing agent.
2SnO + 2NaOH  Na2SnO2 + Sn + H2O

Tin(IV) oxide, SnO2


SnO2 occurs in nature as tinstone or cassiterite.
Preparation:
It is easily obtained by the combustion of tin in air, by ignition of metastannic acid produced from the action
of nitric acid on tin.
Cold and dilute nitric acid reacts with tin forming stannous nitrate, while concentrated nitric acid attacks tin
with the formation of hydrated stannic oxide.
4Sn + 10HNO3  4Sn(NO3)2 + 3H2O + NH4NO3
Sn + 4HNO3  SnO2.H2O + 4NO2 + H2O
(hydrated stannic oxidealso known as meta stannic acid)

Properties:
It is a soft, white solid sparingly soluble in water and acids except concentrated sulphuric acid but readily
soluble in fused alkalies to form stannate.
SnO2 + 2NaOH  Na2SnO3 + H2O

India’s first Colour STUDY MATERIALS for JEE, NEET ,OUAT & OTHERS ENTRANCE EXAMINATION Page 47
Uses:
Tin dioxide is used as a polishing powder and the name “putty powder” and for making milky glass and white
glazes for tiles and enamels.

CHLORIDES OF LEAD
Lead chloride, PbCl2
Preparation:
Prepared by slow direct combination or by the action of boiling concentrated HCl on lead (its oxide or
carbonate).
Pb + Cl2 ⃗
Δ PbCl2
Pb + 2HCl  PbCl2 + H2
PbO + 2HCl  PbCl2 + H2O
The usual method of preparation is to precipitate PbCl 2 as a white crystalline precipitate by adding a soluble
chloride to a lead salt solution.
Pb(NO3)2 + 2NaCl  PbCl2 + 2NaNO3.
Properties:
It is sparingly soluble in cold water but more soluble in hot water. In concentrated solutions of Cl  ions, it
dissolves forming complex ions, [PbCl3] and [PbCl4]2.
Lead tetrachloride, PbCl4
Preparation:
This is made by dissolving PbO2 in ice-cold concentrated HCl. Concentrated H2SO4 decomposes ammonium
chloroplumbate to yield PbCl4.
PbO2 + 4HCl  PbCl2 + Cl2 + 2H2O
(NH4)2[PbCl6] + H2SO4  PbCl4 + (NH4)2SO4 + 2HCl
Properties:
PbCl4 is a yellow oily liquid. It is heavy and dissolves in organic solvents. It is a covalent and unstable
compound, readily decomposes on heating.
PbCl4 ⃗
Δ PbCl2 + Cl2
It is easily hydrolysed by water and forms a double salt with NH4Cl.
PbCl4 + 2H2O  PbO2 + 4HCl
Lead tetrabromide and tetraiodide are unknown, so the dihalides dominate the halogen chemistry of lead.
OXIDES OF LEAD
Lead monoxide, PbO
It naturally occurs as lead ochre (an of various fine earths or days that contain ferric oxide, red, yellow or
brown pigment. The colour of this oxide is yellow or red depending on the mode of preparation.
Preparation:
When lead is gently heated in air, yellow powder is formed as the monoxide, called massicot. When heating is
continued it melts and on cooling gives the reddishyellow scales of litharge. These differ only in crystalline
structure. The transition temperature being 558°C. Lead monoxide can also be prepared by thermal
decomposition of lead nitrate as well as lead carbonate..
2Pb(NO3)2 ⃗
Δ 2PbO + 4NO2 + O2
PbCO3 ⃗Δ PbO + CO2
Dry air has no action on lead, but in moist air it tarnishes, forming a film of oxide first and finally basic
carbonate, which protects it from further action. On heating in air or oxygen, it forms litharge, PbO. But
prolonged heating gives red lead, Pb3O4.
India’s first Colour STUDY MATERIALS for JEE, NEET ,OUAT & OTHERS ENTRANCE EXAMINATION Page 48
2Pb + O2  2PbO
6PbO + O2  2Pb3O4
Properties:
(i) At room temperature, it is a yellow amorphous powder that is insoluble in water but dissolves in acids as
well as alkalies.
PbO + 2HCl  PbCl2 + H2O
PbO + 2NaOH  Na2PbO2 + H2O
Thus, it behaves as an amphoteric oxide. The acidic properties being rather feeble.
(ii) It is easily reduced to the metallic state by hydrogen, carbon or carbon monoxide.
(iii) In the red form of PbO, the Pb(II) ions are four coordinate but the O2 ions around the Pb(II) lie in a
square.
Uses:
Used in paints, in the vulcanisation of rubber and in the preparation of red lead and lead salts.
Lead dioxide, PbO2
Preparation:
(i) Action of cold concentrated nitric acid on red lead gives lead nitrate in solution while lead dioxide is
thrown as a chocolate powder.
Pb3O4 + 4HNO3  PbO2 + 2Pb(NO3)2 + 2H2O
(ii) Action of powerful oxidizing agents like chlorine, bromine or bleaching powder on alkaline lead salt
solution.
Pb(OH)2 + Cl2  PbO2 + 2HCl
Pb(C2H3O2)2 + Ca(OCl)Cl  PbO2 + 2CH3COOH + CaCl2

Properties:
(i) It is a chocolate coloured powder insoluble in water and dilute acids.
(ii) It liberates oxygen on gentle heating.
2PbO2  2PbO + O2
(iii) At 440°C, it is converted into red lead, Pb3O4.

3PbO2 ⃗ 440o C Pb3O4 + O2


(iv) PbO2 is an amphoteric oxide.
PbO2 + 4HCl  PbCl2 + Cl2 + 2H2O
2PbO2 + 2H2SO4  2PbSO4 + 2H2O + O2
(v) It is a good oxidizing agent. It oxidizes manganous salts to pink permanganic acid when boiled in nitric
acid solution.
2MnSO4 + 5PbO2 + 6HNO3  2HMnO4 + 2PbSO4 + 3Pb(NO3)2 + 2H2O
(vi) In alkaline medium, chromium hydroxide is oxidized to yellow chromate by PbO 2.
2Cr(OH)3 + 10KOH + 3PbO2  2K2CrO4 + 3K2PbO2 + 8H2O
The maroon form of lead(IV) oxide, PbO 2, crystallizes in the rutile structure. This oxide is a component of
the cathode of a leadacid battery.
Uses:
In the laboratory, lead dioxide finds application as an oxidizing agent. It is also used as the cathode in lead
storage battery.
Red lead, Pb3O4
Preparation:
Roasting of litharge in air at 450°C gives a bright red powder.
6PbO + O2  2Pb3O4
It is also known as sindur.
Properties:
(i) Sparingly soluble in water but dissolves in dilute nitric acid.
Pb3O4 + 4HNO3  2Pb(NO3)2 + PbO2 + 2H2O
The above reaction indicates that red lead may be considered as plumbous ortho plumbate, 2PbO.PbO 2.
India’s first Colour STUDY MATERIALS for JEE, NEET ,OUAT & OTHERS ENTRANCE EXAMINATION Page 49
(ii) It turns dark when heated but restores the original colour on cooling. At about 550°C, it decomposes
giving off oxygen.
Pb3O4 ⃗
Δ 3PbO + ½ O2
(iii) It reacts with concentrated HCl and sulphuric acid liberating chlorine and oxygen respectively.
Pb3O4 + 8HCl  3PbCl2 + 4H2O + Cl2
2Pb3O4 + 6H2SO4  6PbSO4 + 6H2O + O2
Red lead, Pb3O4 contains Pb(IV) in an octahedral arrangement and Pb(II) in
an irregular six coordinate environment. The assignment of different oxidation number to the lead in
these two sites is based on the shorter PbO distance for the atom identified as Pb(IV).
Uses:
Red lead, mixed with linseed oil is extensively used as a red paint and also for plumbing work.

30 ORGANOMETALLIC COMPOUNDS
These compounds constitute a broad class of substances in which carbon atom is directly
bonded to a metal. Thus organic compounds in which metal atom is directly linked to carbon atom
are known as organometallic compounds.
For example, NaCCNa is an organometallic compound as sodium is directly linked to carbon whereas
C2H5ONa, Ti(OC2H5)4 are not organometallic compounds since the metal atom is linked to carbon through oxygen.
Some representative organometallic compounds are
C2H5MgBr  Ethyl magnesium bromide
(C2H5)2Zn  Diethyl zinc
(CH3)2Cd  Dimethyl cadmium
C6H5Li  Phenyl lithium
(C2H5)4Pb  Tetraethyl lead
Alkyl or aryl magnesium halides (RMgX or ArMgX) are also called as Grignard reagents.

CLASSIFICATION OF ORGANOMETALLIC COMPOUNDS

(i) Ionic compounds of electropositive metals:


These compounds are mostly formed between the electropositive metals and the carbon
compounds which are mostly acidic in nature. Thus organometallic compounds of alkali metals and
alkaline earth metals consist of ions or ion pairs.
− + − 2+ −
R Na R Mg X ¿ Alkyl magnesium ¿ ¿¿
Alkyl sodium halide ¿
R−C ≡ C Na R Zn2 + R−
− + −
Sodium alkynide dialkyl zinc
These compounds are normally soluble in hydrocarbon solvent. They are very reactive towards air and water.
The stability of these compounds depends upon the structure of the carbon containing part of the compound.

(ii) bonded complexes:


In a bonded complex, a metal and a carbon atom of the ligand are joined together with a sigma bond. This
means that the ligand contributes 1 electron and is therefore called one electron donor. Tetramethyl tin, (CH 3)4 Sn and
trimethyl aluminium, (CH3)3Al are bonded organometallic compounds. (CH3)3Al exists as dimer and has structure
analogous to diborane. Two methyl groups act as bridges between two aluminium atoms.
H3 C CH3 CH3
Al Al
H3 C CH3 CH3
Trimethyl aluminium

India’s first Colour STUDY MATERIALS for JEE, NEET ,OUAT & OTHERS ENTRANCE EXAMINATION Page 50
(iii) complexes:
These are organometallic compounds which involve the use of bonds present in organic compounds. For
example, Zeise’s salt, ferrocene and dibenzene chromium are organometallic compounds of this type. In all these
compounds the electrons of the organic compound interact with the metal ion and thus occupy one of the
coordination sites. For example in ferrocene and dibenzene chromium, the iron and chromium atoms are sandwiched
between two aromatic rings.
H H 
C

Cl C K+
Pt H H
Cl Cl

Zeise’s salt K [Pt Cl3 (2 C2H4)]

Fe
2 Cr

5
Fe rro c e ne Fe (   C 5 H5 )2
6
dibe nze ne c hro m ium Cr (  C 6 H6 )2

[bis cyclopentadienyl iron]


The number of carbon atoms involved in the formation of complexes with metals is indicated by the power
of  (eta). For example, ferrocene is represented as Fe[5(C6H6)7] indicating that 5carbon atoms or cyclo pentadienyl
x

anion are involved in complexation with the metal. Similarly one can write dibenzene chromium as [Cr(6C6H6)2]
indicating that all the six carbons of benzene are involved in complexation with chromium.

(iv) Metal Carbonyls:


These are the complexes where carbon of carbon monoxide donates a pair of electrons to the metal. Nickel
carbonyl and iron carbonyl are the common examples.

CO CO CO
CO CO CO

M CO Fe Ni

CO CO CO CO
CO CO
CO CO
[M (C O ) ]
6
(M = C r, Mo , W) [Fe (C O ) ]
5 [Ni (C O ) ]
4

In metal carbonyl the oxidation state of the metal is zero. These metal carbonyls may be monomeric bridged
or polynuclear.

BONDING IN ORGANOMETALLIC COMPOUNDS


(a) Bonding in metal carbonyls:
The metal-carbon bond in metal carbonyls has σ as well as  character.

India’s first Colour STUDY MATERIALS for JEE, NEET ,OUAT & OTHERS ENTRANCE EXAMINATION Page 51
(i) overlap :
In a sigma bonded complex, the lone pair of electrons is present on the bonding orbital of carbon monoxide.
This bonding orbital containing lone pair interacts with the empty d-orbital of the metal to form a metal-carbon
bond as shown below:
M C  O : M C  O
Me ta l o rb ita l Bo nd ing orb ita l of Bond ing in
c a rb on m onoxid e m e ta l c a rb on

(ii) overlap :
In addition to this, the antibonding orbitals of CO can also overlap with the filled
dorbitals of the metal resulting in back bonding as explained earlier. Thus metal carbonyls become much more stable
compounds due to multiple bonding in them.

M C O M CO

Me ta l Antib ond ing o rb ita l Ba c kb o nd ing in


o rb ita l o f c a rb on m o no xid e m e ta l c a rb o nyls

It is important to note here that the σ - bond is in the nodal plane of the σ -electrons whereas π -
overlap is perpendicular to the nodal plane.
(b) Bonding of alkenes to a transition metal
The bonding of alkenes to a transition metal to form complexes has two components.
First, the electron density of the alkene overlaps with a σ -type vacant orbital on the metal atom. Second is the
back bonding formed by the flow of electron density from a filled
dorbital on the metal into the vacant  x  antibonding molecular orbital on the carbon atom as shown.
C
C
M
C C

 o ve rla p

C C

M M
C C

SYNTHESIS OF ORGANOMETALLIC COMPOUNDS


1. By Direct Reaction of Metals :-
(a) Tetramethyl- and Tetraethyl-lead (which are used as anti-knock additives to petrol) are manufactured by the
reaction between the alkyl chloride and a lead-sodium alloy.
4C2H5CLCL + 4NA/PB  (C2H5)4PB + 3PB + 4NACL
(b) Zinc alkyls are prepared by heating an alkyl iodide with zinc-copper couple.
2RI + 2Zn  2RZnI  R2Zn + ZnI2
(c) Grignard reagent is generally prepared by reaction between magnesium and alkyl halide in dry, alcohol-free
ether.
RX + MG  RMGX
(d) Alkyl-Lithium compounds are prepared by direct displacement.

N2
NC4H9CL + 2LI hexane NC4H9LI + LICL

2. By Using an Alkylating agent

Grignard reagent and Alkyl lithium on reaction with most of the metal and non-metal halides in the presence
of ether as solvent yield other organometallic compounds.

India’s first Colour STUDY MATERIALS for JEE, NEET ,OUAT & OTHERS ENTRANCE EXAMINATION Page 52
PCL3 + 3C6H5MGCL ⃗
Ether
P(C6H5)3 + 3MGCL2
Triphenyl Phosphine
SNCL4 + 4NC4H9LI  (NC4H9)4 SN + 4LICL
Tetrabutyl Tin
PBCL4 + 4C2H5MGBR  (C2H5)4PB + 4MGBRCL
Tetraethyl Lead
CDCL2 + 2RMGCL  R2CD + 2MGBRCL2
Dialkyl Cadmium

3. Preparation of complexes :-

(a) Preparation of Ferrocene


Ferrocene is prepared by the reaction of Iron Oxide with Cyclopentadiene at 523 K.
2C5H5 + FEO ⃗
523 K [(C3H5)2FE] + H2O
Ferrocene
It can also be prepared by treatment of Cyclopentadienyl Magnesium Bromide with Ferrous Chloride.
C5H5MGBR + FECL2  [(C5H5)2FE] + 2MGBRCL
Cyclopenta Dienyl Ferrocene
Magnesium Bromide

(b) Preparation of Zeise’s salt


It can be prepared by the action of Ethene with Tetrachloro Platinate (IV) anion.
C2H4 + [PTCL4]2  [C2H4PTCL3] + CL
Ethene Tetrachloro Zeise’s Salt
Platinate (IV)

(c) Preparation of Dibenzene Chromium


Dibenzene Chromium is prepared by allowing the reaction between Benzene and Chromium vapours at
high temperatures.
2C6H6 + CR(Vapour)  (C6H6)2CR
Dibenzene Chromium
Dibenzene Chromium can also be prepared by reacting Chromium (iii) Chloride and Benzene in presence of
Aluminium and Aluminium Chloride.

CRCL3 + 2C6H6 + 3ALCL3 ⃗


Al (C6H6)2CR + 3 AlCl−4

4. Preparation of metal carbonyls :-


(a) By Direct Synthesis

Room temp .
NI + 4CO 1 atm . Pr NI(CO)4
200o C

FE + 5CO 100 atm FE(CO)5

(b) Reduction of Metal compounds in presence of CO



(C 2 H 5 )3 Al
CRCL3 + 6CO in ether CR(CO)6

India’s first Colour STUDY MATERIALS for JEE, NEET ,OUAT & OTHERS ENTRANCE EXAMINATION Page 53
(c) From Fe ( CO )5 :- since CO Ligands in Fe ( CO )5 are labile, Fe ( CO )5 may be used to form certain
carbonyls eg.,
100o C

MCL6 + 3FE(CO)5 ether M(CO)6 + 3FECL2 + 9CO
(M = W, MO)

Applications of Organometallic Compounds


1. The soluble organometallic complexes of transition metals act as homogeneous catalyst. Some examples are:
(a) Selective hydrogenation of certain double bonds using Wilkinson’s catalyst, (Ph 3P)3RhCl.
(b) (ET3P)2NICL2 acts as catalyst for the isomerization of alkenes.
2. Organometallic compounds can also act as heterogeneous catalyst. For example, ZeiglerNatta catalyst (a
solution of TiCl4, containing triethyl aluminium) for the polymerization of ethylene and the other alkenes.
3. Organometallic compounds of Magnesium (rmgx), Cadmium (r2cd) and Lithium (rli) are extensively used in
organic synthesis.
4. Tetra ethyl lead is used as an antiknock compound.
5. A number of organometallics also find application in agriculture. For example, ethyl mercury chloride,
C2H5HgCl is used as a fungicide for the protection of young plants and seeds against fungal infection.
6. Aryl arsenic compounds are used as chemotherapeutic agents.
7. Silicon rubbers, because of their high thermal stability, resistance to oxidation and chemical attack, are used in
modern surgery for the purpose of production of artificial body parts.
8. Since Ni(CO)4 is decomposed to metallic nickel, it is used in the production of nickel by Mond’s process.
9. Ethyl Mercury Chloride is used to prevent the infection in young plants.

Question 1: What is the ratio of uncomplexed to complexed Zn2+ ion in a solution that is 10 M in NH 3, if the stability
constant of [Zn(NH3)4]2+ is 3  109?
(A) 3.3  109 (B) 3.3  1011
(C) 3.3  10 14
(D) 3  1013
Solution:
Zn2+ + 4NH3 [Zn(NH3)4]2+
[ Zn(NH 3 )4 ] 2+
2+ 4
Kf = [ Zn ] [ NH 3 ]
[ Zn 2+ ] 1 1
2+
= = =
[ Zn( NH 3 )4 ] K f [ NH 3 ] 3×10 ×( 10 )4
4 9
3.3  1014
 (c)

Question 2: Oxidation state of Cr in the following complex is


4+
O2

(H2O)4Cr Cr(H2O)4
O O

(A) +3 (B) +6
(C) +4 (D) +5
Solution:
Among the bridging ligands, O2 is a neutral ligand and [OO]2 is a bidentate negative ligand. Since the net
charge over the complex is 4+, each chromium atom has an oxidation state of +3.
 (a)

India’s first Colour STUDY MATERIALS for JEE, NEET ,OUAT & OTHERS ENTRANCE EXAMINATION Page 54
Question 3: A compound has an empirical formula CoCl 3.5NH3. When an aqueous solution of this compound is mixed
with excess of silver nitrate, 2 moles of AgCl precipitates per mole of the compound. On reaction with excess
+
of HCl, no NH 4 is detected. Hence the compound is
(A) [CO(NH3)5CL2]CL (B) [CO(NH3)5CL]CL2
(C) [CO(NH3)5CL3] (D) [CO(NH3)4CL2]CL.NH3
Solution:
NH +
As the moles of AgCl precipitated is 2 and no 4 is detected on reaction with excess of HCl,
so the compound would be [Co(NH3)5Cl]Cl2.
 (B)
Question 4: If excess of AgNO3 solution is added to 100 ml of 0.024 M solution of dichlorobis(ethylene
diamine)cobalt(III) chloride, how many moles of AgCl will be precipitated?
(A) 0.0012 (B) 0.0016
(C) 0.0024 (D) 0.0048

Solution:
The formula of the complex is [CoCl2(en)2]Cl.
[CoCl2(en)2]Cl + AgNO3  AgCl + [CoCl2(en)2]NO3
Moles of complex = Moles of AgCl = 100  103  0.024 = 0.0024
 (c)
Question 5: In nitroprusside ion, the iron and NO exist as Fe II and NO+ rather than FeIII and NO. These forms can be
distinguished by
(a) Estimating the concentration of iron
(b) Measuring the concentration of cn
(c) Measuring the solid state magnetic moment
(d) Thermally decomposing the compound
Solution:
Nitroprusside ion is [Fe(CN)5NO]2. If the central atom iron is present here in Fe 2+ form, its effective atomic
number will be 262 + (62) = 36 and the distribution of electrons in valence orbitals (hybridised and
unhybridized) of the Fe2+ will be
3d 4s 4p 4d

2 3
d sp hybridization
It has no unpaired electron. So this anionic complex is diamagnetic. If the nitroprusside ion has Fe 3+ and NO,
the electronic distribution will be such that it will have one unpaired electron i.e. the complex will be
paramagnetic.
3d 4s 4p 4d

2 3
d sp hybridization
Thus, magnetic moment measurement establishes that in nitroprusside ion, the Fe and NO exist as Fe II and
NO+ rather then FeIII and NO.
 (c)
Question 6: Which of the given statements is not true for the following reaction?
[Cu(H2O)4]2+ + 4NH3 [Cu(NH3)4]2+ +4H2O
(a) It is a ligandsubstitution reaction.
(b) NH3 is a relatively strong field ligand while H2O is a weak field ligand.
(c) During the reaction, there is a change in colour from light blue to dark blue.
(d) [Cu(NH3)4]2+ has a tetrahedral structure and is paramagnetic.
Solution:
For 29Cu, outermost shell has electronic configuration of 3d104s1.
Electronic configuration of Cu2+ = 3d94s°
India’s first Colour STUDY MATERIALS for JEE, NEET ,OUAT & OTHERS ENTRANCE EXAMINATION Page 55
3d9 4so

But due to strong field ligand (NH3), unpaired electron of 3d9 jumps to 4p. Hence,
3d8 4so 4p1

XX XX XX XX

 Hybridisation of Cu2+ in [Cu(NH3)4]2+ is dsp2 and it gives the square planar geometry.
 (d)

Question 7: The IUPAC name of the complex [Ni(C4H7O2N2)2] formed from the reaction of Ni 2+ with dimethyl
glyoxime is
(a) Bis(methylgloxal)nickel(II) (b) Bis(dimethyloxime)nickelate(IV)
(c) Bis(2,3butanedioldioximato)nickel(II) (d) Bis(2,3butanedionedioximato)nickel(II)
Solution:
O H O

H3CC=N N=CCH3
Ni
H3CC=N N=CCH3

O H O
Bis(dimethylglyoximato)nickel(II)
or Bis(2,3butanedionedioximato)nickel(II)
 (d)

Question 8: The hybridization states of the central atoms in the complexes [Fe(CN) 6]3, [Fe(CN)6]4 and
[Co(NO2)6]3 are
(a) d2sp3, sp3 and d4s2 respectively (b) d2sp3, sp3d and sp3d2 respectively
(c) d2sp3, sp3d2 and dsp2 respectively (d) all d2sp3
Solution:
[Fe(CN)6]3 has Fe3+ ion.
Outermost shell of Fe3+ (Z = 26) has the following configuration,
3d6 4s2

and outermost shell of Fe3+ has the configuration:


3d5 4s2

But due to strong field ligand (CN), the pairing of electrons takes place.
3d 4s 4p

XX XX XX XX XX XX

 Hybridisation is d2sp3.
[Fe(CN)6]4 has Fe2+ ion.
Outermost shell of Fe2+ has the following configuration:
3d6 4so

But due to strong field ligand (CN), the pairing of electrons takes place.
3d 4s 4p

XX XX XX XX XX XX

 Hybridisation is d2sp3.
[Co(NO2)6]3 has Co3+ ion.
Outermost shell of Co (Z = 24) has the following configuration,
3d7 4s2

India’s first Colour STUDY MATERIALS for JEE, NEET ,OUAT & OTHERS ENTRANCE EXAMINATION Page 56
and outermost shell of Co3+ has the configuration:
3d6 4so

But due to strong field ligand (NO2), pairing of electrons occurs.


3d 4s 4p
XX XX XX XX XX XX

Hybridisation is d2sp3.
 (d)
Question 9: Which of the following statement is incorrect?
(a) Most fourcoordinated complexes of Ni2+ ions are square planar rather than tetrahedral.
(b) The [Fe(H2O)6]3+ ion is more paramagnetic than the [Fe(CN)6]3 ion.
(c) Square planar complexes are more stable than octahedral complexes.
(d) The [Fe(CN)6]4 ion is paramagnetic but [Fe(CN)6]3 ion is diamagnetic.
Solution:
[Fe(CN)6]4 has Fe2+ and have no unpaired electron, so it is diamagnetic and [Fe(CN) 6]3 has Fe3+ and having one
unpaired electron, so it is paramagnetic.
 (d)
Question 10: Which of the following statement is correct?
(a) [Co(NH3)6]2+ is oxidized to diamagnetic [Co(NH3)6]3+ by the oxygen in air.
(b) [Fe(CN)6]3 is stable but [FeF6]3 is unstable.
(c) [NiCl4]2 is unstable with respect to [NiBr4]2.
(d) None of these.
Solution:
With the promotion of one 3delectron to 5s or 4d, it becomes loosely bonded to the nucleus and hence, it
may easily be removed and so, Co(II) will easily be oxidised into Co(III).
 (a)

Question 11:Show that all octahedral complexes of Ni 2+ must be outer-orbital complexes.


Solution:
The electric configuration of Ni2+ ion (3d8) indicates that two inner d-orbitals (3d-orbitals) cannot be made
available to allow d2sp3 hybridisation. However, by using two 4d-orbitals, sp3d2 hybridisation may be possible.
3d 4s 4p 4d
II 2+
[Ni L6] ion XX XXXXXX XXXX

L L L L L L
3 2
sp d hybridisation-outer
orbital octahedral complex
Question 12:The magnetic moment of [MnBr 4]2– is 5.9 B.M. What is the geometry of this
complex ion?
Solution:
Since the coordination number of Mn2+ ion in this complex ion is 4, it may be either tetrahedral (sp 3
hybridisation) or square planar (dsp2 hybridisation) as shown below at (b) and (c). But the fact that the
magnetic moment of the complex ion is 5.9 B.M. shows that it should be tetrahedral in shape rather than
square-planar.

India’s first Colour STUDY MATERIALS for JEE, NEET ,OUAT & OTHERS ENTRANCE EXAMINATION Page 57
3d 4s 4p
2+
(a) Mn ion
(3d5)

3d 4s 4p
(b) [MnBr4]2–
(sp3 hybridisation– ×× ×× ×× ××
tetrahedral shape)
(n = 5,  = 5.9 B.M) Br – Br – Br – Br –

sp3 hybridisation-
tetrahedral shape
3d 4s 4p
(c) [MnBr4]2–
(dsp2 hybridisation– ×× ×× ×× ×× ××
square planar shape)
(n=3, =3.8 B.M)
Br – Br – Br– Br –

dsp2 hybridisation–
square planar shape
Question 13: How would you account for the following?
(a) [Fe(CN)6]3– is weakly paramagnetic while [Fe(CN)6]4– is diamagnetic.
(b) Ni(CO)4 possesses tetrahedral geometry while [Ni(CN) 4]2– is square planar.
(c) [Ni(CN)4]2– is diamagnetic while [NiCl4]2– is paramagnetic.
Solution:
(a) [Fe(CN)6]3– involves d2sp3 hybridization.
3d 4s 4p
3+ 5
Fe (d )

[Fe(CN)6]3– XX XX XX XX XX XX

2 3
d sp hybridization
One dorbital is singly occupied, hence it is weakly paramagnetic in nature.
[Fe(CN)6]4– involves also d2sp3 hybridization but it has Fe2+ ion as central ion.
3d 4s 4p
2+ 6
Fe (d )

[Fe(CN)6]4– XX XX XX XX XX XX

2 3
d sp hybridization
All orbitals are doubly occupied, hence it is diamagnetic in nature.
(b) In the formation of Ni(CO)4, nickel undergoes sp3 hybridization, hence it is tetrahedral in shape.
3d 4s 4p
8 2
Ni 3d 4s

Ni(CO)4 XX XX XX XX

3
sp hybridization
(c) In [Ni(CN)4]2– ion, Ni2+ undergoes dsp2 hybridization, hence it is square planar in shape.
3d 4s 4p
2+
Ni

[Ni(CN)4]2– XX XX XX XX

2
dsp hybridization
In [Ni(CN)4]2–, all orbitals are doubly occupied, hence it is diamagnetic ; while in [NiCl 4]2–, two orbitals are
singly occupied, hence it is paramagnetic in nature.

India’s first Colour STUDY MATERIALS for JEE, NEET ,OUAT & OTHERS ENTRANCE EXAMINATION Page 58
3d 4s 4p
[NiCl4]2– XX XX XX XX

3
sp hybridization

Strong field ligands like CN–, CO, en, NO 2 have very strong electron donating tendency, hence electrons
of central metal ion pair up against Hund’s rule and low spin complexes are formed.

Question 14: A metal complex having composition Cr(NH 3)4 Cl2Br has been isolated in two forms (A) and (B). The
form (A) reacts with AgNO3 to give a white precipitate readily soluble in dilute aqueous ammonia, whereas
(B) gives a pale yellow precipitate soluble in concentrated ammonia. Write the formula of (A) and (B) and
state the hybridisation of chromium in each. Calculate the magnetic moments (spin-only value).
Solution:
Complex, Cr(NH3)4Cl2Br, has two isomers. Since, coordination number of Cr is six, the two forms may be
represented in the following way
[Cr(NH3)4ClBr]Cl [Cr(NH3)4Cl2]Br
(A) (B)
[Cr(NH3)4)ClBr] Cl + AgNO3  [Cr(NH3)4ClBr]NO3 + AgCl
(A) White ppt

AgCl + 2NH4OH  Ag(NH3)2Cl + 2H2O


Soluble
[Cr(NH3)4 Cl2] Br + AgNO3  [Cr(NH3)4Cl2] NO3 + AgBr
(B) Pale yellow
AgBr + 2NH4OH  Ag(NH3)2 Br + 2H2O
Soluble
The state of hybridisation of chromium in both the complexes is d 2sp3. Chromium is in trivalent state (Cr3+).
3d 4s 4p
XX XX XX XX XX XX

2 3
d sp hybridization

As three unpaired electrons are present, the magnetic moment = B.M. √ n(n+2)
= 3 × 5 B.M.

= 3.87 B.M.
Question 15: Platinum (II) forms square planar complexes and platinum (IV) gives octahedral complexes. How many
geometrical isomers are possible for each of the following complexes? Describe their structures.
(a) [Pt (NH3)3 Cl]+ (b) [Pt (NH3) Cl5]–
(c) [Pt (NH3)2 ClNO2] (d) [Pt(NH3)4 ClBr]2+
Solution:
(a) No isomers are possible for a square planar complex of the type MA 3B.
+
Cl NH3

Pt

H3N NH3

(b) No isomers are possible for an octahedral complex of the type MAB 5.
NH3 –
Cl Cl

Pt

Cl Cl
Cl

(c) Cis and trans isomers are possible for a square planar complex of the type MA 2BC.

India’s first Colour STUDY MATERIALS for JEE, NEET ,OUAT & OTHERS ENTRANCE EXAMINATION Page 59
Cl NH3 Cl NH3

Pt Pt

O 2N NH3 H3N NO2


cis trans
(d) Cis and trans isomers are possible for an octahedral complex of the type MA 4BC.
NH3
Cl
H3N Cl
H3N NH3

Pt
Pt

H3N Br
H3N NH3
NH3
Br
cis trans

Question 16: Write the IUPAC name of the following complexes:


(i) [Co(en)3]Cl3 (ii) [Co(C2O4)3]3
(iii) [NH3)5CoO2Co(NH3)5]4+ (iv) [Cr(NH3)6] [Co(CN)6]
(v) (Ph4As)2 [PtCl2HCH3]
Solution:
(i) Tris(ethylediamine)cobalt(III) chloride
(ii) Trioxalatoccobaltate(III) ion
(iii) Decaammineperoxodicobalt(III) ion
(iv) Hexaamminechromium(III) hexacyanocobaltate(III)
(v) Tetraphenylarsenium dichlorohydridomethylplatinate(II)

Question 17: A solution containing 0.319 g of complex CrCl 3.6H2O was passed through cation exchanger and the
solution given out was neutralised by 28.5 ml of 0.125 M NaOH. What is the correct formula of complex?
Solution:
The Cl atoms outside the coordination sphere will be ionised to produce the acid, HCl.
Thus, milliequivalent of Cl ions outside = milliequivalent of HCl formed
= milliequivalent of NaOH used
= 28.5  0.125
= 3.56
0.319
266 .5 mole or 1.197 millimole of complex produce 3.56 milliequivalent or millimoles of Cl . Thus,
1 mole of complex will give 3 mole of Cl , i.e. all the three Cl atoms are outside the coordination sphere.
Thus, the complex is [Cr(H2O)6]Cl3.

Question 18: (A), (B) and (C) are three complexes of chromium (III) with the empirical formula H 12O6Cl3Cr. All the
three complexes have water and chloride ions as ligands. Complex (A) does not react with concentrated
H2SO4, whereas complexes (B) and (C) lose 6.75% and 13.5% of their original weight, respectively, on
treatment with concentrated H2SO4. Identify the octahedral complexes (A), (B) and (C).
Solution:
(A) : [Cr(H2O)6]Cl3 (Violet)
(B) : [Cr(H2O)5Cl]Cl2.H2O (Green) Molecular weight = 266.5
(C) : [Cr(H2O)4Cl2]Cl.2H2O (Dark green)
Compound (A) contains six water molecules as coordinated water and thus, does not lost H 2O on treatment
with H2SO4. Compound (B) contains five water molecules as coordinated water and one molecule as lattice
water which is lost to H2SO4 showing a loss of 18 g out of 266.5 g, i.e., 6.75% loss. Similarly, compound (C)
contains four coordinated water molecules and two molecules of lattice water, which are taken out by H 2SO4
to show a loss of 13.5%.

India’s first Colour STUDY MATERIALS for JEE, NEET ,OUAT & OTHERS ENTRANCE EXAMINATION Page 60
Question 19: (a) Write down the IUPAC nomenclature of the given complex along with its hybridisation and structure
K2[Cr(NO)(NH3)(CN)4] ;  = 1.73 B.M.
(b) Draw the structures of [Co(NH3)6]3+, [Ni(CN)4]2 and [Ni(CO)4]. Write the hybridisation of atomic
orbitals of the transition metal in each case.
Solution:
(a) Potassium amminetetracyanonitrosoniumchromate(I)
Cr is in +1 oxidation state and possess d2sp3 hybridisation with one unpaired electron.

 = n(n+2) = 1(1+2) = 3 = 1.73 BM.


√ √ √
(b) [Co(NH3)6] : Co3+ is d2sp3 hybridised to show octahedral shape.
3+

NH3
H3N NH3
3+
Co

H3N NH3
H3N
2 2+ 2
[Ni(CN)4] : Ni is dsp hybridised to show square planar shape.

NC CN
2+
Ni


NC CN
3
[Ni(CO)4]: Ni is sp hybridised to show tetrahedral shape.
CO

Ni
OC CO

CO

An application of co-ordination compounds is there use as catalyst which serves to


DO YOU alter the rate of chemical reaction.
It play a key role in production of Polyethylene and
KNOW Polypropylene.

MIND MAP

Salt

Single salt Double salt


For example, K2SO4 e.g. K2SO4.Al2(SO4)3.24H2O

India’s first Colour STUDY MATERIALS for JEE, NEET ,OUAT & OTHERS ENTRANCE EXAMINATION Page 61
Complex salt like
[Pt(NH3)4] [PtCl4]
n, simple anion or simple cation andItcomplex anion
is formed or dcomplex
by the Ligands
and cation
when
f block and
metals complex
combine anion.
mostly.with metals (d and f block) form complexes. by IUPAC
Nomenclature Spectrochemical
rule series for ligand field is,
I, Br, Cl, F < H2O < NH3 < en, etc.

Bonding Isomerism

CFT Structural Stereoisomerism


Werner’s theory VBT Hybridisation theory

Ligand Hydrate Linkage Ionisation Coordination Coordination sphere Polymerization

Geometrical Optical

Cis Optically active


Trans Optically inactive
can be shown by AB2D2, AB4D2, ABCDEFG etc. (d, l )
can be shown by AB2D2, AB4 D2, ABCDEFG etc.

1. Fe3+ complexes are more stable than Fe2+ complexes. Why?

2. Write IUPAC name of linkage isomer of [Cr(H2O)5SCN]2+.

3. What is the coordination number of Fe in [Fe(EDTA)]?

4. What is the hybridisation of Ni in [Ni(CN)4]2 complex? Also predict the shape of the complex.

5. Explain the following:


(i) ferrocyanide ion is diamagnetic and ferricyanide ion is paramagnetic.
(ii) aqueous solution of potassium ferricyanide does not give test for ferric ions.
(iii) Pt(NH3)2Cl2 has a square planar geometry but Ni(CO)4 has tetrahedral geometry.
India’s first Colour STUDY MATERIALS for JEE, NEET ,OUAT & OTHERS ENTRANCE EXAMINATION Page 62
(iv) [Sc(H2O)6]3+ is colurless while [Ti(H2O)6]3+ is coloured.

6. What is meant by isomerism? Give examples of each of the following in relation to coordinate compounds:
(i) ionization isomerism
(ii) linkage isomerism
(iii) geometrical isomerism
(iv) optical isomerism

7. Give a chemical test to distinguish between [Co(NH 3)5Br]SO4 and [Co(NH3)5SO4]Br. What kind of isomerism
do they exhibit?

8. What is polymerisation isomerism? Discuss with one example.

9. Complete the following statements for the coordination entity (complex ion) [CrCl 2(OX)2]3
(a) OX is abbreviation for ____________
(b) The oxidation number of chromium is ____________
(c) The coordination number of chromium is ____________
(d) ____________ is a bidentate ligand.

10. Deduce the magnetic behaviour of each of the following:


(i) [Cr(NH3)5Cl]2+ (ii) Fe(CO)5
(Atomic numbers of Cr = 24, Fe = 26)

Multiple Choice Questions For IIT JEE (Part-1)


1. The IUPAC name for [Pt(NH3)3(Br)(NO2)(Cl)]Cl is
(a) triamminechlorobromonitroplatinum(IV) chloride
(b) triamminebromochloronitroplatinum(IV) chloride
(c) triaminenitrochlorobromoplatinum(IV) chloride
(d) triamminechloronitrobromoplatinum(IV) chloride

2. The IUPAC name of the complex Ni[C4H7O2N2)2], formed by the reaction between Ni 2+ and
dimethylglyoxime, is
(a) bis(methylglyoxal)nickel(II)
(b) bis(dimethylyoxime)nickel
(c) bis(2,3butanediol dioximato)nickel(II)
(d) bis(2,3butanedione dioximato)nickel(II)

3. Which of the following complex ions obeys Sidgwick’s effective atomic number (EAN) rule?
(a) [Fe(CN)6]3 (b) [Fe(CN)6]4

India’s first Colour STUDY MATERIALS for JEE, NEET ,OUAT & OTHERS ENTRANCE EXAMINATION Page 63
(c) [Cr(NH3)6]3+ (d) [Ni(en)3]2+

4. Which one of the following coordination compounds exhibits ionization isomerism?


(a) [Cr(NH3)6]Cl3 (b) [Cr(en)3Cl3]
(c) [Cr(en)3]Cl3 (d) [Co(NH3)5Br]SO4

5. The pair [Co(NH3)5NO3]SO4 and [Co(NH3)5SO4]NO3 will exhibit


(a) hydrate isomerism (b) linkage isomerism
(c) ionization isomerism (d) coordinate isomerism

6. Which of the following will have three stereoisomeric forms?


(i) [Cr(NO3)3(NH3)3] (ii) K3[Co(C2O4)3]
(iii) K3[Co(C2O4)2Cl2] (iv) [Co(en)2ClBr]
(where en = ethylene diamine)
(a) (iv) and (iii) (b) (iv) and (i)
(c) (iii) and (ii) (d) (i) and (ii)

7. A coordination compound of cobalt has the molecular formula containing five ammonia molecules, one nitro
group and two chlorine atoms for one cobalt atom. One mole of this compound produces three moles of ions
in an aqueous solution. The aqueous solution on treatment with an excess of AgNO 3 gives two moles of AgCl
as a precipitate. The formula of this complex would be
(a) [Co(NH3)4NO2Cl] [NH3Cl] (b) [Co(NH3)Cl][ClNO2]
(c) [Co(NH3)5NO2]Cl2 (d) [Co(NH3)5][(NO2)2Cl2]

8. The hybridization states of the central atom in the complex ions [FeF 6]3, [Fe(H2O)6]3+ and [Ni(NH3)6]2+ are
(a) sp3d2, dsp2 and d4s2 respectively (b) all 3d24s4p3
3 2
(c) all 4s4p 4d (d) sp3d2, dsp3 and p4d2 respectively

2− 3−
9. Among TiF 6 , CoF 6 , CuCl2 and NiCl 2−
4 (atomic numbers of Ti = 22, Co = 27, Cu = 29,
Ni = 28), the colourless species are

(a) CoF 63− and NiCl 2−


4 (b) TiF 2−
6 and CoF 6
3−

2−
(c) Cu2Cl2 and NiCl 4 (d) TiF 3−
6 and Cu2Cl2

10. Among [Ni(CO)4], [Ni(CN)4]2 and [NiCl4]2


(a) [Ni(CO)4] and [Ni(CN)4]2 are diamagnetic and [Ni(CN)4]2 is paramagnetic.
(b) [NiCl4]2 and [Ni(CN)4]2 are diamagnetic and [Ni(CO)4] is paramagnetic.
(c) [Ni(CO)4] and [Ni(CN)4]2 are diamagnetic and [NiCl4]2 is paramagnetic.
(d) [Ni(CO)4] is diamagnetic and [NiCl4]2 and [Ni(CN)4]2 are paramagnetic.

11. Zeise salt, an organometallic compound, has the formula



(a) (C6H6)2Cr+ AlCl 4 (b) (CH3)2AlF
(c) Ni(CO)4 (d) K+[PtC2H4Cl3] H2O

12. Which of the following mixtures is known as ZeiglerNatta catalyst?


(a) Al(OCH3)3 + TiCl4 (b) (C2H5)3Al + TiCl4
(c) [(CH3)2CHO]3Al + TiCl4 (d) LiCH3 + TiCl4

13. Which of the following is not an organometallic compound?


(a) Ferrocene (b) Ruthenocene
(c) Beryllium acetylacetonate (d) bis(benzene)chromium

India’s first Colour STUDY MATERIALS for JEE, NEET ,OUAT & OTHERS ENTRANCE EXAMINATION Page 64
14. Which is not a bonded complex?
(a) Zeise salt (b) Ferrocene
(c) bis(benzene) chromium (d) Tetraethyl lead

15. Which of the following is an organometallic compound?


(a) Lithium methoxide (b) Lithium acetate
(c) Lithium dimethylamide (d) Methyllithium

16. One mole of the complex compound [Co(NH 3)5Cl3] gives 3 moles of ions on dissolution in water. One mole of
the same complex reacts with two moles of AgNO 3 solution to yield two moles of AgCl(s). The structure of
the complex is
(a) [Co(NH3)4Cl]Cl2.NH3 (b) [Co(NH3)5Cl]Cl2
(c) [Co(NH3)3Cl3].2NH3 (d) [Co(NH3)4Cl2]Cl.NH3

17. Which of the following aquated metal ions has the highest paramagnetism?
(a) [Cr(H2O)6]3+ (b) [Fe(H2O)6]2+
(c) [Cu(H2O)6]3+ (d) [Zn(H2O)2]2+

18. Which of the following species is expected to be colourless?


(a) [Ti(H2O)6]3+ (b) [Ti(NO3)4]
+
(c) [Cr(NH3)2] (d) [Fe(CN)6]4

19. Tollen’s reagent contains


(a) AgNO3 (b) AgOH
(c) [Ag(NH3)2]+ (d) [Ag(NO3)2]+

20. The ferric ion is detected by the formation of a Prussian blue precipitate on addition of potassium ferrocyanide
solution. The formula of the Prussian blue precipitate is

(a) Fe4III [ FeII (CN )6 ]3 (b) Fe3II [ Fe III (CN )6 ]4

(c) KFeIII [ Fe II (CN )6 ] (d) KFe II [ Fe III (CN )6 ]

21. When a solution of potassium ferricyanide is added to an aqueous solution of ferrous sulphate, a deep blue
colour, known as Turnbull’s blue, is produced. The formula of the compound responsible for this deep blue
colour is

(a) KFeIII [ Fe II (CN )6 ] (b) Fe4III [ FeII (CN )6 ]3

(c) KFeII [ Fe III (CN )6 ] (d) Fe3III [ FeIII (CN )6 ]3

22. The coordination number of Ag in [Ag(NH3)2]Cl is


(a) one (b) two
(c) three (d) zero

23. The formation of the complex ion [Co(NH 3)6]3+ involves sp3d2 hybridization of Co3+. Hence, the complex ion
should possess
(a) octahedral geometry (b) tetrahedral geometry
(c) square planar geometry (d) tetragonal geometry

24. The compounds [Cr(H2O)6]Cl3, [Cr(H2O)5Cl]Cl2.H2O and [Cr(H2O)4Cl2]Cl.2H2O exhibit


(a) linkage isomerism (b) geometrical isomerism
(c) ionization isomerism (d) hydrate isomerism

India’s first Colour STUDY MATERIALS for JEE, NEET ,OUAT & OTHERS ENTRANCE EXAMINATION Page 65
25. Which of the following complex compounds exhibits cistrans isomerism?
(a) [PtCl2(NH3)2] (b) [PdCl2BrI]
(c) [Pt(NH3)(py)(Cl)(Br)] (d) All of these

Multiple Choice Questions For IIT JEE (Part-2)

1. Which of the following species has the electron configuration [Ar]3d 4?


(a) Ti (b) V2+
2+
(c) Cr (d) Fe2+

2. For transition elements, which of the following occurs as the effective nuclear charge increases?
(a) Both the atomic radius and density increases.
(b) Both the atomic radius and density decreases.
(c) The atomic radius increases and the density decreases.
(d) The atomic radius decreases and the density increases.

3. What is the name of the complex [Ni(H2O)4(NH2CH2CH2NH2)]SO4.5H2O as per IUPAC rules?


(a) Aquaethylenediaminenickel(II) sulfate 1water
(b) Tetraaquaethylenediaminenickel(II) sulfate 5water
(c) Tetraaquabis(ethylenediamine)nickel(II) sulfate 5water
(d) Tetraaquabis(ethylenediamine)nickel(III) sulfate 5water

4. How many unpaired electrons are present in the high spin form of the [CoF 6]3 complex and which metal
orbitals are used in bonding?
(a) 0 unpaired electrons and 4s, 4p and 4d orbitals to give sp3d2 hybridization.
(b) 4 unpaired electrons and 4s, 4p and 4d orbitals to give sp3d2 hybridization.
(c) 0 unpaired electrons and 3d, 4s and 4p orbitals to give d2sp3 hybridization.
(d) 4 unpaired electrons and 3d, 4s and 4p orbitals to give d2sp3 hybridization.

5. The complex [Ni(CN)4]2 is diamagnetic and the complex [NiCl4]2 is paramagnetic. What can you conclude
about their molecular geometries?
(a) Both complexes have square planar geometries.
(b) Both complexes have tetrahedral geometries.
(c) [NiCl4]2 has a square planar geometry while [Ni(CN) 4]2 has a tetrahedral geometry.
(d) [NiCl4]2 has a tetrahedral geometry while [Ni(CN)4]2 has a square planar geometry.

6. What is the expected order for increasing octahedral (0) crystal field splitting for the ligands: I , F, H2O,
NH3, en and CO?
(a) I < F < H2O < NH3 < en < CO (b) F < I < NH3 < en < CO < H2O
(c) I < F < H2O < CO < NH3 < en (d) CO < en < NH3 < H2O < F < I

3− 4− 3−
7. The hybridization states of the central atom in the complexes Fe(CN )6 , Fe(CN )6 and Co( NO 2 )6
are
(a) d2sp3, sp3d2 and dsp2 respectively (b) d2sp3, sp3d and sp3d2 respectively
2 3 3 4 2
(c) d sp , sp and d s respectively (d) all d2sp3
8. Tetrahedral complexes of the types of [Ma 4] and [Ma3b] (where M = metal, a, b = achiral ligands) are not able
to show optical isomerism because
(a) these molecules/ions possesses Cn axis of symmetry.
(b) these molecules ions possesses a plane of symmetry and hence are achiral.
India’s first Colour STUDY MATERIALS for JEE, NEET ,OUAT & OTHERS ENTRANCE EXAMINATION Page 66
(c) these molecules possesses a centre of symmetry.
(d) these molecules/ions have nonsuperimposable mirror images

9. Which of the following is an organometallic compound?


(a) Cyclobutadiene
(b) Thiotetraamminecopper(II)sulphate
(c) Potassium tetrafluorooxochromate(V)
(d) Bis(cyclopentadienyl)iron(II)

10. The magnetic moment of a complex ion is 2.83 BM. The complex ion is
(a) [Cr(H2O)6]3+ (b) [Cu(CN)4]2
(c) [V(H2O)6]3+ (d) [MnCl4]2

11. The E° values for some transition metals are as follows:


Cr2+ | Cr = 0.9 V, Mn2+ | Mn = 1.2 V, Fe2+ | Fe = 0.4 V,
Cr3+ | Cr2+ = 0.4 V, Mn3+ | Mn2+ = + 1.5 V and Fe3+ | Fe2+ = 0.8 V
The correct statement is
(a) Fe3+ is more reducible than Cr3+ as well as Mn3+.
(b) Cr is a better reducing agent than Mn but lesser than Fe.
(c) Fe3+ is more reducible than Cr3+ but less than Mn3+.
(d) Fe2+ is a better oxidising agent than Mn2+ but lesser than Cr2+.

12. What is the least reducing agent of the following set: MnCl 2, Mn(OH)3, MnO2 and KMnO4?
(a) MnCl2 (b) Mn(OH)3
(c) MnO2 (d) KMnO4

13. Of the following, which ligand does not posses the name suggested by IUPAC when it acts as a ligand in
complex?
(a) H2O, aqua (b) NH3, ammonia
(c) CO, carbonyl (d) F, fluoro

14. Which pair of isomers illustrates the concept of ionisation isomers?


(a) [Cr(SCN)(NH3)5]2+ and [Cr(NCS)(NH3)5]2+
(b) [CoCl(NH3)5]SO4 and [Co(SO4)(NH3)5]Cl
(c) cis[PtCl2(NH3)2] and trans[PtCl2(NH3)2]
(d) (+)[Co(en)3]3+ and ()[Co(en)3]3+

15. Which of the following complex ion is most likely to be colorless?


(a) [Co(H2O)6]2+ (b) [Mn(CN)6]3
(c) [CrCl3(H2O)3] (d) [Ag(NH3)2]+

16. Which of the following complex has five unpaired electrons?


(a) [Mn(H2O)6]2+ (b) [Mn(CN)6]3
(c) [CrCl3(H2O)3] (d) [Ag(NH3)2]+

17. Which ion would you expect to have the largest splitting of dorbitals?
(a) [Fe(CN)6]4 (b) [Fe(CN)6]3
2+
(c) [Fe(H2O)6] (d) [Fe(H2O)6]3+

18. Which of the following coordination compound is incapable of showing geometrical isomerism?
(a) [PtCl2(NH3)2] (b) [CoCl2(NH3)4]+
(c) [Co(NO2)3(NH3)3] (d) [Co(en)3]3+

India’s first Colour STUDY MATERIALS for JEE, NEET ,OUAT & OTHERS ENTRANCE EXAMINATION Page 67
19. A six coordinate complex of formula CrCl 3.6H2O has green colour. A 0.1 M solution of the complex when
treated with excess of AgNO3 gave 28.7 g of white precipitate. The formula of the complex would be
(a) [Cr(H2O)6]Cl3 (b) [Cr(H2O)5Cl]Cl2.H2O
(c) [Cr(H2O)4Cl2]Cl.2H2O (d) [Cr(H2O)3Cl3].3H2O

20. Which of the following statements is incorrect?


(a) Most of the fourcoordinated complexes of Ni2+ ions are square planar rather than tetrahedral.
(b) The [Fe(H2O)6]3+ ion is more paramagnetic than the [Fe(CN)6]3 ion.
(c) Square planar complexes are more stable than octahedral complexes.
(d) The [Fe(CN)6]4 ion is paramagnetic but [Fe(CN)6]3 is diamagnetic.

21. Which of the following is a highspin (spinfree) complex?


(a) [Co(NH3)6]3+ (b) [Fe(CN)6]4
(c) [CoF6]3 (d) [Zn(NH3)6]2+

22. Which of the following ligands are bidentate or tridentate ligands, capable of forming chelate rings?
(i) NH2CH2CH2NH2 (ii) CH3CH2CH2NH2
 
(iii) NH2CH2CH2NHCH2CO2 (iv) NH2CH2CH2NH3
(a) (i) and (iii) (b) (ii) and (iv)
(c) (i), (ii) and (iii) (d) (i), (ii), (iii) and (iv)
Consider the following isomers of [Co(NH 3)4Br2]+. The black sphere represents Co, gray spheres represent
NH3 and unshaded spheres represent Br.

(1) (2) (3) (4)


Answer the following questions:

23. Which of the following are cisisomers?


(a) isomers (1) and (2) (b) isomers (1) and (3)
(c) isomers (2) and (4) (d) isomers (3) and (4)

24. Which of the following are transisomers?


(a) isomers (1) and (2) (b) isomers (1) and (3)
(c) isomers (2) and (4) (d) isomers (3) and (4)

25. Whish structures are identical?


(a) None of the structures are identical
(b) Structure (1) = structure (2) and structure (3) = structure (4)
(c) Structure (1) = structure (3) and structure (2) = structure (4)
(d) Structure (1) = structure (4) and structure (2) = structure (3)

EXERCISE - I
ONE OR MORE THAN ONE CHOICE CORRECT

1. What is/are the coordination number(s) of Au in the complexes formed by Au?


(a) 6 (b) 4
(c) 5 (d) 2

2. If coordination number of cobalt in its complex is six then oxidation number of Co may be …… or …… .
(a) +2 (b) +3
(c) +4 (d) +6

3. K4[Fe(CN)6] is

India’s first Colour STUDY MATERIALS for JEE, NEET ,OUAT & OTHERS ENTRANCE EXAMINATION Page 68
(a) Outer orbital octahedral complex (b) High spin complex
(c) Low spin complex (d) Inner orbital octahedral complex

4. The compounds which dissolve in NH3 and form the soluble colourless complexes are
(a) CuSO4 (b) AgCl
(c) ZnSO4 (d) AgI

5. The oxides which dissolve in alkali and form the soluble complexes are
(a) ZnO (b) As2O3
(c) B2O3 (d) none

6. The sulphides which dissolve in yellow ammonium sulphide and give colourless soluble complexes are
(a) SnS (b) As2S3
(c) CuS (d) none of these

7. The dorbitals involved in sp3d2 or d2sp3 hybridisation of the central metal ion are
d x 2− y 2 d xy
(a) (b)

(c)
d yz (d)
d
z2

8. [Cu(NH3)4]SO4 possesses
(a) dsp2 hybridisation (b) tetrahedral geometry
(c) sp3 hybridisation (d) square planar

9. The complex(es) which is/are blue in colour


(a) Fe4[Fe(CN)6]3 (b) Zn2[Fe(CN)6]
(c) Cu2[Fe(CN)6] (d) Fe3[Fe(CN)6]2

10. Which of the following are coordination isomers of [Co(NH3)6] [Cr(CN)6]?


(a) [Cr(NH3)6] [Co(CN)6] (b) [Cr(NH3)2(CN)4] [Co(CN)4(NH3)2]
(c) [Cr(NH3)3(CN)3] [Co(NH3)3(CN)3] (d) none of these

11. Identify the complexes which are expected to be coloured.



(a) Ti(NO3)4 (b) [Cu(CNCH3)]+ BF 4
(c) [Cr(NH3)6]Cl3 (d) Fe4[Fe(CN)6]3

12. Bidentate legends are


2−
(a) C2 O 4 (b) en
(c) DMG (d) Gly

13. Which of the following can show co–ordination isomerism?


(a) [Cu(NH3)4] [PtCl4] (b) [Fe(NH3)6]2 [Pt(CN)6]3
(c) [Co(NH3)6] [Cr(C2O4)3] (d) [Pt(en)3] (SO4)2

14. Which is/are correct statement(s)?


(a) [Co(en)3] [Cr(CN)6] will display co–ordination isomerism.
(b) [Mn(CO)5(SCN)] will display linkage isomerism.
(c) [Co(NH3)5(NO3)]SO4 will display ionisation isomerism.
(d) None is correct.

15. Which of the following are paramagnetic?


(a) [Ni(CN)4]2– (b) [NiCl4]2–

India’s first Colour STUDY MATERIALS for JEE, NEET ,OUAT & OTHERS ENTRANCE EXAMINATION Page 69
(c) [CoF6]3– (d) [Co(NH3)6]3+

EXERCISE – II
MATCH THE FOLLOWING
Note: Each statement in column I has one or more than one match in column II.
1.

Column I Column II
(Complex) (Hybridisation of central atom / ion)
I. [Ni(H2O)6]2+ (A) sp3
II. [Ni(CN)4]2 (B) sp3d2
III. [Ni(CO)4] (C) d2sp3
IV. [Cu(NH3)4]2+ (D) dsp2
2.

Column I (Ligands) Column II (Type of ligands)


I. (en) (A) monodentate
II. EDTA (B) bidentate
III. Trien (C) tetradentate
IV. gly (D) hexadentate
3.
Column I (Complex) Column II (No. of unpaired electrons)
I. [Fe(H2O)6]2+ (A) 0
II. [Fe(H2O)6]3+ (B) 2
III. [Fe(CN)6]4– (C) 5
IV. [Ni(H2O)4]2+ (D) 4

_REASONING TYPE_

Directions: Read the following questions and choose


(A) If both the statements are true and statement-2 is the correct explanation of statement-1.
(B) If both the statements are true but statement-2 is not the correct explanation of statement-1.
(C) If statement-1 is True and statement-2 is False.
(D) If statement-1 is False and statement-2 is True.

1. Statement-1: Tetrahedral complexes with chiral structure exhibit optical isomerism.


Statement-2: They lack plane of symmetry.
(a) (A) (b) (B) (c) (C) (d) (D)

2. Statement-1: Oxidation state of Fe in Fe(CO)5 is zero.


Statement-2: EAN of Fe in all its complexes is 36.
(a) (A) (b) (B) (c) (C) (d) (D)

3. Statement-1: Zeise’s salt contains C2H4 molecule as one of the ligands.


Statement-2: Zeise’s salt is an organometallic compound.
(a) (A) (b) (B) (c) (C) (d) (D)

4. Statement-1: [Co(NH3)3Cl3] does not give white precipitate with AgNO3 solution.
Statement-2: Chlorine is not present in the ionisable part of the given complex.
(a) (A) (b) (B) (c) (C) (d) (D)

5. Statement-1: Transition metal ion forming octahedral complexes undergo sp 3d2 or d2sp3 hybridisation.

India’s first Colour STUDY MATERIALS for JEE, NEET ,OUAT & OTHERS ENTRANCE EXAMINATION Page 70
Statement-2: Strong field ligands force the unpaired electrons of central metal ion to pair up causing d 2sp3
hybridisation whereas weak field ligands cannot force the pairing and hence the metal ion
undergoes sp3d2 hybridisation.
(a) (A) (b) (B) (c) (C) (d) (D)

_LINKED COMPREHENSION TYPE_

The IUPAC rules of writing names of mononuclear coordination compounds are


as given below:
(a) If the compound is ionic, name of the cation is mentioned first followed by the name of the anion.
(b) For nonionic compounds the name of the complex is written in one word.
(c) The sequence of naming coordination sphere is to write the names of ligands in the alphabetical order
followed by the name of central metal atom / ion and then oxidation number of the metal in Roman
numeral.
(d) For the ligands carrying a negative charge the name of the ligand has a characteristic ending in ‘O’.
(e) For the ligands carrying a positive charge the name of the ligand has a characteristic ending of “ium”.
(f) For organic ligands their names are used as such.
(g) In case of anionic complexes the suffix ‘ate’ is attached to the name of central atom/ion.
(h) Numerical prefixes are used to indicate number of ligands.

1. What is the name of the ligand NO+?


(a) Nitronium (b) Hydrazinium
(c) Nitrosonium (d) Nitrosyl

2. The IUPAC name of [Ag(NH3)2]Cl is


(a) Amine silver chloride (b) Diammine silver chloride
(c) Diammine silver (I) chloride (d) Chloroamine silver

3. Iron (III) hexacyanoferrate (II) is:


(a) Fe4[Fe(CN)6]3 (b) Fe3[Fe(CN)6]2
(c) Fe(CN)6 (d) Fe[Fe(CN)6]
4. The correct structural formula for Aquabromobis(ethylenediamine)chromium(III) chloride is
(a) [CrBr2(H2O)2(en)]Cl (b) [CrBr2(H2O)2(en)]Cl2
(c) [CrBr(H2O)(en)2]Cl2 (d) [CrBr(H2O)(en)2]Cl2

EXERCISE – III
SUBJECTIVE PROBLEMS

1. One pink solid has the formula CoCl 3.5NH3.H2O. A solution of this salt is also pink and rapidly gives 3 mol
AgCl on titration with silver nitrate solution. When the pink solid is heated, it loses 1 mol H 2O to give a purple
solid with the same ratio of NH 3:Cl:Co.
The purple solid releases two of its chlorides rapidly; then, on dissolution and after titration with AgNO 3,
releases one of its chlorides slowly. Deduce the structures of the two octahedral complexes and draw and
name them.

2. The hydrated chromium chloride that is available commercially has the overall composition CrCl 3.6H2O. On
boiling a solution, it becomes violet and has a molar electrical conductivity similar to that of [Co(NH 3)6]Cl3. In
contrast, CrCl3.5H2O is green and has a lower molar conductivity in solution. If a dilute acidified solution of

India’s first Colour STUDY MATERIALS for JEE, NEET ,OUAT & OTHERS ENTRANCE EXAMINATION Page 71
the green complex is allowed to stand for several hours, it turns violet. Interpret these observations with
structural diagrams.

3. The complex first denoted [PtCl2(NH3)2] was identified as the trans isomer.
(The cis isomer was denoted by ). It reacts slowly with solid Ag 2O to produce [Pt(NH3)2(OH2)2]2+. This
complex does not react with ethylenediamine to give a chelated complex. Name and draw the structure of the
diaqua complex.

4. The ‘third isomer’ (neither  nor ) of composition Pt2Cl4.4NH3, is an insoluble solid which, when grounded
with AgNO3, gives a solution containing [Pt(NH 3)4) (NO3)2 and a new solid phase of composition Ag 2[PtCl4].
Give the structures and names of each of the three Pt(II) compounds.

5. A solution containing 2.665 g of CrCl 3.6H2O is passed through a cation exchanger. The chloride ions obtained
in solution were treated with excess of AgNO 3 to give 2.87 g of AgCl. Deduce the structure of complex
compound.

6. A rosecoloured compound (A) has the empirical formula CoCl 3.5NH3.H2O. Two moles of this compound
reacts with concentrated sulphuric acid to form HCl(g) and 1 mole of a new compound (B) with empirical
formula Co2(SO4)3.10NH3.5H2O. When this new compound (B) is dried at room temperature, it loses three
moles of water per mole of Co 2(SO4)3.10NH3.5H2O. Both complexes (A) and (B) have octahedral geometry.
State the significance of each observation and deduce the formula of the complexes (A) and (B)

7. Studies of a complex gave a composition corresponding to the formula CoBr(C 2O4).4NH3. Conductance
measurements indicate that there are two ions per formula unit. If calcium nitrate gives no immediate
precipitate of calcium oxalate, then give the structural formula of the octahedral complex? Write the structural
formula of an isomer of this complex.

8. Give the structural formula of these complexes.


(i) Tetraamminenickel(II) perchlorate
(ii) Hexaamminenickel(II) hexanitrocobalate(III)
(iii) Dicarbonylbis(triphenylphosphine)nickel(0)
(iv) Tetrakis(oxalato)dihydroxodichromium(III)
(v) ChlorothiocyanatoSbis(ethylenediamine)cobalt(II)
9. Write the IUPAC name of the following complexes.
(i) [Co(en)3]Cl3
(ii) [(NH3)5CoO2Co(NH3)5]4+
(iii) [Cr(NH3)6] [Co(CN)6]
(iv) [(NH3)5CoNH2Co(NH3)4H2O]Cl5

10. (i) In each of the following pair of complexes, choose the one that absorbs light at a longer wavelength.
(a) [Co(NH3)6]2+ , [Co(H2O)6]2+ (b) [FeF6]3, [Fe(CN)6]3
(c) [Cu(NH3)4]2+, [CuCl4]2
(ii) Magnetic moments of four complexes are given. Predict the type of hybridization in each of these complexes.
Examples Magnetic moment (in BM)
3+
[Cr(NH3)6] 3.57
3
[Fe(C2O4)3] 5.75
[Ni(CN)4]2 0
2
[MnCl4] 5.90

India’s first Colour STUDY MATERIALS for JEE, NEET ,OUAT & OTHERS ENTRANCE EXAMINATION Page 72
ANSWERS

1. Size of Fe3+ and Fe2+ are nearly same but the charge on Fe 3+ is greater than the charge on Fe 2+ hence more
charge density, if the metal ion have more charge density the complex is more stable.

2. Linkage isomer of [Cr(H2O)5SCN]2+ is [Cr(H2O)5NCS]2+


The IUPAC name of its is Pentaavaaisothiocyanatochromium(III) ion

3. Coordination number is 6.

4. dsp2. Square planar.

5. (i) In K4[Fe(CN)6]  no unpaired electrons (Fe in d2sp3 hybridisation)


In K3[Fe(CN)6]  one unpaired electron (Fe in d2sp3 hybridisation)
aqueous
(ii) K3[Fe(CN)6] 3K+ + [Fe(CN)6]3
(iii) Pt(NH3)2Cl2  dsp2 hybridisation
Ni(CO)4  sp3 hybridisation
(iv) [Sc(H2O)6]3+  no unpaired electron
[Ti(H2O)6]3+  one unpaired electron.

6. (i) Ionization isomerism [Co(NH3)5Br]SO4 and [Co(NH3)5SO4]Br


(ii) Linkage isomerism [Co(NH3)4SCN]2+
[Co(NH3)4NCS]2+
(iii) Optical isomerism

India’s first Colour STUDY MATERIALS for JEE, NEET ,OUAT & OTHERS ENTRANCE EXAMINATION Page 73
en
en
Cl Cl

Co
Co

Cl Cl
en
en
Mirror c is m irro r im a g e
c is o b je c t

o p tic a lly a c tive form s (no n-sup e rim p o sa b le )


Dic hlo ro b is (e thyle ne d ia m ine ) c o b a lt (III) io n

(iv) Geometrical isomerism


Cl

en
en
Cl Cl

en en
Co
Co Co

Cl Cl
en
en
Mirror cis mirror ima ge
cis object

Cl
optic a l y a ctive forms (non-superimposa ble )
Dic hlorobis (ethyle nediam in e) c oba lt (III) io n tra ns

7. [Co(NH3)5Br]SO4 + 2AgNO3  Ag2SO4


White ppt.
[Co(NH3)5SO4]Br + AgNO3  AgBr
Ionisation isomerism yellow ppt.

8. [Pt(NH3)4]2+ [PtCl4]2 and [Pt(NH3)2Cl2]

9. (a) oxalato (b) +3 (c) 6 (d) C2O42

10. Number of unpaired electron in Cr in the complex [Cr(NH 3)5Cl]2+ are 3.


 Paramagnetic behaviour.

r= n(n+2) = 3(3+2) = 15 B.M.


√ √ √
Number of unpaired electrons in Fe in the complex Fe(Co) 5 are 0.
 Diamagnetic behaviour.

Multiple Choice Questions For IIT JEE (Part-1)


1. (b) 2. (d) 3. (b) 4. (d) 5. (c)

6. (b) 7. (c) 8. (c) 9. (d) 10. (c)


11. (d) 12. (b) 13. (c) 14. (d) 15. (d)
16. (b) 17. (b) 18. (b) 19. (c) 20. (c)
21. (c) 22. (b) 23. (a) 24. (d) 25. (d)

Multiple Choice Questions For IIT JEE (Part-2)

1. (c) 2. (d) 3. (b) 4. (b) 5. (d)

India’s first Colour STUDY MATERIALS for JEE, NEET ,OUAT & OTHERS ENTRANCE EXAMINATION Page 74
6. (a) 7. (d) 8. (b) 9. (d) 10. (c)
11. (c) 12. (d) 13. (b) 14. (b) 15. (d)
16. (a) 17. (b) 18. (d) 19. (b) 20. (d)
21. (c) 22. (a) 23. (b) 24. (c) 25. (c)

EXERCISE – I
ONE OR MORE THAN ONE CHOICE CORRECT

1. (b, d) 2. (a, b) 3. (c, d) 4. (b, c) 5. (a, b, c)

6. (a, b) 7. (a, d) 8. (a, d) 9. (a, d) 10. (a, b)


11. (c, d) 12. (a,b,c,d) 13. (a, b, c) 14. (a, b, c) 15. (b, c)

EXERCISE – II
MATCH THE FOLLOWING

1. I  (B) ; II  (D) ; III  (A) ; IV  (D)


2. I  (B) ; II  (D) ; III  (C) ; IV  (B)
3. I  (D) ; II  (C) ; III  (A) ; IV  (B)

REASONING TYPE

1. (a) 2. (c) 3. (b) 4. (a) 5. (a)

LINKED COMPREHENSION TYPE

1. (c) 2. (c) 3. (a) 4. (c)

EXERCISE – III
SUBJECTIVE PROBLEMS

1. Pink: [Co(NH3)5(H2O)]Cl3 ; Petaammineaquacobalt(III) chloride


Purple: [CoCl(NH3)5]Cl2 ; Pentaamminechlorocobalt(III) chloride
2. Violet: [Cr(H2O)6]Cl3 ; Green: [Cr(H2O)5Cl]Cl2
Both complexes are octahedral.
3. transDiamminediaquaplatinum(II) ; Square planar complex.
4. [Pt(NH3)4] [PtCl4] ; Tetraammineplatinum(II) tetrachloroplatinate(II)
[Pt(NH3)4] (NO3)2 ; Tetraammineplatinum(II) nitrate
Ag2[PtCl4] ; Silver tetrachloroplatinate(II)
5. [Cr(H2O)5Cl]Cl2.H2O
6. (A): [Co(NH3)5H2O]Cl3.
(B): [Co(NH3)5H2O]2(SO4)3.3H2O.

India’s first Colour STUDY MATERIALS for JEE, NEET ,OUAT & OTHERS ENTRANCE EXAMINATION Page 75
7. [Co(C2O4)(NH3)4]Br, NH4[Co(C2O4)(NH2)(NH3)2Br]
8. (i) [Ni(NH3)4](ClO4)2
(ii) [Ni(NH3)6]3[Co(NO2)6]2
(iii) [Ni(CO)2(PPh3)2]
OH 4

(C2O4)2Cr Cr(C2O4)2
(iv) OH
(v) [CoCl(SCN)(en)2]
9. (i) Tris(ethylenediamine)cobalt(III) chloride
(ii) Decaammineperoxodicobalt(III) ion
(iii) Hexaamminechromium(III) hexacyanocobaltate(III)
(iv) Pentaamminecobalt(III)amidotetraammineaquacobalt(III) chloride
10. (i) (a) [Co(H2O)6]2+ , (b) [FeF6]3 (c) [CuCl4]2
(ii) d2sp3, sp3d2, dsp2, sp3

India’s first Colour STUDY MATERIALS for JEE, NEET ,OUAT & OTHERS ENTRANCE EXAMINATION Page 76

You might also like